Exam practice

Réussis tes devoirs et examens dès maintenant avec Quizwiz!

How many times can you repeat dispense the following prescriptions? An NHS prescription for bendroflumethiazide with no mention of repeats. 0 1 2 3 4 5 6 7

0 NHS prescriptions are not repeatable.

How many times can you repeat dispense the following prescriptions? A private prescription for bendroflumethiazide with no mention of repeats. 0 1 2 3 4 5 6 7

0 Private prescriptions where the prescriber has not added the instruction to "repeat" are not valid for repeatable dispensing.

How many times can you repeat dispense the following prescriptions? A standard NHS prescription for bendroflumethiazide with "repeat x 2" stated in the body of the prescription. 0 1 2 3 4 5 6 7

0 Standard NHS prescriptions are not repeatable. In England the repeat dispensing service exists, but specific forms are used for this service and stating "repeat x 2" in the body of a standard prescription would not be suitable for dispensing.

How many NHS prescription charges does Yasmin®, an oral contraceptive pill levy (assuming the patient does NOT have an exemption card) for an NHS prescription written in England? 0 1 2 3 4

0 Yasmin® is an oral contraceptive pill and is free of charge

You owe a patient some medicine from dispensing a prescription. Which date should the following medicines be fully dispensed by? Morphine sulfate 10 mg/5 mL oral solution on a private prescription signed on 30/10/20. 27/11/20 28/11/20 21/01/21 24/01/21 30/04/21 01/05/21 25/06/21 30/10/21

30/04/21 Owings must be dispensed within the validity of a prescription, which is 6 months for this prescription. The appropriate date of a private prescription is the signature date.

You owe a patient some medicine from dispensing a prescription. Which date should the following medicines be fully dispensed by? Imipramine tablets 10 mg on an NHS prescription signed 30/10/20. 27/11/20 28/11/20 21/01/21 24/01/21 30/04/21 01/05/21 25/06/21 30/10/21

30/04/21 Owings must be dispensed within the validity of a prescription, which is 6 months for this prescription.

What is the last date you could dispense the following prescriptions? A private prescription for a Schedule 2 CD signed on 03/09/21. 30/06/21 28/07/21 30/09/21 30/03/22 30/06/22 30/06/26 Repeats are not allowed There is no legally defined final dispensing date, it is based on clinical judgement

30/09/21 Prescriptions for Schedule 2 CDS are valid for 28 days from the appropriate date, which in the case of a private prescription is the date of signing.

Regarding the dosing of ibuprofen, choose the appropriate dosing for the indication described. A 22-year-old female who presents to her GP because of her period pain which she describes as 6/10 on a pain scale. 200 mg TDS 200-400 mg TDS 300-400 mg TDS 400-600 mg single dose 1.6 g OD 1.2 g BD 1.6 g BD 2.4 g OD

300-400 mg TDS Moderate dysmenorrhoea is dosed initially at 300-400 mg 3-4 times a day according to the BNF, and as the GP can prescribe for the patient this dose can be prescribed.

Whilst working in a community pharmacy, an 18-year-old patient becomes unconscious and is not breathing. You have first aid training and are required to perform cardiopulmonary resuscitation (CPR). What is the recommended compression-ventilation ratio for CPR on this patient? 15:2 20:2 20:3 30:2 30:3

30:2 In regard to this question, the answer is 30 chest compressions followed by two rescue breaths for as long as you can, or until help arrives.

For the patients described, select the most likely contraindication or caution from the list below. Nitrazepam. 35-year-old male with unstable myasthenia gravis 58-year-old female who regularly drinks ginger tea 45-year-old female with blood pressure of 187/112 mmHg 38-year-old female with a haematoma on her upper arm 29-year-old male with an abnormally low neutrophil count 21-year-old male with type 1 diabetes mellitus 18-year-old male 72-year-old male with erectile dysfunction

35-year-old male with unstable myasthenia gravis Nitrazepam is contraindicated in patients with unstable myasthenia gravis.

For each of the following situations, select the single most likely timeline. The number of times a private prescription for an oral contraceptive may be repeated. 0 2 5 6 7 24 30 72

5 If a private prescription does not state the number of times the item maybe repeated, it can be repeated once, except if it is for contraception where it can be repeated five times.

Chloramphenicol is an antibiotic which can be bought over the counter to treat acute bacterial conjunctivitis. What is the maximum duration of treatment which can be given via over the counter chloramphenicol? 2 days 3 days 5 days 7 days 10 days

5 days. Chloramphenicol may be sold OTC for the treatment of bacterial conjunctivitis. The total treatment time for OTC chloramphenicol must not exceed 5 days.

For each of the following situations, select the single most likely timeline. A veterinary prescription for a controlled drug should be retained in the pharmacy for this length of years. 0 2 5 6 7 24 30 72

5 years

Your responsibility as the superintendent pharmacist for a pharmacy is to ensure that all record keeping is appropriate. What is the legal requirement for how long the Responsible Pharmacist record be kept for from the date of the last entry? 1 month 2 months 6 months 1 year 2 years 3 years 5 years 10 years

5 years The Responsible Pharmacist record should be kept for 5 years from the last day to which the record relates.

What is the minimum period of time you should keep the following? Responsible Pharmacist record. No record kept 1 day 1 month 3 month 6 months 1 year 2 years 5 years

5 years The Responsible Pharmacist record should be kept for five years. For electronic records, this is five years from the day the record was created. For written records, this is five years from the last entry in the record.

Ensuring appropriate GP referral will help safeguard patients. Which of the following should result in referral to the patient's GP? A 15-year-old patient who is complaining of constipation. A 46-year-lady complaining of cystitis like symptoms. A patient aged 68 complaining of a migraine like symptoms for the first time. A 7-year-old child who is congested. A 12-year-old boy who has pyrexia.

A patient aged 68 complaining of a migraine like symptoms for the first time

In which one of the following patients should combined oral contraceptives be avoided due to arterial disease risk factors? A patient who smokes 30 cigarettes a day. A patient with a blood pressure of 150/90. A 45-year-old patient. A patient with a BMI of 33kg/m2. A patient with severe migraines with aura.

A patient with severe migraines with aura. Under risk factors for arterial disease - only migraine as described is an avoid criteria. Patients with migraine with aura are at an increased risk of stroke and should not take combined contraceptives because of the increased risk of stroke. The other options are use with caution criteria unless they are combined and a patient has two or more factors.

From the following scenarios, which one of the following patients would have to pay the NHS prescriptions charge for an NHS prescription written in England? A woman who has had a still birth within the last year. A 63-year-old woman on hormone replacement therapy. A patient with type II diabetes mellitus controlled by diet. A patient with epilepsy requiring continuous anticonvulsant therapy. A patient who currently receives income support

A patient with type II diabetes mellitus controlled by diet

You receive a requisition for a controlled drug. Which one of the following supplies is NOT acceptable? - A schedule 3 controlled drug can be given to a doctor in an emergency, if a written requisition will be supplied in 6 hrs. - A supply made against a faxed requisition. - A written requisition is obtained by the supplier before delivery of a schedule 3 controlled drug to a care home. - In an emergency, a dentist can be supplied with methadone on the undertaking the requisition will be supplied within 24 hours. - The stock is collected by a messenger on behalf of a purchaser. The messenger shows the pharmacist a written authorisation that empowers them to receive the medicines.

A supply made against a faxed requisition. Supplies made against a faxed or photocopied requisition are not acceptable.

Which of the following could you sell to the patient without a prescription if you had a Pharmacy (P) pack of the product? Atovaquone 250 mg with proguanil hydrochloride 100 mg tablets Doxycycline 50 mg capsules Mefloquine 250 mg tablets Pyrimethamine 25 mg tablets Quinine 200 mg tablets

Atovaquone 250 mg with proguanil hydrochloride 100 mg tablets Maloff Protect 250 mg/100 mg film-coated tablets (atovaquone with proguanil hydrochloride) was launched as a Pharmacy (P) medicine in June 2017.

For each of the following statements, select the drug that fits most closely. Routine monitoring of white blood cell count must be carried out during treatment with this medication. Allopurinol Azathioprine Azithromycin Cholestyramine Clindamycin Co-beneldopa Colchicine Ethosuximide

Azathioprine Azathioprine may cause blood dyscrasias. As a result, routine WBC monitoring must be carried out during treatment with this medication.

Patients should be warned to immediately report any signs of bone marrow suppression (e.g. flu-like symptoms) for a number of drugs in the BNF. Which of the following drugs is NOT associated with bone marrow suppression? Mycophenolate mofetil Azilsartan Methotrexate Azathioprine Carbimazole

Azilsartan

You are working in Medicine's Information when a newly qualified doctor contacts you to discuss the treatment of chlamydia. Which of the following antibiotic drugs is normally given as a '1 gram STAT' dose in uncomplicated genital chlamydial infections? Azithromycin Amoxicillin Clarithromycin Doxycycline Nitrofurantoin

Azithromycin 1 gram azithromycin STAT is a first-line treatment option for uncomplicated chlamydia.

For each of the following statements, select the drug that fits most closely. This drug is commonly prescribed at a dose of 1 gram STAT for uncomplicated genital chlamydial infections. Allopurinol Azathioprine Azithromycin Cholestyramine Clindamycin Co-beneldopa Colchicine Ethosuximide

Azithromycin Azithromycin is commonly prescribed at a dose of 1 gram STAT for uncomplicated genital chlamydial infections.

For each of the following patients, select the drug which is contraindicated due to drug disease interactions. A patient is admitted to the ward after suffering repetitive hypoglycaemic episodes. Which of the following drug may be masking the signs of hypoglycaemia? Aciclovir Atenolol Atorvastatin Bendroflumethiazide Chloramphenicol Flucloxacillin Levomepromazine Paracetamol

Atenolol Beta blockers may mask the symptoms of hypoglycaemia.

For each of the following patients, select the drug which is most likely to be responsible for the symptoms described. Mrs D, a 58-year-old nurse, is reporting cold hands and feet following the addition of a new drug. Amlodipine Atenolol Bendroflumethiazide Bumetanide Doxazosin Lisinopril Methyldopa Verapamil

Atenolol Beta-blockers such as atenolol cause coldness of the extremities.

For each of the following questions or statements, select the illness which is correct. Which condition would be suitable to treat with topical steroids if required? Atopic eczema Chicken pox (varicella) German measles (rubella) Impetigo Measles (morbilli) Mumps (epidemic parotitis) Slap cheek syndrome (parvovirus) Whooping cough (pertussis)

Atopic eczema Atopic eczema is an inflammatory condition that can be treated with topical steroids. It should also be controlled with emollients.

Mr K is a 23-year-old student. He has itchy patches on the inside of his elbows and he comes into your pharmacy regularly to collect his emollients. He has had this condition most of his life. Which one of the following conditions is Mr K most likely to have? Acne rosacea Atopic eczema Candidiasis Impetigo Psoriasis

Atopic eczema. Atopic eczema is classically itchy. It affects flexural sites such as folds of the elbows and backs of the knees.

For each of the following patients, select the drug which is contraindicated due to drug disease interactions. A patient has been recently diagnosed with rhabdomyolysis. Which medication should be AVOIDED in this patient? Aciclovir Atenolol Atorvastatin Bendroflumethiazide Chloramphenicol Flucloxacillin Levomepromazine Paracetamol

Atorvastatin Statins may cause rhabdomyolosis, thus should be avoided in patients suffering with this condition.

Your locality has decided to switch patients from simvastatin to atorvastatin. A doctor calls you to ask what dose of atorvastatin her patient should be transferred to. The patient was previously on simvastatin 40 mg daily. What is a suitable dose? 10 mg 20 mg 30 mg 40 mg 80 mg

Atorvastatin 10mg The intensity of LDL-cholesterol reduction (%) is the same for 40 mg simvastatin and 10 mg atorvastatin.

High intensity statins produce a reduction in low-density lipoprotein (LDL) cholesterol of more than 40%. Which one of the following doses would be most likely to reduce LDL cholesterol by more than 40%? Atorvastatin 20 mg Fluvastatin 80 mg Pravastatin 40 mg Rosuvatatin 5 mg Simvastatin 40 mg

Atorvastatin 20 mg. Atorvastatin 20 mg is the only high intensity statin listed.

A lady seeks advice regarding an unpleasant condition. The lady complains that her vaginal region is itchy, inflamed with a thick white vaginal discharge. Which of the following would be the most likely causative organism? Candida albicans Escherichia coli Gardnerella vaginalis Pseudomonas aeruginosa Staphylocous aureus

Candida albicans, The patient is complaining of symptoms related to vaginal thrush. The common organism causing this kind of condition is candida albicans.

50 male, prescribed oxybutynin HCL tabs to treat urinary incontinence. How often should the need for continuing therapy be reviewed? A. every 1-2 weeks until symptoms stabilise then annually B. every 4 weeks until symptom stabilise then no need to review further C. every 4-6 weeks until symptoms stabilise then every 6-12 months D. every 6 weeks for the duration of therapy E. every 6 months for duration of treatment

C. every 4-6 weeks until symptoms stabilise then every 6-12 months

Which of the following non-prescription topical preparations should be used once at night? HC45® hydrocortisone 1% cream Clotrimazole 2% cream Daktarin® oral gel Zovirax® 5% cream Canesten® soft gel pessary

Canesten® soft gel pessary. A vaginal pessary is considered a topical preparation exerting its effect on the vaginal mucosa. This is distinct from a systemic agent which would be taken orally.

Which of the drugs below have the following counselling points? Keep taking this medicine until the course is finished, unless you are told to stop. Tacrolimus topical Co-careldopa Ketoprofen Cefalexin Detrusitol® Sandocal® Amitriptyline Triamterene

Cefalexin

You differentially diagnose thrush in a 32-year-old female. Which symptom is LEAST likely experienced with thrush? A thin, white discharge. Discharge associated with a strong odour. Dysuria. Superficial dyspareunia. Vaginal itching and soreness.

Discharge associated with a strong odour. In thrush, itching is the hallmark symptom; a white discharge does occur in a proportion of patients; dyspareunia and dysuria are possible but uncommon. However, discharge with odour is very unlikely.

An adult presents to the pharmacy with a rash on their lower left leg. Which condition is classically associated with the lower extremities? Discoid eczema Ringworm Seborrhoeic dermatitis Shingles Urticaria

Discoid eczema. Shingles is usually back and face; ringworm is usually body; seborrhoeic dermatitis is face and chest. This leaves urticarial and discoid eczema as the two likely right answers. Urticaria can affect any body part including the legs, but discoid eczema is classically associated with lower extremities.

Mrs WB is a 77-year-old woman, who is admitted to hospital for the treatment of hallucinations. Her past medical history is multiple sclerosis. Her doctor decides to prescribe haloperidol. During treatment Mrs WB complains of feeling unwell and you are asked if this could be due to the haloperidol. Which ONE of the following symptoms suffered by Mrs WB is most likely to be caused by haloperidol? Sweating Dizziness Diarrhoea Dyspnoea Fatigue

Dizziness

A 13-month-old child has bilateral, red, watery eyes with some pus noticeable at the corners of both eyes. Her mother takes her to the GP who diagnoses simple conjunctivitis. Which antibiotic is the most appropriate to treat this patient? Amoxicillin Chloramphenicol Flucloxacillin Linezolid Metronidazole

Chloramphenicol. Chloramphenicol eye preparations are not licensed for use under the age of two when sold over the counter. Prescribers can prescribe medicines off-licence.

For each of the following statements, select the drug that fits most closely. Medicine-induced colitis is a risk factor with use of this medication. Allopurinol Azathioprine Azithromycin Cholestyramine Clindamycin Co-beneldopa Colchicine Ethosuximide

Clindamycin Antibiotic induced colitis is a risk for patients when they are prescribed courses of clindamycin. They must be counselled on this adequately prior to commencing treatment.

For each of the following questions, select the drug which is correct in relation to the scenario. This medication is indicated for bacterial vaginosis and is usually given at a dose of one applicatorful daily for 3-7 nights. Sildenafil Dapoxetine Oxytocin Carbetocin Misoprostol Clindamycin Clotrimazole Estriol

Clindamycin Clindamycin is indicated for the treatment of bacterial vaginosis.

For each of the following statements, select the drug that fits most closely. This drug may be used in the treatment of cellulitis. It can be administered orally or intravenously for this condition. Allopurinol Azathioprine Azithromycin Cholestyramine Clindamycin Co-beneldopa Colchicine Ethosuximide

Clindamycin Clindamycin is indicated in soft tissue infections such as cellulitis. It may be administered orally or via the IM or IV routes.

Which one of the following medicines does NOT affect the monoamine oxidase enzyme family? Selegiline Linezolid Moclobemide Phenelzine Clonidine

Clonidine. Clonidine is the only one that does NOT affect the monoamine oxidase enzyme family. Linezolid is a reversible, non-selective inhibitor of monoamine oxidase; however, at the doses used for antibacterial therapy, it does not exert an anti-depressive effect.

Which one of the following sources of information is NOT confidential? Electronic data concerning patients Hard copy data with patient identifiers on it Coded audit material concerning individual patients and side effects to drugs The fact a patient's husband has died, something they have told you in the pharmacy Patient's address information

Coded audit material concerning individual patients and side effects to drugs

A 25-year-old woman is breastfeeding her 5-month-old child and requires pain relief. Which drug has a risk of overdosing an infant if used in breastfeeding women due to a maternal variation in capacity to metabolise? Codeine Ibuprofen Morphine Paracetamol Tramadol

Codeine. Genetic polymorphism of enzymes involved in the metabolism of codeine to morphine can lead to increased levels of morphine in the mother.

Mr W comes into your pharmacy. He tells you that he suffers from severe pain and is currently taking paracetamol. Which of the following painkillers can Mr W take in addition to his paracetamol? Co-codamol Codis® Co-dydramol Panadol Actifast® Tramacet®

Codis® Codis is made up of aspirin and codeine. All of the other preparations contain paracetamol.

You attend a CPD seminar on side-effects of mineralocorticoid and glucocorticoids. Which one of the following effects is associated more with glucocorticoid use? Calcium loss Diabetes Hypertension Potassium loss Water retention

Diabetes Glucocorticoid side-effects include diabetes, osteoporosis and muscle wasting. The other listed side-effects are associated with mineralocorticoids.

Which of the listed conditions are cautioned with the following products? Pseudoephedrine tablets. Diabetes and hypertension Hay fever Asthma Hypertension and asthma Epilepsy Migraine Ulcer Ulcer and asthma

Diabetes and hypertension Pseudoephedrine is cautioned in hypertension and diabetes.

Which of the listed conditions are cautioned with the following products? Paracetamol 500 mg and pseudoephedrine 30 mg. Diabetes and hypertension Hay fever Asthma Hypertension and asthma Epilepsy Migraine Ulcer Ulcer and asthma

Diabetes and hypertension The latter, i.e. pseudoephedrine, is cautioned in hypertension and diabetes.

For each of the following patients, select the drug which is contraindicated due to drug disease interactions. A patient is admitted to the ward suffering with cholestatic jaundice. Which of the following drugs is most likely to be a causative factor in this admission? Aciclovir Atenolol Atorvastatin Bendroflumethiazide Chloramphenicol Flucloxacillin Levomepromazine Paracetamol

Flucloxacillin Flucloxacillin can cause cholestatic jaundice - this may occur up to 8 weeks post treatment.

For each of the following statements, select the drug that fits most closely. This drug can cause cholestatic jaundice up to 8 weeks after treatment has ceased. Desmopressin Diazepam Flucloxacillin Lamotrogine Misoprostol Paroxetine Salbutamol Vigabatrin

Flucloxacillin Flucloxacillin can cause cholestatic jaundice. This may occur during treatment or up to 8 weeks post treatment.

Mrs R is a 41-year-old sales assistant. She has been prescribed fludrocortisone for adrenal insufficiency and is unclear about its possible side-effects. She lists different conditions which she may develop as a result and asks you if they are correct. Which one of the following is most likely to develop with fludrocortisone use? Avascular necrosis Diabetes Hypertension Muscle wasting Osteoporosis

Hypertension. Mineralocorticoid side effects, listed in the BNF, are most marked with fludrocortisone. The other side effects listed are glucocorticoid steroid effects.

Which of these medical terms best describes the medication side effects discussed in the scenarios below? Mr L experienced an increase in hair growth and a "ringing in the ears" secondary to the use of diazoxide for the treatment of chronic intractable hypoglycaemia. Hypertrichosis and tenesmus Hypertrichosis and tinnitus Hypertriglyceridemia and tympanosclerosis Akathisia and tardive dyskinesia Hyperhydrosis and tinnitus Dystonia and dyskinesia Akathisia and dystonia Akathisia and myoclonic jerks

Hypertrichosis and tinnitus

Ms G is a 78-year-old female who is a regular customer at your pharmacy. She has recently been initiated on Flupentixol. Which of the following risks are NOT associated with the use of anti-psychotics in elderly patients? Hypothermia Hyperthermia Hypoglycaemia Postural hypotension Stroke

Hypoglycaemia Hyperglycaemia (NOT hypoglycaemia) and sometimes diabetes is a risk with all antipsychotics.

Which of the following can be a side-effect of fludrocortisone? Hypokalaemia Hyponatraemia Hypocalcaemia Hypomagnesaemia Hypophosphataemia

Hypokalaemia

Which of the following can be a side-effect of ciclosporin? Hypophosphataemia Hypomagnesaemia Hypocalcaemia Hyponatraemia Hypokalaemia

Hypomagnesaemia

Which of the following can be a side-effect of mirtazapine? Hypokalaemia Hyponatraemia Hypocalcaemia Hypomagnesaemia Hypophosphataemia

Hyponatraemia

Which ONE of the following is NOT a reason to STOP the combined oral contraceptive pill? Sudden breathlessness Severe stomach pains Hypertension - 150/90 mmHg Jaundice Unexplained swelling of the calf of one leg

Hypertension - 150/90 mmHg. Only blood pressure above systolic 160 mmHg or diastolic 95 mmHg is a reason to stop the combined oral contraceptive pill.

Select the best answer from the following question... Patients using this insulin must be counselled that they are to inject their insulin 30 minutes before the start of their meal. Hypurin porcine neutral Insulin aspart Insulin glargine Insulin degludec Biphasic insulin aspart

Hypurin porcine neutral SPC for hypurin porcine neutral states: "After subcutaneous injection onset of action occurs within 30-60 minutes". Therefore it should be injected 30 minutes before meal.

In each of the following scenarios select what would be the most appropriate treatment option for the patient if you were to add to the patient's existing treatment. A 50-year-old Caucasian lady who has been taking ramipril and amlodipine for the past 4 weeks at the maximum licensed doses for hypertension. The patient has a blood pressure reading of 170/105 mmHg that was taken in the surgery. Atenolol Aspirin Candesartan Indapamide Lacidipine No drug treatment Perindopril Spironolactone

Indapamide

Warfarin is a commonly used drug that is high risk and requires detailed patient management. A patient requires regular blood tests which monitor the INR. Which of the following conditions does NOT have a target INR of 2.5? Deep vein thrombosis. Dilated cardiomyopathy. Mechanical prosthetic heart valves. Myocardial infarction. Pulmonary embolism

Mechanical prosthetic heart valves The target INR for mechanical prosthetic heart valves 3.5.

You are training to become an independent prescriber with the aim of specialising in diabetes. Choose the most appropriate next treatment step for each of the patients described. A patient newly diagnosed with type 2 diabetes who has a raised HbA1c despite lifestyle interventions. Address adherence Basal insulin DDP-4 inhibitor GLP-1 agonist Metformin SGLT2 inhibitor Soluble insulin Sulphonylurea

Metformin. Metformin is a first-line choice in the treatment of type 2 diabetes mellitus.

Excessive consumption of which of these is most likely to have caused the signs and symptoms described below. 17-year-old female who cannot sleep, is hyperactive and hallucinating. Methamfetamine Alcohol Sodium oxybate Fluoxetine Atenolol Amitriptyline Paracetamol Chloroquine

Methamfetamine

Which of the following symptoms is most likely to be attributed to rosuvastatin? Myositis Anxiety Weight gain Bruising that may be heavy Black stools

Myositis

You are asked to give a presentation to the physiotherapists on medicines that contribute to falls in the elderly. Which one of the following groups would you NOT include because it is least likely to cause falls? Benzodiazepines Diuretics NSAIDs Opioids Phenothiazines

NSAIDs

Rheumatoid arthritis presents with a myriad of symptoms, of which a very common presentation is pain and inflammation. Which of the following is licensed for pain and inflammation in rheumatoid arthritis? Domperidone Esketamine Febuxostat Nabumetone Naloxone

Nabumetone The only medication on this list licensed for pain and inflammation in rheumatoid arthritis is nabumetone.

Miss D is a 37-year-old artist. She started a new medication which has been causing her nightmares and is disturbing her sleep. Her GP contacts you to discuss an alternative beta-blocker. Which of the following beta-blockers is LESS likely to cause nightmares? Labetolol Nadolol Oxprenolol Propranolol Timolol

Nadolol. Miss D is less likely to experience nightmares with water soluble beta blockers as they are less likely to enter the brain, and may therefore cause less sleep disturbance and nightmares.

Miss S is an 18-year-old student who has recently been diagnosed with acute porphyria. She comes into your pharmacy to collect her medication. Which single drug is most likely to induce an acute attack in Miss S? Aspirin Codeine Nalidixic acid Naproxen Paracetamol

Nalidixic acid. Nalidixic acid is listed as an unsafe drug for use in patients with acute porphyria.

For each of the following patients, select the most appropriate antidote. A 37-year-old man has taken an overdose of his morphine tablets. Acetylcysteine Aspirin Digoxin-specific antibody European viper snake venom antiserum Flumazenil Methaemoglobinaemia Naloxone hydrochloride Protamine sulfate

Naloxone hydrochloride The antidote for opioid toxicity is naloxone hydrochloride.

You are writing a standard operating procedure for making CD records in the pharmacy. Which of the following is legally required to be recorded on receipt of a Schedule 2 CD? Date order was placed. Name of patient it will be dispensed to. Name of pharmacy staff member accepting the order. Name of pharmacy staff member that ordered it. Name of wholesaler.

Name of wholesaler. The name and address of whom the CD is received from is legally required to be kept in the CD register.

You are working in a community pharmacy. A patient presents to you with indigestion. You are aware that a few weeks ago you dispensed some new medicines for the patient. Which of the following medicines would be the most likely cause of the patient's symptoms? Atenolol Bendroflumethiazide Lisinopril Nifedipine Valsartan

Nifedipine. Calcium antagonists, nitrates, theophyllines, bisphosphonates, corticosteroids and nonsteroidal anti-inflammatory drugs can cause dyspepsia.

A concerned patient, Mrs TV has just started taking tibolone tablets. She asks you for advice regarding side effects. Taste disturbance Seizures Nipple pain Lethargy Alopecia

Nipple pain

Miss M is a 29-year-old pregnant women with suprapubic pain over the past 4 days and a "burning" sensation on urination. Given that Miss M is in her first trimester of pregnancy and is not on any other medication, which one of the following antibiotics is the most appropriate for her condition? Ciprofloxacin 250 mg twice daily Doxycycline 200 mg once daily Nitrofurantoin 50 mg four times a day Ofloxacin 200 mg daily Trimethoprim 200 mg twice daily

Nitrofurantoin 50 mg four times a day. Miss M has a urinary tract infection (UTI). Trimethoprim should not be given to Miss M (folate antagonist and teratogenic in the first trimester). Ciprofloxacin and ofloxacin are quinolone antibiotics and should thus be avoided in pregnancy (shown to cause arthropathy in animal studies and there are safer alternatives available. Doxycycline is a tetracycline that should not be given to pregnant women (effects on skeletal development being documented in the first trimester in animal studies). Note that nitrofurantoin should be avoided in pregnant mothers at term and in those who are breastfeeding. Therefore, it is the most appropriate for Miss M.

Mrs J, a 41 year old, has a non-productive cough. You believe it to be a viral infection. She has hypertension and suffers from type 1 diabetes. What would be the most appropriate treatment/course of action? A cough suppressant A demulcent An antihistamine An expectorant No treatment

No treatment. Viral infections are self-limiting and no treatment is necessary, therefore it would be a suitable 'recommendation'. A demulcent could be tried. As they are diabetic a sugar-free alternative would be useful but it is unlikely that a few days treatment with a demulcent containing sugar will affect their diabetic control. If a demulcent containing sugar is recommended then the person could be told to monitor their blood sugar more regularly. Cough suppressants and antihistamines have no evidence of efficacy and should not be recommended. An expectorant is only suitable for productive coughs.

Opiates are known drugs of abuse. You are running a training work shop to educate community pharmacists on the abuse potential of these drugs. Which of the following is NOT a symptom of opioid toxicity? Coma Pinpoint pupils Pupil dilation Respiratory depression Unconsciousness

Pupil dilation. Pupil narrowing or 'miosis' is an effect of opiate toxicity; pupil dilation is not.

You are the Responsible Pharmacist in a community pharmacy and you receive a written requisition for morphine tablets. Which of the following is a legal requirement for this requisition? Date of birth of recipient Purpose of the requisition Quantity of the drug in words and figures Strength of the drug in words and figures Your signature

Purpose of the requisition The six legal requirements for a CD requisition include: Recipients signature Name of recipient Address of recipient Profession/occupation Total quantity of drug Purpose of requisition

Mrs A presents to the pharmacy complaining of dysuria, fever and flank pain. Which condition is this symptom cluster most indicative of? Cystitis Pyelonephritis Salpingitis Trichomoniasis Vaginitis

Pyelonephritis. Fever suggests infection, and from this list cystitis, vaginitis and salpingitis are not infections and not associated with fever as a symptom. This leaves trichomoniasis and pyelonephritis. Pyelonephritis is associated with fever and flank pain, compared to trichomoniasis whose predominant feature is discharge.

For each of the following patients described, select the most likely possible consequence of the drug interaction from the list above. A 72-year-old man has been on citalopram 40 mg daily for 6 years. He has been newly prescribed sildenafil 25 mg daily for erectile dysfunction. Bleeding risk increased Bradycardia Constipation First dose hypotension Hypertensive crisis Reduced eGFR Serotonin syndrome QT interval prolongation

QT interval prolongation QT interval prolongation potentiated by SSRIs, such as citalopram, and sildenafil.

In each of the following scenarios select what would be the most appropriate treatment option for the patient if you were to add to the patient's existing treatment. A 60-year-old Caucasian gentlemen with a recent blood pressure reading of 160/100 mmHg that was taken by the nurse in the surgery and a home blood pressure reading of 150/95 mmHg. He is not on any prescribed medication. Atenolol Aspirin Candesartan Indapamide Lacidipine No drug treatment Perindopril Spironolactone

Lacidipine

In each of the following scenarios select what would be the most appropriate treatment option for the patient if you were to add to the patient's existing treatment. A 65-year-old black gentlemen with type 2 diabetes and with a recent blood pressure reading of 150/100 mmHg taken in the surgery by the nurse. The patient has been taking losartan and verapamil for the past 4 weeks at the maximum doses. Atenolol Aspirin Candesartan Indapamide Lacidipine No drug treatment Perindopril Spironolactone

Indapamide

Mr Q, a 21 year old who weighs 55 kg, is admitted to hospital seriously ill with diabetic ketoacidosis. The doctor prescribes soluble insulin and intravenous fluid therapy with electrolyte replacement. By which route will the soluble insulin be administered? Subcutaneous injection Intradermal injection Intramuscular injection Intravenous injection Intravenous infusion

Intravenous infusion

Which ONE of the following is most likely to cause ketoacidosis? Intravenous salbutamol Salmeterol Ipratropium Tiotropium Inhaled salbutamol

Intravenous salbutamol. Beta 2 agonists must be used with caution in patients with diabetes. Intravenous salbutamol can cause ketoacidosis.

Regarding exenatide, which one of the following statements is NOT true? It is a GLP-1 receptor antagonist. It suppresses glucagon secretion. It is administered by the subcutaneous route. It may be combined with metformin and pioglitazone. It may cause weight loss.

It is a GLP-1 receptor antagonist. Exanitide is a GLP-1 agonist belonging to the class of incretrin mimetics. It is NOT therefore a GLP-1 receptor antagonist.

Oral contraception is a very commonly used reversible method of contraception, Many women rely on the pill to help prevent unwanted pregnancy. One of the commonly prescribed pills in the UK is Yasmin. Which of the following is UNTRUE regarding Yasmin? It can cause vaginal discharge. It contains drospirenone 3 mg. It contains ethinylestradiol 30 mcg. It is a monophasic 28-day preparation. It may cause sexual dysfunction.

It is a monophasic 28-day preparation Yasmin is a monophasic 21-day preparation.

Peptic ulcer disease affects all age groups and can be quite a debilitating condition. Which of the following is NOT a cause of peptic ulcer disease? Helicobacter pylori NSAIDs Smoking Lack of exercise Excessive alcohol consumption

Lack of exercise. Peptic ulcers can be caused byH. Pyloriinfection, prolonged or excessive use of NSAIDs and excessive alcohol consumption. Smoking can also increase the risk of developing an ulcer.

You have a new member of staff starting on you pharmacy counter team. You are providing them with training on the sale of over-the-counter (OTC) medicines. Which of the following OTC medicines would you NOT require him to exercise additional cautions to sell due to the risk of misuse? Codeine Ephedrine Kaolin Loperamide Pseudoephedrine

Kaolin and morphine would need to have additional caution, but kaolin itself is not subject to abuse.

The Misuse of Drugs Regulations 2001 (as amended) classify controlled drugs into 5 schedules according to the different levels of control attributed to each. Which schedule includes ketamine? Schedule 1 Schedule 2 Schedule 3 Schedule 4 Schedule 5

Ketamine is a Schedule 2 CD.

For each of the following statements, select the drug that fits most closely. This drug is available as shampoo for the treatment of fungal infections affecting the scalp. Alendronate Disulfiram Isoniazid Isotretinoin Ketoconazole Lithium Rifampicin Sulfasalazine

Ketoconazole Ketoconazole is available as 2% shampoo for the use of scalp conditions.

For each of the following patients, select the most appropriate treatment option for the scenario described. Mr A is a 42-year-old man who comes into your pharmacy. He shows you hyperpigmented patches on his trunk and back. They are not red or itchy. Betamethasone Clobetasone butyrate Griseofulvin Hydrocortisone Ketoconazole shampoo Miconazole Rest and ibuprofen Tetracycline

Ketoconazole shampoo. Pityriasis versicolor, sometimes called tinea versicolor, is a common condition that causes small patches of skin to become scaly and discoloured (hypo/hyperpigmented). They usually occur on the trunk and back and upper arms. Pityriasis versicolor is caused by a type of yeast called Malassezia and is treated by ketoconazole shampoo.

For each of the following scenarios, choose the appropriate treatment for the patient. A non-smoking patient who has spirometically confirmed COPD and asthmatic features, but remains breathless despite optimised SABA treatment. ICS LABA LABA + ICS LABA + LAMA LAMA LAMA + LABA + ICS SABA SAMA

LABA + ICS Consider LABA + ICS for people who: have spirometrically confirmed COPD and have asthmatic features/features suggesting steroid responsiveness and remain breathless or have exacerbations despite: having used or been offered treatment for tobacco dependence if they smoke and, optimised non-pharmacological management and relevant vaccinations and, using a short-acting bronchodilator

For each of the following scenarios, choose the appropriate treatment for the patient. A non-smoking patient with spirometrically confirmed COPD who does not have asthmatic features but remains breathless despite use of a SABA. ICS LABA LABA + ICS LABA + LAMA LAMA LAMA + LABA + ICS SABA SAMA

LABA + LAMA Offer LAMA + LABA to people who: have spirometrically confirmed COPD and do not have asthmatic features/features suggesting steroid responsiveness and remain breathless or have exacerbations despite: having used or been offered treatment for tobacco dependence if they smoke and, optimised non-pharmacological management and relevant vaccinations and, using a short-acting bronchodilator.

For each of the following scenarios, choose the appropriate treatment for the patient. A patient with spirometrically confirmed COPD who has features suggesting steroid responsiveness and remains breathless despite taking LABA + ICS. ICS LABA LABA + ICS LABA + LAMA LAMA LAMA + LABA + ICS SABA SAMA

LAMA + LABA + ICS For people with COPD who are taking LABA + ICS, offer LAMA + LABA + ICS if: - their day-to-day symptoms continue to adversely impact their quality of life or - they have a severe exacerbation (requiring hospitalisation) or - they have 2 moderate exacerbations within a year.

In each of the following scenarios select what would be the most appropriate treatment option for the patient if you were to add to the patient's existing treatment. A 52-year-old Caucasian lady with a recent blood pressure reading of 160/100 mmHg that was taken in the surgery, who has been taking ramipril at the maximum dose for the past 6 weeks. Atenolol Aspirin Candesartan Indapamide Lacidipine No drug treatment Perindopril Spironolactone

Lacidipine

In each of the following scenarios select what would be the most appropriate treatment option for the patient if you were to add to the patient's existing treatment. A 54-year-old black gentlemen with a recent blood pressure reading of 160/100 mmHg taken by the nurse in the surgery and a home blood pressure reading of 150/95 mmHg. He is not taking any prescribed medication. Atenolol Aspirin Candesartan Indapamide Lacidipine No drug treatment Perindopril Spironolactone

Lacidipine

For each of the following questions, select the drug which is correct in relation to the scenario. Whilst delivering a revision session to a set of pharmacy students, you are asked which of the listed laxatives is an osmotic laxative? Isphaghula husk Methylcellulose Bisacodyl Co-danthramer Docusate sodium Senna Lactulose Glycerol

Lactulose Lactulose is an osmotic laxative which exerts its action by increasing the amount of water in the large bowel.

For each of the following questions, select the drug which is correct in relation to the scenario. A doctor in A&E is seeing a patient with a history of alcohol misuse and is querying which laxative can be used to treat hepatic encephalopathy. Isphaghula husk Methylcellulose Bisacodyl Co-danthramer Docusate sodium Senna Lactulose Glycerol

Lactulose Lactulose is indicated for the treatment of hepatic encephalopathy.

Different schedules of controlled drugs have varying prescription requirements and legalities. Of the following controlled drugs, which is allowed to be issued via a repeat prescription? For example the prescriber may write on the prescription 'Repeat x 3'? Lorazepam Pethidine Methadone Morphine Temazepam

Lorazepam, a schedule 4 CD, is allowed on repeats.

A parent is seeking advice for their 6-year-old child. The child is complaining of a blocked nasal passage and appears congested. Which of the following is the most appropriate treatment for the child? Ephedrine tablets Paracetamol suspension Pseudoephedrine tablets Saline nasal spray Xylometazoline 0.1%

Saline nasal spray. Saline nasal spray is effective in reducing congestion and is safe to use in children of 6 years of age.

Which of the drugs below have the following counselling points? Dissolve or mix this medicine with water before taking. Tacrolimus topical Co-careldopa Ketoprofen Cefalexin Detrusitol® Sandocal® Amitriptyline Triamterene

Sandocal

You are explaining the mechanism of action of different anti-diabetes medicines to pharmacy students. Link the medicine to the mechanism of action. Which medicine inhibits dipeptidylpeptidase-4 to increase insulin secretion and lower glucagon secretion? Acarbose Empagliflozin Exenatide Gliclazide Metformin Repaglinide Saxagliptin Short-acting insulin

Saxagliptin Saxagliptin inhibits dipeptidylpeptidase-4 to increase insulin secretion and lower glucagon secretion.

For each of the following scenarios, select the most likely condition. All of the persons living with the individual who presents with this should be treated. Allergic reaction Chickenpox Eczema Impetigo Psoriasis Scabies Shingles Urticaria

Scabies When one individual has scabies everyone in the household needs to be treated at the same time, even if they don't have symptoms.

The Misuse of Drugs Regulations 2001 (as amended) classify controlled drugs into 5 schedules according to the different levels of control attributed to each. Which schedule is cannabis in when it is not being used as a medicinal product? Schedule 1 Schedule 2 Schedule 3 Schedule 4 Schedule 5

Schedule 1. Cannabis is a Schedule 1 CD when not being used medically.

The Misuse of Drugs Regulations 2001 (as amended) classify controlled drugs into 5 schedules according to the different levels of control attributed to each. Amphetamines belong to which schedule of controlled drugs? Schedule 1 Schedule 2 Schedule 3 Schedule 4 Schedule 5

Schedule 2

The Misuse of Drugs Regulations 2001 (as amended) classify controlled drugs into 5 schedules according to the different levels of control attributed to each. Which schedule includes clenbuterol? Schedule 1 Schedule 2 Schedule 3 Schedule 4 Part 1 Schedule 4 Part 2

Schedule 4 Part 2 Clenbuterol is a Schedule 4 (part 2) CD.

The Misuse of Drugs Regulations 2001 (as amended) classify controlled drugs into 5 schedules according to the different levels of control attributed to each. Which of the following classifications require no licence to import or export the controlled drug? Schedule 2 Schedule 3 Schedule 4 Part 1 Schedule 4 Part 2 Schedule 5

Schedule 5

The Misuse of Drugs Regulations 2001 (as amended) classify controlled drugs into 5 schedules according to the different levels of control attributed to each. Which of the following classifications allow for a prescription to be dispensed up to 6 months after the appropriate date? Schedule 2 Schedule 3 Schedule 4 Part 1 Schedule 4 Part 2 Schedule 5

Schedule 5. Schedule 5 Controlled Drugs prescriptions are valid for six months from the appropriate date on the prescription.

For each of the following descriptions, select the condition that is most accurate. Is often associated with radiating pain. Bursitis Chondromalacia Lateral epicondylitis Lower back pain Medial epicondylitis Rotator cuff syndrome Sciatica

Sciatica A number of listed conditions can exhibit radiating pain, e.g. epicondylitis, but classically in sciatica radiation of pain is very common.

Mrs G is a 27-year-old student who presents with red scaly patches on her cheeks, eyebrows and hairline. She has had these patches for 2 years now. Which one of the following conditions is Mrs G most ikely to have? Atopic eczema Candidiasis Psoriasis Seborrhoeic dermatitis Tinea faciei

Seborrhoeic dermatitis Seborrhoeic dermatitis presents with red rash typically affecting the central regions of the face, eyebrows and scalp.

Mr King, a 44 year old, presents with symptoms of nasal congestion, slight sore throat, headache and general loss of smell. The pharmacist decides to make a referral to the doctor because the differential diagnosis suggests? Glandular fever. Influenza. Post-nasal drip. Perrenial rhinitis. Sinusitis

Sinusitis. Nasal congestion and sore throat are hallmark symptoms of the common cold. However, loss of smell in association with these symptoms can suggest sinusitis.

For each of the following questions, select the drug which is correct in relation to the scenario. This drug is used at a dose of 20 mg 3 times a day for pulmonary arterial hypertension. Sildenafil Dapoxetine Oxytocin Carbetocin Misoprostol Clindamycin Clotrimazole Estriol

Sildenafil Sildenafil is licensed for pulmonary arterial hypertension in adults. The dose is 20 mg three times a day.

You are giving a seminar to accredited checking technicians working in your pharmacy on serious drug interactions. Select the most likely drug combination from the list which results in the serious effects described below. This combination may cause a marked hypotensive effect which may be fatal. Digoxin and quinine Digoxin and cefalexin Ketoprofen and paracetamol Sildenafil and isosorbide mononitrate Atenolol and ibuprofen Clonidine and propranolol Pimozide and ketoconazole Metronidazole and cimetidine

Sildenafil and isosorbide mononitrate. Sildenafil may markedly increase the hypotensive effect of isosorbide mononitrate.

You are going through and working out general drug interactions for your own knowledge. Which of the following drug combinations is known to introduce a risk of rhabdomyolysis? Aspirin with paracetamol. Beclomethasone with salbutamol. Digoxin with frusemide. Phenytoin with sodium valproate. Simvastatin with bezafibrate

Simvastatin with bezafibrate. With all statins there is increased risk of rhabdomyolysis. The BNF states 'increased risk of myopathy when simvastatin given with bezafibrate'.

Select from the list below the most likely method of removing the poisons described below. A 56-year-old male with standard release calcium channel blocker overdose presenting within one hour of overdose. Induction of emesis Desferrioxamine administration Repeat administration of activated charcoal Single administration of activated charcoal Alkalinisation of the urine Whole bowel irrigation Haemodialysis Administration of pH neutraliser

Single administration of activated charcoal

For each of the following patients, select the most likely option for their presentations. A 45-year-old male comes into the pharmacy. You note that he has halitosis. He describes a dull, throbbing headache which is made worse when he bends down. Cluster headache Epilepsy Meningitis Migraine Rebound headache Sinus headache Tension headache Vertigo

Sinus headache His signs are typical of a sinus headache: a dull, throbbing headache in the upper half of the face which is made worse on bending down. It can also be associated with halitosis and pain, swelling and tenderness around the eyes, cheeks and forehead.

For each of the following descriptions, select the condition that is most accurate. Is secondary to infection. Cluster headache Medication-overuse headache Meningitis Migraine Sinusitis Sub-arachnoid haemorrhage Temporal arteritis Tension-type headache

Sinusitis Secondary infective causes on the list only relate to sinusitis. Sinusitis occurs as a result of a previous infection, whereas the headache associated with meningitis is one of the symptoms of an infection itself.

You have just finished using a stock bottle of methadone in the pharmacy. You have dealt with any residue remaining in the bottle. How do you dispose of the bottle? Arrange for a specialist disposal company to dispose of the bottle and contents. Place in pharmaceutical waste. Place in recycling or general waste. Remove labels and identifiers and place it pharmaceutical waste. Remove labels and identifiers and place in recycling or general waste.

Remove labels and identifiers and place in recycling or general waste. Once the contents have been dealt with appropriately the bottle can be place in recycling or general waste after removing identifiers.

Which of the following increases the risk of developing gout? Being female Renal impairment A diet high in green leafy vegetables Being underweight Low triglycerides

Renal impairment

Select from the list below the most likely method of removing the poisons described below. A 36-year-old male with theophylline overdose. Induction of emesis Desferrioxamine administration Repeat administration of activated charcoal Single administration of activated charcoal Alkalinisation of the urine Whole bowel irrigation Haemodialysis Administration of pH neutraliser

Repeat administration of activated charcoal

What is the last date you could dispense the following prescriptions? The first repeat on a private prescription for vardenafil signed on 30/06/21 and first dispensed on 01/01/22. 30/06/21 28/07/21 30/09/21 30/03/22 30/06/22 30/06/26 Repeats are not allowed There is no legally defined final dispensing date, it is based on clinical judgement

Repeats are not allowed The first dispensing should be within 6 months of the appropriate date. The first dispensing was incorrectly carried out a day after this time and the mistake must not be carried on.

You are the responsible pharmacist. During his check, your accuracy checking technician notices that you have dispensed atorvastatin instead of atenolol. What is the most appropriate course of action? Ask the accuracy checking technician not to tell anyone Notify the patient Notify the patient's GP Re-dispense and ignore the error Report as a near miss error after re-dispensing

Report as a near miss error after re-dispensing. There should be a culture of open reporting in the profession. All near misses should be recorded so they can be learnt from in the future.

For the patients below, select the most suitable counselling advice you will give. A 23-year-old female who is taking carbimazole for hyperthyroidism. Drowsiness may affect your driving. This effect is enhanced with alcohol consumption. Take this medication at night. Oral solution should be swallowed as a single 100 mL dose with plenty of water while sitting or standing upright. Take this medication on an empty stomach. Avoid factors which cause overheating or dehydration like exercise. Report symptoms including sore throat, mouth ulcers, bruising, fever, tiredness or non-specific illnesses to your doctor immediately. Report immediately if a skin rash develops or if any signs or symptoms of hypersensitivity develops. Take this medication after food

Report symptoms including sore throat, mouth ulcers, bruising, fever, tiredness or non-specific illnesses to your doctor immediately

For each of the following patients, select the single most likely cause behind their complaint. Miss G is a 26-year-old student who is quiet, has few friends and avoids social gatherings especially dining out as she feels that people will laugh at the way she eats. Acute stress disorder Agoraphobia Depression Epilepsy Generalised anxiety disorder Panic disorder Post-traumatic stress disorder Social phobia

Social phobia Social phobia is where a person experiences a marked, persistent and unreasonable fear of being observed, embarrassed or humiliated in a social or performance situation (e.g. public speaking or eating in front of others).

Mrs M is usually prescribed theophylline 175 mg once every 12 hours for her asthma. Lately, this has been insufficient to control her symptoms. Which one of the following products that Mrs M is buying could be causing a reduction in theophylline plasma concentration in Mrs M? St John's Wort Paracetamol Pepto-bismol Naproxen Diclofenac gel

St John's Wort. St John's wort is an enzyme inducer which increases theophylline metabolism and subsequently reduces plasma concentration of theophylline.

A patient presents with a presents with a prescription that states 'Flucloxacillin 500 mg q.d.s. for 7/7'. Upon questioning the patient informs you the antibiotic is for impetigo. What is the most likely causative organism for this infection? Canidida albicans Escherichia coli Gardnerella vaginalis Sacromyces cerivusea Staphylococcus aureus

Staphylococcus aureus. Impetigo is a common contagious pyogenic infection of the superficial layers of the skin, usually caused byStaphylococcus aureus.

Select from the following reference sources which ONE is most likely to contain: Detailed analysis of the outcomes of giving medicines together British National Formulary Martindale Stockley's Drug Interactions Drug Tariff British Pharmacopoeia

Stockley's Drug Interactions The BNF includes information on drug interactions but does not include a detailed analysis of the possible outcomes of an interaction, with reference to clinical trial data or case reports.

Mr FT has come into your pharmacy with previously diagnosed haemorrhoids but seeks further help from you for a better cream to use. Which one of the following symptoms would indicate referral to the GP? Bright red blood on the surface of the stool. Dull ache on defecation. Mucous discharge. Perianal itching. Stools that are tarry.

Stools that are tarry. Blood mixed in stool should be referred to GP to eliminate GI bleed. Slight fresh rectal bleeding is often associated with haemorrhoids and can be visible on the toilet bowl or on the surface of the stool. The other symptoms are also typical of haemorrhoids.

A man presents to your pharmacy complaining of a rash all over his body. He tells you that he started taking phenytoin 9 days ago. What recommendation(s) would you give to this patient? Stop drug, urgent referral for medical attention Refer to GP as dose of drug may need to be reduced Refer to GP as dose of drug may need to be increased Stop drug, refer to GP as the patient is experiencing an allergic reaction Continue taking drug as this is a harmless adverse effect of the drug

Stop drug, urgent referral for medical attention Patients or their carers should be told how to recognise signs of blood or skin disorders, and advised to seek immediate medical attention if symptoms such as fever, rash, mouth ulcers, bruising, or bleeding develop.

A 46-year-old woman was commenced on lithium 400 mg once daily by her GP three months ago for treatment of bipolar disorder. Today she has come in to get a prescription for amoxicillin 500 mg three times a day to treat a dental abscess. She complains of feeling nauseous and has had severe diarrhoea over the last few hours. Upon further questioning you find that she may have accidentally taken a double dose of her lithium tablets yesterday. What is the most appropriate advice to give to this patient? Sell her loperamide capsules and advise her that she should start feeling better within 24 hours. She is experiencing a side-effect of lithium, and whilst safe to continue taking it, she may wish to see her GP for an alternative. She should see her GP as the dose of lithium may need to be reduced. Stop taking lithium straight away and go to her nearest accident and emergency department as soon as possible. The symptoms described are not known to be caused by lithium.

Stop taking lithium straight away and go to her nearest accident and emergency department as soon as possible. The symptoms described are signs of lithium toxicity which is likely as the patient may have taken a double dose the day before. Due to the narrow therapeutic range and the severity of potential poisoning the patient requires urgent medical attention and should be referred to her nearest accident and emergency department.

Regarding the below storage conditions, how would you advise a patient to store the following medications once they have been opened? Daktacort® 30 g cream. No specific instructions Store at or below 25 °C Protect from frost and direct sunlight Store at 2-8 °C Store at or below 30 °C Store at -40°C Keep away from direct sunlight Store in a dark place

Store at 2-8 °C

Regarding the below storage conditions, how would you advise a patient to store the following medications once they have been opened? Reconstituted Augmentin® suspension. No specific instructions Store at or below 25 °C Protect from frost and direct sunlight Store at 2-8 °C Store at or below 30 °C Store at -40°C Keep away from direct sunlight Store in a dark place

Store at 2-8 °C

Regarding the below storage conditions, how would you advise a patient to store the following medications once they have been opened? Daktacort® ointment. No specific instructions Store at or below 25 °C Protect from frost and direct sunlight Store at 2-8 °C Store at or below 30 °C Store at -40°C Keep away from direct sunlight Store in a dark place

Store at or below 25 °C

Regarding the below storage conditions, how would you advise a patient to store the following medications once they have been opened? Ventolin accuhaler®. No specific instructions Store at or below 25 °C Protect from frost and direct sunlight Store at 2-8 °C Store at or below 30 °C Store at -40°C Keep away from direct sunlight Store in a dark place

Store at or below 30 °C

Mrs N explains that her 5-year-old niece has sores around her mouth and nose that quickly burst to leave a yellow-brown crust. Which bacterium do you suspect could have caused this infection? Helicobacter pylori Streptococcus pyogenes Escherichia coli Sarcoptes scabiei Streptococcus pneumoniae

Streptococcus pyogenes. The description the patient has given is that of impetigo secondary to infection with bacteria, usually. Staphylococcus aureus or Streptococcus pyogenes.

Evidence-based practice promotes better clinical outcomes for patients. To ensure that medicines and practice are evidence-based, a process of reviewing and appraising the literature is needed. Which of the following types of literature would be deemed to provide 'the best available evidence'? Case-control studies Cohort studies Expert opinion Randomised controlled trials Systematic reviews

Systematic reviews. Systematic reviews take into account all literature created, therefore, provide the best evidence in research.

For the patients described, select the most likely drug from the list below which may have caused the stated side-effect. Bezoar formation in a 59-year-old male who has slow gastric emptying. Ciclesonide Ranitidine Sucralfate Docusate sodium Prochlorperazine Felodipine Glyceryl trinitrate Carbocisteine

Sucralfate. A bezoar is a solid mass of indigestible material that accumulates in the digestive tract, sometimes causing a blockage. This can be a side effect of sucralfate.

You are running an educational workshop for patients with diabetes in the community. You receive a question from a patient regarding OTC medicines and diabetes. Which of the following medicines must be cautioned in diabetes due to affects on blood glucose? Beconase Dulcolax Nurofen 200 mg tablets Sterimar Sudafed decongestant tablets

Sudafed decongestant tablets. Pseudoephedrine must be used in caution in patients with diabetes. Pseudoephedrine may increase blood glucose levels.

Which ONE of the following is a toxic effect of theophylline? Tachycardia Watering eyes Drowsiness Bruising Goitre

Tachycardia

A 45-year-old patient, who works on the night shift, is prescribed a 10-day supply of temazepam 10 mg when required to help sleep. Which counselling point is the most appropriate for this patient for them to get the best effect from their medicine? Do not take with food Do not use for more than 3 weeks Take at night Take 30-60 minutes before you want to sleep This medicine may make you drowsy

Take 30-60 minutes before you want to sleep. 'Do not take with food' is irrelevant. 'Do not use for more than 3 weeks' is less important as the patient only has a 10 day supply. 'Take at night' is not appropriate as this is when the patient will need to be alert during the night to be able to work. 'Take 30-60 minutes before you want to sleep' is the most relevant as this information will best match their need. 'This medicine may make you drowsy' is less important as the indication is to help sleep.

You work within a community pharmacy that has very long opening hours. One afternoon you realise that you are getting tired and are making mistakes. What should you do? Shut the pharmacy Send patients away Drink a cup of coffee Take a break Call a locum to take over

Take a break. It would be appropriate to take a break initially to try and reduce the tiredness.

A 22-year-old female comes into your pharmacy for advice about contraceptive effectiveness. She has been taking a progesterone-only contraceptive pill (Cerazette®) regularly for the past 12 months. She took her pill this morning at 7 am but started to feel unwell and vomited at 9.15 am. She is concerned about contraceptive failure. Which of the following is the most appropriate advice for this lady? Emergency hormonal contraception is required. No further action required. Recommence Cerazette® when she feels better. Take another Cerazette® as soon as possible. Use barrier contraception for the next 7 days.

Take another Cerazette® as soon as possible

Intestinal absorption of which ONE of the following drugs may be impaired when taken with magnesium trisilicate? Amoxicillin Cefalexin Co-amoxiclav Tetracycline Metronidazole

Tetracycline

Most instances of nausea and vomiting are acute and short-lived. Which scenario would alert you to potentially refer the patient? Symptoms of less than 24 hours in an adult. Symptoms of more than 24 hours but less than 36 hours in an adult. Symptoms of up to 48 hours in an adult. Symptoms of more than 24 hours in a 5 year old. Symptoms of more than 24 hours in a 9 month old.

Symptoms of more than 24 hours in a 9 month old. Age is an important consideration and the young should be viewed with greater caution in management, especially the very young as they are at greater risk of complications such as dehydration. In this case symptoms of more than 24 hours in a 9 month old would be the most appropriate answer.

For each of the following patients, select the drug which is most likely to be responsible for the symptoms described. Mrs R, a 73-year-old retired teacher, has reported extreme lethargy and dizziness 7 days after starting a second antihypertensive. She has already been taking atenolol for 10 years. Her heart rate was reported to be 45 bpm and her blood pressure as 75/40 mmHg. Amlodipine Atenolol Bendroflumethiazide Bumetanide Doxazosin Lisinopril Methyldopa Verapamil

Verapamil Both verapamil and atenolol can increase the risk of bradycardia. This is a severe interaction.

Which ONE of the following drugs is NOT suitable for both supraventricular and ventricular arrhythmias? Verapamil Sotalol Amiodarone Flecainide Propafenone

Verapamil. Verapamil is usually effective for supraventricular tachycardias only.

A young man visits the pharmacy and asks you to look at an inward growth on the sole of his foot. You notice that the lesion has tiny black dots on its surface. He says that it is painful when he walks. Which of the following conditions is he most likely to be suffering from? Athlete's foot Basal cell carcinoma Corn Molluscum contagiosum Verruca

Verruca. Symptoms indicative of verrucae/wart like growth with black dots in the middle.

Which of the following prescribers CANNOT request an emergency supply of a prescription-only medicine? Supplementary nurse prescriber Swiss doctor Vet Dentist Community nurse prescriber

Vet

For each of the following statements, select the drug that fits most closely. Patients who are newly started on this drug should be warned they may need ophthalmological referral should they develop side effects. Desmopressin Diazepam Flucloxacillin Lamotrigine Misoprostol Paroxetine Salbutamol Vigabatrin

Vigabatrin As this drug may cause visual field defects, it is important that patients are aware of the need they may need eye check ups during treatment.

For each of the following statements, select the drug that fits most closely. Treatment of this drug may cause visual field defects. Desmopressin Diazepam Flucloxacillin Lamotrogine Misoprostol Paroxetine Salbutamol Vigabatrin

Vigabatrin Patients using this drug must have regular eye checks as the use of this drug may cause visual field defects.

For each of the following patients, select the condition they are likely to be suffering from. A lady visits your pharmacy. She is just recovering from a flu and has noticed that both of her eyes appear red. She tells you that she has a watery discharge coming from each eye. Bacterial conjunctivitis Basel cell carcinoma Blepharitis Corneal ulcer Ectropion Stye Subconjunctival haemorrhage Viral conjunctivitis

Viral conjunctivitis Viral conjunctivitis usually associated with a cold or flu. Watery discharge reported.

The absorption of which one of the following vitamins is NOT decreased by orlistat? Vitamin A Vitamin B Vitamin D Vitamin E Vitamin K

Vitamin B. Vitamins ADEK are fat soluble and orlistat may reduce their absorption.

Which of the listed vitamins and minerals should be prescribed to the following patients? Wernicke's encephalopathy secondary to chronic alcoholism is treated in the long term with the oral administration of this. Vitamin E Vitamin K Vitamin A Vitamin B12 Calcium Vitamin B1 Phosphate Vitamin B6

Vitamin B1

Which of the listed vitamins and minerals should be prescribed to the following patients? A 35-year-old female suffered from a history of pernicious anaemia (in which a lack of gastric intrinsic factor results from an autoimmune gastritis). This caused the malabsorption of a vitamin and subsequently megaloblastic anaemia. Vitamin E Vitamin K Vitamin A Vitamin B12 Calcium Vitamin B1 Phosphate Vitamin B6

Vitamin B12

For each of the following questions or statements, select the illness which is correct. Vaccination against this disease is usually given within the first year of life in a combination vaccine with diphtheria, tetanus, haemophilus influenza type b and polio. Atopic eczema Chicken pox (varicella) German measles (rubella) Impetigo Measles (morbilli) Mumps (epidemic parotitis) Slap cheek syndrome (parvovirus) Whooping cough (pertussis)

Whooping cough (pertussis) Given as a single injection (at 8 weeks, 12 weeks and 16 weeks) containing vaccines to protect against six separate diseases: diphtheria; tetanus; whooping cough (pertussis); polio; Haemophilus influenzae type b (Hib); and hepatitis B.

Which of the following symptoms is most likely to be attributed to prednisolone? Myositis Constipation Weight gain Bruising that may be heavy Black stools

Weight gain. Increased appetite is a pronounced side-effect of Prednisolone. Myositis reflects inflammation in the muscle possibly resulting from muscle damage. That is distinct from degradation of the muscle through chronic steroid use and is therefore not the correct answer.

Pharmacists have a responsibility to safeguard children and vulnerable adults. You are working in a pharmacy based in England. Under which circumstance are you most likely to be legally obliged to report a case of female genital mutilation to the police? When an adult female reports female genital mutilation having taken place to them. When a child is at risk of female genital mutilation. When a child reports it having happened to their friend. When a child reports it having happened to themselves. When a pharmacist suspects that female genital mutilation has taken place to a child.

When a child reports it having happened to themselves. Although for all situations consideration should be given to reporting to the police, the legal obligation is: "All pharmacy professionals now have a legal duty to report, orally or in writing, to the police if: they observe physical signs that female genital mutilation (FGM) may have taken place in a girl under the age of 18, or a child or young person tells them that they have had FGM."

Which one of the following statements regarding the treatment of nocturnal enuresis in a 7-year-old is FALSE? Avoid caffeine-based drinks before going to bed. Ensure the child eats a healthy diet. Encourage the child to use the toilet to pass urine at regular intervals during the day and before sleep. Reward the child for going to the toilet before bed. When the enuresis alarm goes off the child should go back to sleep if there is no urge to urinate.

When the enuresis alarm goes off the child should go back to sleep if there is no urge to urinate

You are creating a local prescribing guidance document on hypertension. What is the recommended target blood pressure for: Hypertension in type 1 diabetes with kidney problems present.

≤130/80 mmHg Adult with type 1 diabetes and albuminuria or 2 or more features of metabolic syndrome, in which case it should be 130/80 mmHg or less.

You are labelling prescriptions in your pharmacy. What is NOT legally required to appear on the dispensing label? Date of dispensing Directions for use "Keep out of the sight and reach of children" Name and address of dispensing pharmacy Name of medicine

"Keep out of the sight and reach of children" "Keep out of the sight and reach of children" is advised by the Royal Pharmaceutical Society, it is not a legal requirement.

A patient is given a single intravenous dose of drug A and their peak serum concentration is found to be 500 mcg/mL. The elimination half-life of this drug is 240 minutes. What will be their drug A serum concentration 12 hours after the dose is given? (You can assume that the distribution was complete and the elimination is described by a 1st order process) 0.0625 mg/10 mL 0.0625 g/L 6.25 mcg/mL 625 mg/L 6.25 mg/mL

0.0625 g/L. Half life is the time to reduce serum concentration by half. Half life = 240 mins (4 hours) In 12 hours, the serum concentration will have reduced by 3 half lives. Starting point = 500 mcg/mL First half life = 250 mcg/mL Second half life = 125 mcg/mL Third half life = 62.5 mcg/mL At 12 hours the serum concentration will be 62.5 mcg/mL Conversion to answer: 62.5 mcg per ml is equal to 0.0625 mg per mL (divide mcg by 1000 to get mg) 0.0625 mg per mL is equal to 62.5 mg per L (times mg by 1000 to get mg in 1 L) 62.5 mg per L is equal to 0.0625 g per L (divide mg by 1000 to get g).

Whilst working in a community pharmacy you receive a phone call from a community nurse practitioner. She wishes to prescribe an Epipen for an adult in case of anaphylaxis. What is the strength of Epipen that is used in adults? 0.015 mg 0.03 mg 0.15 mg 0.3 mg 3 mg

0.3 mg The adult strength Epipen comes in the 0.3 mg strength.

What is the volume of enoxaparin sodium 1 in 10 solution that should be subcutaneously injected to deliver a daily dose of 40 mg? 0.4 mL 4 mL 20 mL 250 mL 590 mL

0.4 mL. 1 in 10 solution = 1 g in 10 mL = 1000 mg in 10 mL = 100 mg in 1 mL = 10 mg in 0.1 mL = 40 mg in 0.4 mL.

How many times in total (i.e. first dispensing plus repeats) can you dispense the following prescriptions? A private prescription for a Schedule 2 CD stating repeat x 3. 0 1 2 3 4 5 6 7

1 It is not possible to repeat prescriptions for Schedule 2 CDs.

Based on the below days/packs supplied, what is the maximum number of days/packs of medication that you could legally give to a patient as an emergency supply for the following medications, if any? Tilade® CFC-free Inhaler. Please give number of pack(s). 0 1 3 5 14 21 28 30

1 One smallest pack of an aerosol dispenser could be supplied as an emergency supply.

How many times in total (i.e. first dispensing plus repeats) can you dispense the following prescriptions? A private prescription for a combined hormonal contraceptive where the word "repeat" is NOT stated on the prescription. 0 1 2 3 4 5 6 7

1 Private prescriptions where the prescriber has not added the instruction to "repeat" are not valid for repeatable dispensing. The MEP states that repeatable prescriptions are those "where the prescriber adds an instruction to the main prescription for the prescribed item to be repeated". The exemption for oral contraceptives only applies to a prescription where the word "repeat" is stated, but the number of repeats required is not. In this question the prescriber has made no repeat request and hence the prescription cannot be repeated.

How many times in total (i.e. first dispensing plus repeats) can you dispense the following prescriptions? A private prescription for a schedule 4 drug where the word "repeat" is NOT stated on the prescription. 0 1 2 3 4 5 6 7

1 Private prescriptions where the prescriber has not added the instruction to "repeat" are not valid for repeatable dispensing. The MEP states that repeatable prescriptions are those "where the prescriber adds an instruction to the main prescription for the prescribed item to be repeated". In this question the prescriber has made no repeat request and hence the prescription cannot be repeated.

How many times can you repeat dispense the following prescriptions? A private prescription for bendroflumethiazide with the word "repeat" stated in the body of the prescription. 0 1 2 3 4 5 6 7

1 Where the number of repeats is not stated, the prescription can only be repeated once.

As part of your pharmacist check on a prescription, you are required to ensure that the prescription is supplied within its valid date. What is the first date you could supply each of the following prescriptions? An NHS prescription for 100 x paracetamol 500 mg dated 1 May 2021. 30 April 2021 1 May 2021 2 May 2021 10 May 2021 11 May 2021 28 May 2021 29 May 2021 30 May 2021

1 May 2021 The appropriate date of an NHS prescription is either the signature date or the date indicated by the prescriber as a date before which the medicine should not be supplied, whichever is the later.

As part of your pharmacist check on a prescription, you are required to ensure that the prescription is supplied within its valid date. What is the first date you could supply each of the following prescriptions? A private prescription for amoxicillin capsules 250 mg signed 1 May 2021, with the words "supply on 10 May" in the body of the prescription. 30 April 2021 1 May 2021 2 May 2021 10 May 2021 11 May 2021 28 May 2021 29 May 2021 30 May 2021

1 May 2021 The appropriate date of an NHS prescription is either the signature date or the date indicated by the prescriber as a date before which the medicine should not be supplied, whichever is the later. However, for a private prescription the appropriate date will always be the date on which it was signed.

As part of your pharmacist check on a prescription, you are required to ensure that the prescription is supplied within its valid date. What is the first date you could supply each of the following prescriptions? A private prescription for 100 x paracetamol 500 mg dated 1 May 2021. 30 April 2021 1 May 2021 2 May 2021 10 May 2021 11 May 2021 28 May 2021 29 May 2021 30 May 2021

1 May 2021 The appropriate date on a private prescription is the date it is signed by the prescriber.

From the below list, what is the incubation period for these infectious illnesses in children? Slapped cheek disease No incubation period 1-20 days 5-14 days 14-25 days 1-3 weeks 4-6 weeks 2 months A couple of years

1-20 days

From the below list, what is the incubation period for these infectious illnesses in children? Chickenpox No incubation period 1-20 days 5-14 days 14-25 days 1-3 weeks 4-6 weeks 2 months A couple of years

1-3 weeks

From the below list, what is the incubation period for these infectious illnesses in children? Whooping cough No incubation period 1-20 days 5-14 days 14-25 days 1-3 weeks 4-6 weeks 2 months A couple of years

1-3 weeks

How many NHS prescription charges do 28 x warfarin 1 mg tablets and 56 x warfarin 5 mg tablets levy (assuming the patient is NOT exempt from NHS charges) for an NHS prescription written in England? 0 1 2 3 4

1. Same drug with the same formulation, but different strengths is classed as one prescription charge.

What is the last date you could dispense the following prescriptions? A private prescription for atenolol dated 30/09/21. 30/06/21 28/07/21 30/09/21 30/03/22 30/06/22 30/06/26 Repeats are not allowed There is no legally defined final dispensing date, it is based on clinical judgement

30/03/22 Prescriptions are valid for 6 months from the appropriate date.

What is the last date you could dispense the following prescriptions? An NHS prescription for atenolol dated 30/09/21. 30/06/21 28/07/21 30/09/21 30/03/22 30/06/22 30/06/26 Repeats are not allowed There is no legally defined final dispensing date, it is based on clinical judgement

30/03/22 Prescriptions are valid for 6 months from the appropriate date.

The amount of elemental iron contained in one ferrous fumarate 200 mg tablet is what? 35 mg 55 mg 60 mg 65 mg 75 mg

65 mg

Ibuprofen is a commonly used NSAID medication to help relieve pain and inflammation, which can be bought GSL. What is the maximum TOTAL daily dose of Ibuprofen in an adult when used as a GSL product? 1.2 g 1.6 g 2 g 2.2 g 2.4 g

1.2g Adults can take 200 mg-400 mg every FOUR hours if required, however, they must not exceed 1.2 g in 24 hours when taking over-the-counter. The BNF doses are much higher as they are usually prescription only and patient should be under the care of a physician.

You receive a prescription for morphine sulfate tablets 10 mg. What is the most appropriate wording to meet legal requirements for the total quantity on the prescription when the prescriber wants you to give a total of 10 tablets? 10 tablets Ten tablets 100 mg (one hundred mg) 10 (ten) tablets 100 mg

10 (ten) tablets The total quantity must be expressed in words and figures and this should preferably be as the number of dosage units.

You receive an NHS prescription for a Schedule 2 controlled drug. The prescription is signed on 6 March 2021. The prescriber has written in the body of the prescription "Supply on 10 April 2021". For each of the questions select the correct answer. Each option may be used once, more than once or not at all. What is the first date you can dispense this prescription? 5 March 2021 6 March 2021 7 March 2021 2 April 2021 3 April 2021 4 April 2021 10 April 2021 7 May 2021

10 April 2021. The appropriate date of a NHS CD prescription is either the signature date or the date indicated by the prescriber as a date before which the medicine should not be supplied, whichever is the later. The 28 day validity of a CD prescription runs from the appropriate date.

As part of your pharmacist check on a prescription, you are required to ensure that the prescription is supplied within its valid date. What is the first date you could supply each of the following prescriptions? An NHS prescription for amoxicillin capsules 250 mg signed 1 May 2021, with the words "supply on 10 May" in the body of the prescription. 30 April 2021 1 May 2021 2 May 2021 10 May 2021 11 May 2021 28 May 2021 29 May 2021 30 May 2021

10 May 2021 The appropriate date of an NHS prescription is either the signature date or the date indicated by the prescriber as a date before which the medicine should not be supplied, whichever is the later.

In each of the following scenarios select the most appropriate treatment option for the patient. A 55-year-old gentleman is undergoing elective knee surgery. The doctor would like to prescribe dabigatran for the prevention of venous thromboembolism. The patient has a creatinine clearance of 68 mL/min and he weighs 65 kg. How long should the patient take the maintenance treatment for? 7 days 10 days 14 days 21 days 28 days 90 days 180 days 365 days

10 days Creatinine clearance = 68 mL/min, indicating no dose adjustment is required. For patients following elective knee replacement surgery it is advised to give a maintenance dose starting on the first day after surgery of 220 mg once daily taken as 2 capsules of 110 mg for 10 days.

In each of the following scenarios select the most appropriate treatment option for the patient. A 65-year old-gentleman is undergoing elective knee surgery. The doctor would like to prescribe dabigatran for the prevention of venous thromboembolism. The patient has a creatinine clearance of 46 mL/min and he weighs 75 kg. How long should the patient take the maintenance treatment for? 7 days 10 days 14 days 21 days 28 days 90 days 180 days 365 days

10 days. For patients with moderate renal impairment (creatinine clearance (CrCL 30-50 mL/min) it is advised to give a maintenance dose starting on the first day after surgery of 150 mg once daily for 10 days.

Chloramphenicol is an antibiotic which can be bought over the counter to treat acute bacterial conjunctivitis. What is the maximum pack size of chloramphenicol 0.5% eye drops which can be sold over the counter? 3 mL 5 mL 7 mL 10 mL 20 mL

10 mL. Chloramphenicol may be sold OTC for the treatment of bacterial conjunctivitis. The total treatment pack for OTC chloramphenicol must not exceed 10 mLs.

What age is hydrocortisone 1% (P) cream licensed from? 2 years 10 years 12 years 16 years 18 years

10 years. OTC hydrocortisone 1% is licensed from the age of 10 years. Over-the-counter hydrocortisone preparations should not be sold without medical advice for children under 10 years.

A 6-month-old child with no long-term medical conditions requires paracetamol for the treatment of pyrexia associated with flu-like symptoms. What is the most appropriate dose of paracetamol to be administered every six hours? 60 mg 120 mg 180 mg 240 mg 360 mg

120 mg Paracetamol dose for child 6-23 months: 120 mg every 4-6 hours; maximum 4 doses per day.

You offer a patient group direction (PGD) for emergency contraception in your pharmacy. The PGD allows for supply to those under 16 years of age. Under what age are children too young to legally consent to sexual activity and consideration should be given to raising the issue with social services? 10 years 12 years 13 years 16 years 18 years

13 years. Children under the age of 13 are legally too young to consent to sexual activity and consideration should be given to reporting to Social Services. Sexual activity with children under the age of 16 is an offence but may be consensual.

From the below list, what is the incubation period for these infectious illnesses in children? Mumps No incubation period 1-20 days 5-14 days 14-25 days 1-3 weeks 4-6 weeks 2 months A couple of years

14-25 days

Mr O is a patient living with HIV, he is 29 years old and weighs 70 kg (he is not obese). Mr O is prescribed aciclovir intravenous infusion for prophylaxis of herpes simplex for 5 days at a dose of 5 mg/kg every 8 hours. The nurse on the ward asks you to dispense the full 5 day supply. The pharmacy department stocks only 250 mg and 500 mg vials. N.B. Vials are to be reconstituted immediately before use and any remainder discarded. How many vials of aciclovir should you dispense? 10 Aciclovir 250 mg vials and 10 Aciclovir 500 mg vials 15 Aciclovir 250 mg vials 15 Aciclovir 500 mg vials 15 Aciclovir 250 mg vials and 15 Aciclovir 500 mg vials 40 Aciclovir 500 mg vials

15 Aciclovir 500 mg vials. Dose for herpes simplex is 5 mg/kg every 8 hours therefore 70 x 5 = 350 mg dose requires 500 mg vials to be dispensed therefore 3 vials per day for 5 days = 15 aciclovir 500 mg vials. In this situation the examiners believe that 15 aciclovir 500 mg is the best answer; this is because it would probably be easier to reconstitute and have potential for less errors - mixing the type of vials could lead to errors in reconstitution.

A 56-year-old lady weighing 66 kg is prescribed norepinephrine base 600 micrograms per hour by the ITU doctor. The nurse looking after the patient makes up the infusion containing 20 mg of norepinephrine base in 500 mL of Glucose 5%. At what rate should the nurse set the controlled infusion device to administer the prescribed dose? 1.5 mL/hour 7.5 mL/hour 15 mL/hour 75 mL/hour 150 mL/hour

15 mL/hour 20 mg = 20,000 micrograms therefore 600 / 20,000 x 500 mL = 15 mL/hour.

From the selection, what is the minimum age suitable for sale of the following? Nurofen® for Children 100 mg/5 mL. 2 months 3 months 6 months 12 years 16 years 18 years 45 years 65 years

18 years

Patient is 52 years old. He has type 2 diabetes and has been taking metformin 2g and pioglitazone 30mg. His GP wants to add a third agent what can he add?

1st line = metformin 1st intensification is sulfonylurea 2nd intensification is either sulfonylurea or SGLT2 inhibitors. Canagliflozin is best option

How many times in total (i.e. first dispensing plus repeats) can you dispense the following prescriptions? A private prescription for fluconazole 200 mg capsules with the word "repeat" stated. 0 1 2 3 4 5 6 7

2 Private prescriptions without the number of repeats stated can be repeated once - so dispensed twice in total.

This question is about the Responsible Pharmacist regulations. How long is a pharmacist legally allowed to be absent from a pharmacy in which they are the Responsible Pharmacist for the whole of the working day? 1 hour 2 hours 3 hours 4 hours Not at all

2 hours. Responsible Pharmacist Regulations 2008 - 2 hour absence allowed within a 24 hour period from midnight to midnight.

How soon after stopping the phenelzine can Mrs AL start taking the fluoxetine? Immediately 1 week 2 weeks 3 weeks 4 weeks

2 weeks

What is the minimum period of time you should keep the following? Controlled drugs register. No record kept 1 day 1 month 3 month 6 months 1 year 2 years 5 years

2 years CD register should be kept for a minimum of two years after the date of the last entry.

Your responsibility as the superintendent pharmacist for a pharmacy is to ensure that all record keeping is appropriate. What is the legal requirement for how long the CD register should be kept for from the date of last entry? 1 month 2 months 6 months 1 year 2 years 3 years 5 years 10 years

2 years CD registers should be kept for 2 years from the date of last entry.

What is the minimum period of time you should keep the following? Signed order for a non-CD medicine No record kept 1 day 1 month 3 month 6 months 1 year 2 years 5 years

2 years Signed orders should be kept for two years from the date of supply.

Your responsibility as the superintendent pharmacist for a pharmacy is to ensure that all record keeping is appropriate. What is the maximum interval your responsible pharmacist should leave for reviewing the pharmacy procedures? 1 month 2 months 6 months 1 year 2 years 3 years 5 years 10 years

2 years Standard operating procedures (SOPs) should be reviewed at least every 2 years.

You are making a record of the destruction of expired schedule 2 controlled drugs that you have in your pharmacy controlled drugs cabinet. What is the minimum period of time that this individual record of destruction has to be kept? 1 year 2 years 5 years 7 years 10 years

2 years. Records of CD drugs destroyed that are stock items must be kept for 2 years from the date of destruction. For patient return medicines the minimum recommended duration is 7 years from the date of destruction.

What is the minimum period of time you should keep the following? Private prescriptions for a non-CD medicine. No record kept 1 day 1 month 3 month 6 months 1 year 2 years 5 years

2 years. Private prescription records should be kept for two years from the date of dispensing.

You are counselling a patient on the use of her newly prescribed conventional DMARD. How long is it until she is most likely to see an effect from her treatment? 2-3 days 1-2 weeks 2-3 weeks 1-2 months 2-3 months

2-3 months. Conventional DMARDs have a slow onset of action and can take 2-3 months to take effect.

How many NHS prescription charges would you levy for a prescription of amoxicillin 500 mg capsules and amoxicillin 250 mg/5 mL syrup (assuming the patient is NOT exempt for NHS charges) for an NHS prescription written in England? 0 1 2 3 4

2. Same drug but different formulation is classed as a charge per item, therefore two prescription charges.

A doctor contacts you in relation to analgesia and maximum dosing of analgesics that can be safely prescribed under their care. They wish to use an anti-inflammatory and would like to know the maximum daily dose. What is the maximum TOTAL daily dose of immediate release ibuprofen for an adult? 1.2 grams 1.6 grams 1.8 grams 2.4 grams 2.8 grams

2.4 grams. The maximum dose of Ibuprofen that can be prescribed as per BNF is 600 mg q.d.s. Which equates to a maximum daily dose of 2.4 grams.

Miss J is the new pharmacy summer student at your pharmacy. You are teaching her about the common dosages of various drugs. For the drugs and indications listed, select the most commonly prescribed doses from the list below. Tamoxifen for breast cancer. 400 micrograms once daily 5 mg once in the morning 10 mg once a week 7.5 mg once daily 20 mg once daily 70 mg once a week 500 mg twice a day 1 g three times a day

20 mg once daily

Regarding the dosing of ibuprofen, choose the appropriate dosing for the indication described. A 19-year-old female presenting to you over the counter with period pain that she describes as 4/10 on a pain scale. 200 mg TDS 200-400 mg TDS 300-400 mg TDS 400-600 mg single dose 1.6 g OD 1.2 g BD 1.6 g BD 2.4 g OD

200-400 mg TDS Mild dysmenorrhoea is dosed initially at 300-400 mg 3-4 times a day according to the BNF, but as the patient is presenting OTC the dosing for the OTC product is 200-400 mg TDS max 6 tablets in 24 hours.

Regarding the dosing of ibuprofen, choose the appropriate dosing for the indication described. A 34-year-old male who is suffering from the flu and presents in your pharmacy asking for advice. 200 mg TDS 200-400 mg TDS 300-400 mg TDS 400-600 mg single dose 1.6 g OD 1.2 g BD 1.6 g BD 2.4 g OD

200-400 mg TDS The dosing for the OTC product is 200-400 mg TDS max 6 tablets in 24 hours.

Based on the below days/packs supplied, what is the maximum number of days/packs of medication that you could legally give to a patient as an emergency supply for the following medications, if any? Cilest®. Please give number of day(s). 0 1 3 5 14 21 28 30

21 A full cycle of an oral contraceptive could be given as an emergency supply. Cilest® is a 21-day preparation.

You owe a patient some medicine from dispensing a prescription. Which date should the following medicines be fully dispensed by? Methadone oral liquid 1 mg/1 mL on an NHS (non-instalment) prescription signed 1/11/20 with the instruction to dispense on 25/12/20. 27/11/20 28/11/20 21/01/21 24/01/21 30/04/21 01/05/21 25/06/21 30/10/21

21/01/21 Owings must be dispensed within the validity of a prescription. The appropriate date of a NHS CD prescription is either the signature date or the date indicated by the prescriber as a date before which the medicine should not be supplied, whichever is the later. The 28 day validity of a CD prescription runs from the appropriate date.

For each of the following situations, select the single most likely timeline. A requisition for a controlled drug which has been requested as an emergency by a doctor should be furnished within this many hours. 0 2 5 6 7 24 30 72

24 In an emergency, a doctor or dentist maybe supplied with a Sch 2 or 3 CD on the undertaking that a requisition will be furnished within 24 hours.

You regularly dispense methotrexate for this patient. What would be appropriate counselling for the patient? He calls the pharmacy to state he is stopping his methotrexate treatment to try for a baby. He would like to know how long he needs to continue to use contraception for. 1 week 4 weeks 30 days 6 weeks 3 months 90 days 26 weeks One year

26 weeks. Sexually active male patients or their female partners are recommended to use reliable contraception during treatment of the male patient and for at least 6 months after cessation of methotrexate. Men should not donate semen during therapy or for 6 months following discontinuation of methotrexate.

You regularly dispense methotrexate for this patient. What would be appropriate counselling for the patient? She calls the pharmacy to state she is stopping her methotrexate treatment to try for a baby. She would like to know how long she needs to continue to use contraception for. 1 week 4 weeks 30 days 6 weeks 3 months 90 days 26 weeks One year

26 weeks. Women must not get pregnant during methotrexate therapy, and effective contraception must be used during treatment with methotrexate and at least 6 months thereafter.

In each of the following scenarios select the most appropriate treatment option for the patient. An 80-year-old gentleman is undergoing an elective hip replacement. The doctor would like to prescribe dabigatran for the prevention of venous thromboembolism. The patient's creatinine clearance is 55 mL/min and he weighs 75 kg. How long should the patient take the maintenance treatment for? 7 days 10 days 14 days 21 days 28 days 90 days 180 days 365 days

28 days For patients aged 75 or above it is advised to give a maintenance dose starting on the first day after surgery of 150 mg once daily taken as 2 capsules of 75 mg for 28-35 days.

In each of the following scenarios select the most appropriate treatment option for the patient. A 55-year-old lady is undergoing an elective hip replacement. The doctor would like to prescribe dabigatran for the prevention of venous thromboembolism. The patient has a creatinine clearance of 47 mL/min and she weighs 65 kg. How long should the patient take the maintenance treatment for? 7 days 10 days 14 days 21 days 28 days 90 days 180 days 365 days

28 days For patients with moderate renal impairment (creatinine clearance (CrCL 30-50 mL/min) it is advised to give a maintenance dose starting on the first day after surgery of 150 mg once daily for 28-35 days.

You owe a patient some medicine from dispensing a prescription. Which date should the following medicines be fully dispensed by? Morphine sulfate tablets 100 mg on an NHS prescription signed 1/11/20. 27/11/20 28/11/20 21/01/21 24/01/21 30/04/21 01/05/21 25/06/21 30/10/21

28/11/20 Owings must be dispensed within the validity of a prescription. The appropriate date of a NHS CD prescription is either the signature date or the date indicated by the prescriber as a date before which the medicine should not be supplied, whichever is the later. The 28 day validity of a CD prescription runs from the appropriate date.

For the patients described, select the most likely contraindication or caution from the list below. Ganciclovir. 35-year-old male with unstable myasthenia gravis 58-year-old female who regularly drinks ginger tea 45-year-old female with blood pressure of 187/112 mmHg 38-year-old female with a haematoma on her upper arm 29-year-old male with an abnormally low neutrophil count 21-year-old male with type 1 diabetes mellitus 18-year-old male 72-year-old male with erectile dysfunction

29-year-old male with an abnormally low neutrophil count Ganciclovir is contraindicated in patients with an abnormally low neutrophil count.

Based on the below days/packs supplied, what is the maximum number of days/packs of medication that you could legally give to a patient as an emergency supply for the following medications, if any? Rifaximin for travellers' diarrhoea. Please give number of day(s). 0 1 3 5 14 21 28 30

3 For an orally administered liquid antibiotic, the smallest quantity to provide a full course should be given. For tablet antibiotics, the maximum supply is 30 days treatment, but a pharmacist should consider the appropriateness regarding the amount they supply. For the case of an antibiotic with a fixed course, the amount to complete the course would be appropriate - which at the most would be 3 days.

How many times can you repeat dispense the following prescriptions? A private prescription for bendroflumethiazide with "repeat x 3" stated in the body of the prescription. 0 1 2 3 4 5 6 7

3 Private prescriptions can be repeated the number of times requested by the prescriber.

From the selection, what is the minimum age suitable for sale of the following? Nurofen® for Children 100 mg/5 mL. 2 months 3 months 6 months 12 years 16 years 18 years 45 years 65 years

3 months

A 68-year-old man weighing 70 kg is administered drug X 500 nanograms/kg/minute. At intervals of 15 minutes it is increased up to 2 micrograms in increments of 500 nanograms. How much drug X in total will the patient have received at 45 minutes? 315 mg 158 mg 1.58 g 3.06 g 3.15 mg

3.15 mg. (500 nanograms/kg x 15 minutes) + (1000 nanograms/kg x 15) + (1500 nanograms/kg x 15) = 45,000 nanograms/kg = 45 mcg/kg, therefore 45 mcg x 70 = 3150 mcg = 3.15 mg.

A junior doctor wishes to prescribe morphine sulfate on a discharge prescription following a hip replacement for a 45-year-old patient. What is the maximum number of days that should be prescribed in this instance? 7 days 10 days 14 days 28 days 30 days

30 days. It is strongly recommended that the maximum quantity of Schedule 2, 3 or 4 CDs prescribed should not exceed 30 days.

As part of your registration as a pharmacist you are required to re-validate your practice. How many CPD entries are you required to submit per year? 1 4 7 9 10

4

You are providing a pharmacy-led rheumatology service. What is the most appropriate length of time for patients be monitored for infections following cessation of adalimumab? The SPC is provided here: https://www.medicines.org.uk/emc/product/7986/smpc 1 month 2 months 4 months 6 months 12 months

4 months. As the elimination of adalimumab may take up to 4 months, monitoring should be continued throughout this period.

How many NHS prescription charges do compression hosiery 2 pairs levy (assuming the patient is NOT exempt from NHS charges) for an NHS prescription written in England? 0 1 2 3 4

4. With stockings/hosiery, one charge applies per piece of hosiery. So 2 pairs = 4 pieces, therefore, 4 charges should be levied.

A male patient has been prescribed fludarabine phosphate for chronic lymphocytic leukaemia at a dose of 40 mg/m2 daily for five days. This patient weighs 80 kg and his height is 1.8 m. Which of the following is an appropriate number of Fludara® 10 mg tablets for you to dispense to this patient for the five days of treatment? Body Surface Area (m2) = √ [(height {cm}) x weight {kg})/ 3600] 5 10 25 40 80

40. BSA (m2) = √{(180 x 80) / 3600} =√4 = 2 m2. Fludarabine phosphate dose is 40 mg/m2daily for 5 days. Each fludarabine phosphate dose = 40 mg x 2 = 80 mg. Tablet strength = 10 mg. Number of tablets per dose = 80 / 10 = 8 tablets. Over treatment period of 5 days patient will take 8 tablets daily. Therefore should dispense 5 x 8 Fludara® 10 mg tablets = 40 tablets.

Miss J is the new pharmacy summer student at your pharmacy. You are teaching her about the common dosages of various drugs. For the drugs and indications listed, select the most commonly prescribed doses from the list below. Tamsulosin hydrochloride for benign prostatic hyperplasia. 400 micrograms once daily 5 mg once in the morning 10 mg once a week 7.5 mg once daily 20 mg once daily 70 mg once a week 500 mg twice a day 1 g three times a day

400 micrograms once daily

Regarding the dosing of ibuprofen, choose the appropriate dosing for the indication described. A patient presenting to the GP with an acute migraine, described as 6/10 on a pain scale and is suffering severe symptoms such as vomiting and dizziness. The patient has tried over-the-counter treatment but it has not helped much with the pain. 200 mg TDS 200-400 mg TDS 300-400 mg TDS 400-600 mg single dose 1.6 g OD 1.2 g BD 1.6 g BD 2.4 g OD

400-600 mg single dose Acute migraine By mouth using immediate-release medicines For Adult 400-600 mg for 1 dose, to be taken as soon as migraine symptoms develop.

For the patients described, select the most likely contraindication or caution from the list below. Ergotamine. 35-year-old male with unstable myasthenia gravis 58-year-old female who regularly drinks ginger tea 45-year-old female with blood pressure of 187/112 mmHg 38-year-old female with a haematoma on her upper arm 29-year-old male with an abnormally low neutrophil count 21-year-old male with type 1 diabetes mellitus 18-year-old male 72-year-old male with erectile dysfunction

45-year-old female with blood pressure of 187/112 mmHg Ergotamine is contraindicated in inadequately controlled hypertension.

A patient requires a 500 mL infusion bag containing dopamine 0.1% w/v. Using a 40 mL vial of dopamine 1.25% w/v, what volume of glucose 5% is required to make up the infusion bag? 4.6 mL 46 mL 460 mL 4,600 mL 46,0000 mL

460 mL. 1.25% solution means 1.25 g in 100 mL or 1250 mg of dopamine in 100 mL of dopamine solution; therefore 500 mg in the 40 mL vial. To make the 500 mg into a 0.1% w/v solution we must dilute it by x 1000. Thus, the vial contents will make 500 mL at this concentration, and so we need to add 460 mL to the 40 mL from the vial. Fortuitously, 500 mL fills the bag specified in the question exactly without leaving a surplus or needing augmentation.

There are several reasons for withdrawing glucocorticoids slowly. Which option, when stopped suddenly, is the most likely to cause withdrawal symptoms? 40 mg prednisolone daily for 5 days. 25 mg prednisolone daily for 14 days. 10 mg prednisolone daily for 21 days. 20 mg prednisolone daily for 10 days. 50 mg prednisolone daily for 10 days

50 mg prednisolone daily for 10 days. Gradual withdrawal should be considered for patients receiving more than 40 mg prednisolone (or equivalent) daily for more than 1 week.

How many times in total (i.e. first dispensing plus repeats) can you dispense the following prescriptions? A private prescription for a combined hormonal contraceptive with the word "repeat" stated on the prescription. 0 1 2 3 4 5 6 7

6 Private prescriptions for combined hormonal contraceptives that state "repeat" can be dispensed a total of 6 times.

For each of the following situations, select the single most likely timeline. The invoice for Sativex® is recommended by NICE to be retained for this length of years for the purpose of HM Revenue and Custom. 0 2 5 6 7 24 30 72

6 Sativex is a Sch 4 part I, thus no requirement to retain invoice. However, NICE recommends to retain for 6 years for HM Tax and Custom purposes.

A 79-year-old non-obese patient weighing 60 kg requires a loading dose of drug X. Which one of the following is the most suitable intravenous loading dose of drug X for this patient? Vd = 0.7 L/kg. Therapeutic range 10-15 mg/L. 600 mg 700 mg 800 mg 900 mg 1000 mg

600 mg. Vd = 0.7 x 60 kg = 42 L. Therapeutic range is 10-15 mg/L; Applying this to this patient equates to a therapeutic range of (10 x 42) - (15 x 42) mg = 420-630 mg. The only option within this range is 600 mg.

For each of the following situations, select the single most likely timeline. A prescription for isotretinoin capsules for a 25-year-old woman would expire after this many days. 0 2 5 6 7 24 30 72

7 As part of the Pregnancy Prevention Programme, prescription for isotretinoin only valid for 7 days (high risk of serious congenital malformations, pregnancy status important).

You receive an NHS prescription for a Schedule 2 controlled drug. The prescription is signed on 6 March 2021. The prescriber has written in the body of the prescription "Supply on 10 April 2021". For each of the questions select the correct answer. What is the last date you can supply an owing against this prescription? 5 March 2021 6 March 2021 7 March 2021 2 April 2021 3 April 2021 4 April 2021 10 April 2021 7 May 2021

7 May 2021. All owing must be dispensed within the validity of the prescription. The appropriate date of a NHS CD prescription is either the signature date or the date indicated by the prescriber as a date before which the medicine should not be supplied, whichever is the later. The 28 day validity of a CD prescription runs from the appropriate date.

You receive an NHS prescription for a Schedule 2 controlled drug. The prescription is signed on 6 March 2021. The prescriber has written in the body of the prescription "Supply on 10 April 2021". For each of the questions select the correct answer. What is the last date you can dispense this prescription? 5 March 2021 6 March 2021 7 March 2021 2 April 2021 3 April 2021 4 April 2021 10 April 2021 7 May 2021

7 May 2021. The appropriate date of a NHS CD prescription is either the signature date or the date indicated by the prescriber as a date before which the medicine should not be supplied, whichever is the later. The 28 day validity of a CD prescription runs from the appropriate date.

Miss J is the new pharmacy summer student at your pharmacy. You are teaching her about the common dosages of various drugs. For the drugs and indications listed, select the most commonly prescribed doses from the list below. Alendronic acid for postmenopausal osteoporosis. 400 micrograms once daily 5 mg once in the morning 10 mg once a week 7.5 mg once daily 20 mg once daily 70 mg once a week 500 mg twice a day 1 g three times a day

70 mg once a week

Mrs C has been receiving 180 mg twice daily of MST Continus (morphine sulfate sustained release). Her doctor now wants to change her to morphine sulfate oral solution every 6 hours as she is finding it difficult to swallow tablets. What daily dose of morphine sulfate oral solution will provide an equivalent dose for Mrs C? 40 mg 6 hourly 60 mg 6 hourly 80 mg 6 hourly 90 mg 6 hourly 180 mg 6 hourly

90 mg 6 hourly. 180 mg x 2 = 360 mg in 24 hours. Every 6 hours equates to 24/6 = 4 times daily. Therefore 360 mg / 4 = 90 mg 6 hourly.

What volume of normal saline 0.9% should be added to 1 mL of insulin (Humulin S) to give a final concentration of 50 units per 50 mL? 4.9 mL 490 mL 9.9 mL 99 mL 990 mL

99 mL. There is 100 units per mL of insulin (this is standard). 100 units in 1 mL, add 49 mL gives 100 units in 50 mL, so another 50 mL is needed to make up to 100 mL. 100 units in 100 mL is 50 units per 50 mL, so the answer is 49 + 50 = 99 mL.

You are creating a local prescribing guidance document on hypertension. What is the recommended target blood pressure for: Hypertension in type 1 diabetes.

<135/85 mmHg In type 1 diabetes, aim for a clinic blood pressure of 135/85 mmHg or less unless the adult with type 1 diabetes has albuminuria or 2 or more features of metabolic syndrome, in which case it should be 130/80 mmHg or less.

You are creating a local prescribing guidance document on hypertension. What is the recommended target blood pressure for: Hypertension in patients with cardiovascular disease and chronic kidney disease.

<135/85 mmHg SIGN (2017) recommend a blood pressure below 135/85 mmHg should be considered in patients with established cardiovascular disease and chronic kidney disease.

You are creating a local prescribing guidance document on hypertension. What is the recommended target blood pressure for: Hypertension in renal disease.

<140/90 mmHg A target clinic blood pressure below 140/90 mmHg is suggested (below 130/80 mmHg is advised in patients with chronic kidney disease and diabetes, or if proteinuria exceeds 1 g in 24 hours).

In which of the following circumstances would two prescription charges normally be payable on a National Health Service Prescription form FP10? A woman 13 weeks pregnant prescribed labetalol and paracetamol tablets. A 45-year-old woman prescribed one pair of Class II thigh - length stockings. A 21-year-old woman prescribed Gygel® and piroxicam capsules. A 57-year-old woman prescribed Elleste Duet 1 mg and diazepam tablets. A 64-year-old diabetic man prescribed Diastix® and metformin tablets.

A 45-year-old woman prescribed one pair of Class II thigh - length stockings.

QRISK is a tool used to assess a patient's cardiovascular disease (CVD) risk. Which of the following patients would be most likely to have their CVD risk assessed using this tool? A 45-year-old man with familial hypercholesterolaemia. A 47-year-old woman who is a smoker. A 55-year-old man who is recovering from a myocardial infarction (MI). A 77-year-old woman with angina. An 85-year-old man with no previous CVD.

A 47-year-old woman who is a smoker A 47-year-old woman who is a smoker is the only suitable option based on patient age and medical conditions. CHD risk assessment tools should not be used to guide management of people with familial hypercholesterolaemia because they are already at a high risk of premature CHD.

Nitrolingual® sublingual spray is available as a Pharmacy (P) medicine in adults over the age of 18 years. For which one of the following patients could an appropriate sale be made? A 20 year old man having an asthma attack. A 19 year old girl suffering from sinusitis. A 56 year old man for the prevention of an angina attack. A 90 year old lady for the treatment of obstructive pulmonary disease. A 36 year old man for the treatment of allergic rhinitis

A 56 year old man for the prevention of an angina attack Nitrolingual® sublingual spray is licensed as a P medicine for the following indication: Prevention and treatment of angina attack.

27 female, takes fluoxetine 20mg capsules OD for depression, she suffers from migraines and so far today has taken 1 sumatriptan 50mg tablet, 4 paracetamol 500mg tabs and 4 tramadol 50mg caps. She has started to feel anxious, nauseous and has tremor, what is the most appropriate action to take? A advise her to not take any more meds and seek prompt medical attention B advise her to not take any more meds and book a routine appointment with GP C advise her to not take any more meds until symptoms resolved then restart fluoxetine D advise her to take max dose paracetamol and tramadol until symptoms resolve E reassure her that these are migraine symptoms so advise her to stay in darkened room until symptoms resolve

A advise her to not take any more meds and seek prompt medical attention

A mother requests a product over the counter for her 3-year-old daughter who is suffering from a cold. She wants to purchase a nasal decongestant containing xylometazoline. What is the maximum length of time the child should use it for? Two days Three days Five days Seven days A child this age must not use it

A child this age must not use it. Xylometazoline should not be used in children under 6 years of age.

Your are the regular pharmacist in a local community pharmacy. A work experience student comes across the term 'audit'. He asks what an audit is. Which statement best defines a clinical audit? - A continuous cycle of learning and development throughout the life of a professional against professional competencies. - Analysis of staff performance indicators. - A systematic review of care (including procedures and services) against explicit standards and best practice. This allows for the improvement of standards of service. - A systematic review of the literature available, summarising it to suit a specified criteria. - A systematic review that identifies all studies and assess the validity of the findings using statistical software

A systematic review of care (including procedures and services) against explicit standards and best practice. This allows for the improvement of standards of service. An audit is defined as: a continuous cycle of quality improvement that seeks to improve patient care and outcomes through systematic review of care (including procedures and services) against explicit standards and best practice, and to use any shortfalls in standards of care to enable improvements to be made.

Abdominal pain in the right lower quadrant is indicative of which of the following conditions? A. Appendicitis B. Duodenal ulcer C. Gall stones D. Gastric ulcer E. IBS

A. Appendicitis.

Which of the following antipsychotics cannot be crushed? A. Chlorpromazine B. Pimozide C. Prochlorperazine D. Sulpiride E. Trifluperizine

A. Chlorpromazine due to risk of contact sensitisation.

For each of the following statements, select the drug that fits most closely. This medication can be used to treat spasticity in multiple sclerosis. Amoxicillin Baclofen Chloramphenicol Chlorpromazine Ciprofloxacin Fludrocortisone Sodium risedronate Tamsulosin hydrochloride

Baclofen Baclofen is indicated for treatment of spasticity in MS.

Latin abbreviations are commonly seen on prescriptions in both hospital and community pharmacies. What does the Latin abbreviation 'a.c.' stand for? Before food Take in the evening On an empty stomach After food With food

Before food. a.c stands for 'ante cibum' or before food.

For each of the following descriptions, select the condition that is most accurate. Joint swelling is typical. Bursitis Chondromalacia Lateral epicondylitis Lower back pain Medial epicondylitis Rotator cuff syndrome Sciatica

Bursitis

For each of the following questions, select the drug which is correct in relation to the scenario. An antibiotic which when given as a multiple daily dose regimen, should have a peak serum concentration NOT exceeding 30 mg/L. Ceftazidime Ertapenem Tetracycline Amikacin Clarithromycin Sodium fusidate Co-trimoxazole Fidaxomicin

Amikacin When given as a multiple daily dose regimen amikacin 'peak' serum concentration should NOT exceed 30 mg/L.

Excessive consumption of which of these is most likely to have caused the signs and symptoms described below. 19-year-old male with signs of ataxia, dysarthria and nystagmus. Methamfetamine Alcohol Sodium oxybate Fluoxetine Atenolol Amitriptyline Paracetamol Chloroquine

Alcohol

For each of the following questions, select the drug which is correct in relation to the scenario. A patient aged 7 years has contracted Kawasaki disease and a doctor asks which drug is most useful to help treat the condition. Aspirin Clopidogrel Dipyridamole Apixaban Danaparoid sodium Dalteparin sodium Unfractionated heparin Dabigatran etexilate

Aspirin Aspirin is NORMALLY contraindicated in children under 16 years of age unless specifically indicated e.g. for Kawasaki disease.

Certain controlled substances such as morphine are subject to much more stringent controls than a normal POM medicine such as simvastatin. Which of the following does NOT have to be recorded when making an entry into the controlled drugs register in respect to the supply of a controlled drug? Date of supply Name of patient Address of prescriber Details of the prescriber's authority to possess Amount supplied

Address of prescriber It is NOT a requirement for prescriber's address to be recorded in the CD register.

For each of the following scenarios, select the drug which is correct. When administered to a 12-year-old patient this drug can precipitate Reye's syndrome which is a potentially fatal condition. Aspirin Apixaban Clopidogrel Dabigatran Dalteparin sodium Danaparoid sodium Dipyridamole Unfractionated heparin

Aspirin Aspirin is contrindicated in under 16s due to the fact it can precipitate Reye's syndrome which may be fatal.

Controlled substances are subject to stringent regulations and laws which must be followed when handling any controlled substance. Which of the following does NOT have to be recorded when making an entry into the controlled drugs register? Address of prescriber Amount supplied Date of supply Name of patient Prescribers identification number/medical council number

Address of prescriber When making an entry into the CD register it is not a requirement to enter the address of the prescriber.

You receive a supply order for diamorphine from a registered midwife. Which one of the following is NOT required to be on the supply order? - Address of the midwife - Name of midwife - Name of patient - Occupation of midwife - Purpose for which the diamorphine is required

Address of the midwife. The address of the midwife is not required on a supply order.

For each of the following statements, select the drug which is correct. This drug should not be given to children under 16 years of age unless specifically indicated for Kawasaki disease. Apixaban Aspirin Atenolol Digoxin Lacidipine Methyldopa Ramipril Urokinase

Aspirin Aspirin should not be given to under 16 years old as there is a risk of Reye's syndrome.

You are a hospital pharmacist who is discussing treatment options for different patients with the healthcare team. Select the single most suitable treatment option for each of the following cases. Mr K is a 61-year-old office worker. He has successfully completed an alcohol detoxification programme but was admitted to hospital after a drink-driving incident. He does not want to go back to the alcohol detoxification programme but admits developing strong cravings for alcohol. Acamprosate Buprenorphine Disulfiram Fluoxetine Haloperidol Paracetamol Parenteral thiamine Pimozide

Acamprosate. Acamprosate is indicated as therapy to maintain abstinence in alcohol-dependent patients. It should be combined with counselling. Disulfiram could be used, however, it acts by causing an acute sensitivity when Mr K drinks alcohol instead of resolving the alcohol cravings.

You are explaining the mechanism of action of different anti-diabetes medicines to pharmacy students. Link the medicine to the mechanism of action. Which medicine delays the digestion and absorption of starch and sucrose? Acarbose Empagliflozin Exenatide Gliclazide Metformin Repaglinide Saxagliptin Short-acting insulin

Acarbose Acarbose delays the digestion and absorption of starch and sucrose.

For each of the following patients, select the most appropriate antidote. A 22-year-old woman who has consumed 8 g of paracetamol over 24 hours. Acetylcysteine Aspirin Digoxin-specific antibody European viper snake venom antiserum Flumazenil Methaemoglobinaemia Naloxone hydrochloride Protamine sulfate

Acetylcysteine The antidote for paracetamol toxicity is acetylcysteine.

Henry Singh is 17 years old and has type 1 diabetes. He is admitted to hospital with DKA. What is used to drive down serum glucose and ketones?

Actrapid soluble insulin

Mrs S, a 70-year-old patient, comes into your pharmacy and tells you that she is in pain. When you inquire further, she tells you that the pain is in her eye. She describes it as 'constant aching pain in her right eye that goes to her forehead'. She also tells you that she 'can not see properly out of that eye'. When you look at her eye you notice that it is red and the pupil is dilated, but there is no discharge and her eye isn't itchy. She usually wears glasses for her hyperopia. Which of the following is the likely cause of her signs and symptoms? Acute angle closure glaucoma. Cluster headache. Conjunctivitis. Migraine. Tension headache.

Acute angle closure glaucoma. Her signs and symptoms are indicative of acute angle closure glaucoma which is more common in elderly patients who have hyperopia (i.e. are long-sighted). Acute angle-closure glaucoma occurs when the outflow of aqueous humour from the eye is obstructed by bowing of the iris against the trabecular meshwork.

Miss S presents in the pharmacy with a productive cough. She tells you that she has been bringing phlegm up when she coughs and it is yellow-green in colour. Given this information what condition is most likely? Acute bacterial infection Acute viral infection Chronic bronchitis Heart failure Pneumonia

Acute viral infection. Sputum colour can be useful in helping to narrow down the differential diagnosis. Viral and bacterial infection can range from sputum with no colour through to yellow, green or brown. Given that viral infection is much more prevalent than bacterial infection then this is the most likely condition in this scenario.

You are working as an independent prescribing pharmacist in a GP surgery running a clinic for patients with asthma. What next step treatment option would you chose for the patient? A patient with controlled asthma who recently had a chest infection. Since then they state they have been using their blue inhaler every day. They are not currently using anything else for their asthma. Add on an inhaled long-acting beta-2 agonist Add on an inhaled corticosteroid Add on leukotriene receptor agonist Add on tiotropium Increase dose of corticosteroid Increase frequency of corticosteroid Maintain current treatment Refer to specialist

Add on an inhaled corticosteroid Low dose ICS should be prescribed for a patient currently on using a reliever inhaler.

You are working as an independent prescribing pharmacist in a GP surgery running a clinic for patients with asthma. What next step treatment option would you chose for the patient? You meet with a patient who is currently prescribed an inhaled corticosteroid (low dose) twice daily and a reliever inhaler. On testing, their lung function remains sub-optimal. Their inhaler technique is appropriate. Add on an inhaled long-acting beta-2 agonist Add on an inhaled corticosteroid Add on leukotriene receptor agonist Add on tiotropium Increase dose of corticosteroid Increase frequency of corticosteroid Maintain current treatment Refer to specialist

Add on an inhaled long-acting beta-2 agonist Initial add-on therapy should include addition of a LABA.

You are working as an independent prescribing pharmacist in a GP surgery running a clinic for patients with asthma. What next step treatment option would you chose for the patient? Your patient is waking at night coughing and using their reliever inhaler after walking to the shops, which she does daily. She is currently using a "blue inhaler, brown inhaler and something else". You check her record and the "something else" is a long-acting beta-2 agonist and her corticosteroid dosing has been optimised. Add on an inhaled long-acting beta-2 agonist Add on an inhaled corticosteroid Add on leukotriene receptor agonist Add on tiotropium Increase dose of corticosteroid Increase frequency of corticosteroid Maintain current treatment Refer to specialist

Add on leukotriene receptor antagonist Addition of a leukotriene receptor agonist should be considered in patients uncontrolled on a SABA, ICS and LABA.

You are training to become an independent prescriber with the aim of specialising in diabetes. Choose the most appropriate next treatment step for each of the patients described. A patient who was first prescribed metformin 3 months ago. The patient has a HbA1c of 7.4%. The patient's record shows they collected a monthly prescription twice in this time. Address adherence Basal insulin DDP-4 inhibitor GLP-1 agonist Metformin SGLT2 inhibitor Soluble insulin Sulphonylurea

Address adherence In a patient who is not taking their medicine, adherence should be addressed before prescribing additional medicines.

A patient presents in your pharmacy and asks you for advice. When he was cleaning his teeth this morning, he noticed bleeding from his gums. Which of the following is NOT a cause of bleeding gums? Asthma Diabetes Poor oral hygeine Pregnancy Smoking

Asthma Diabetes, poor oral hygiene, pregnancy and smoking are all known causes of bleeding gums.

Miss G is a 37-year-old female who has been admitted to hospital following a motorbike accident. She has attained numerous injuries including trauma to her spinal cord but her organ function is normal. Her consultant wishes to initiate her on baclofen. Miss G takes the following drugs: Atenolol 25 mg o.d. Captopril 25 mg b.d. Zonisamide 300 mg o.d. Loperamide 2 mg m.d.u. Which one of the following is the most suitable intervention for Miss G? No intervention is required Advise Miss G's consultant that baclofen enhances the hypotensive effect of captopril and atenolol Advise Miss G's carers that baclofen should be administrated at night Advise Miss G's consultant that baclofen is contra-indicated in epilepsy Advise Miss G's consultant that the use of baclofen with zonisamide is contra-indicated due to an increased risk of Miss G developing hyperthermia.

Advise Miss G's consultant that baclofen enhances the hypotensive effect of captopril and atenolol. Baclofen increases the hypotensive effects of ACE inhibitors and beta blockers.

Miss WQ is a regular patient of your pharmacy and comes to collect her weekly instalment of methadone. She returns, an hour later, asking for another to be dispensed as she dropped the bottle containing methadone. She shows you pieces of the broken bottle. Which of the following is the most appropriate response? - Dispense next week's methadone instalment and contact her prescriber to obtain a replacement prescription to cover the additional supply. - Tell her there's nothing you can do about it. - Get more details about the area where she dropped it and send a member of staff to confirm her story and, if true, supply another bottle. - Advise Miss WQ that she needs to obtain another prescription from her prescriber. - Inform Miss WQ that you must contact the police and prescriber before dispensing any more methadone to her.

Advise Miss WQ that she needs to obtain another prescription from her prescriber. Methadone is a schedule 2 Controlled Drug which is used to treat opioid misusers and can only be legally prescribed and dispensed against a valid instalment prescription. Pharmacists must be aware of the abuse potential of any Controlled Drug.

Mr L is a 3-year-old boy who comes into your pharmacy with his mother. She tells you that she is concerned about his hearing lately. He is otherwise well. Which one of the following actions is the most suitable to take in this situation? Advise Mr L's mother to take him to see his GP Ask Mrs L to request ear syringing for her son at her GP practice Reassure Mr L's mother that this is normal and that she should see her GP if it persists after 2 weeks Reassure Mr L's mother that this is normal and that she should see her GP if it persists after 3 weeks Supply olive oil ear drops

Advise Mr L's mother to take him to see his GP. Mr L may require referral for audiometry. It is difficult to ascertain hearing problems in young children. However, all parental concerns must be taken seriously.

A patient presents in your pharmacy with a prescription. She is breastfeeding her four week old baby and the baby has oral thrush. She has been prescribed miconazole cream for herself but she is unsure what to do with it. What is the best advice to give to the lady? Advise her that this is not the correct treatment for her. Advise her to apply it to her nipples after feeding the baby. Advise her to apply it to her nipples before feeding the baby. Advise her to stop breastfeeding while undergoing treatment. Refer her to her health visitor.

Advise her to apply it to her nipples after feeding the baby. The cream should be applied after feeding the baby.

Mrs G, a 30-year-old female who is 4 months pregnant and lives alone, comes into your pharmacy complaining of persistent perianal pruritis that's been keeping her up all night for the last couple of weeks. She is currently not taking any medicines or has any pre-existing medical conditions but does occasionally take lactulose for pregnancy-induced constipation. Also, one of her children was diagnosed with threadworm a month ago but this was treated successfully. What is the most appropriate advice you should give Mrs G? - Advice Mrs G to wear loose clothes to avoid itching. - Ask Mrs G if she has recently changed her laundry detergent, as her symptoms may be an allergic reaction. - Take a single oral dose of mebendazole, which may need to be repeated in two weeks if re-infection occurs. - Advise her to maintain hygiene measures for a week (e.g. cut finger nails and launder bedding and towels on a daily basis). - Advise her to maintain hygiene measures for six weeks (e.g. cut finger nails and launder bedding and towels on a daily basis)

Advise her to maintain hygiene measures for six weeks (e.g. cut finger nails and launder bedding and towels on a daily basis). It is likely the patient has caught threadworm from her child, but anthelmintics are not recommended during pregnancy and so the patient should be given advice regarding hygiene measure only, which should be followed for six weeks.

A woman presents to your community pharmacy requesting a painkiller, because she just fell over outside and banged her head. There is no visible blood, and she says she feels otherwise fine. She mentions that she takes warfarin. Her last INR test, from a week ago, was 2.9. What would you recommend to the patient? Advise her to take paracetamol tablets Advise her to take ibuprofen tablets Advise her to use an ice pack Advise her to see a doctor immediately Advise her to see her GP in the morning

Advise her to see a doctor immediately. Patients on current anticoagulant therapy such as warfarin are at a higher risk of internal bleeding. They should be advised to seek immediate medical attention if they suffer a significant blow to the head.

A patient presents in your pharmacy with a prescription. She is breastfeeding her four week old baby and the baby has oral thrush. Her and her baby are being treated for the condition. She asks you about expressing milk. What is the best advice to give the lady? Advise her that expressing milk is not advised when treating oral thrush. Advise her to freeze the expressed milk before using it. Advise her to refrigerate the expressed milk before using it. Advise her to use the expressed milk only while she and the baby are undergoing treatment. Refer her to her health visitor

Advise her to use the expressed milk only while she and the baby are undergoing treatment. Expressed milk should be taken and used while the mother and baby are undergoing treatment to prevent re-infection.

You are giving a training session on the effects of alcohol consumption with various drugs. Regarding the concomitant use of alcohol with the following drugs which one is LEAST likely to be correct? Alcohol plus isoniazid increases the risk of isoniazid-related liver disease Alcohol plus phenytoin increases phenytoin metabolism Alcohol plus promethazine increases the risk of sedation Alcohol plus glipizide increases the risk of hyperglycaemia Alcohol plus diazepam decreases a person's motor skills

Alcohol plus glipizide increases the risk of hyperglycaemia. The antidiabetic effect of sulphonylureas is enhanced by alcohol - leading to a risk of hypoglycaemia.

For the following requests for emergency supplies, what would be the most appropriate course of action? A Scottish lady who is asking for dutasteride for her husband who has taken his last tablet this morning. - Advise the patient to see a local prescriber to get a prescription. - Advise that you must speak to the patient themselves before considering further action. - Consider making an emergency supply if it can be clarified that the item has been prescribed by a valid prescriber before . - Contact the patient's prescriber to see if you can get them to send you an NHS prescription so that the patient doesn't have to pay - Get the prescriber to fax you a prescription and use that as a private prescription - Send the patient to A&E to get a prescription - Tell the patient that there is nothing you can do for them - Tell the patient to try the non-NHS pharmacy down the road

Advise that you must speak to the patient themselves before considering further action. The pharmacist must speak to the patient before an emergency supply at the request of a patient is made.

You are working as the responsible pharmacist in a community pharmacy. A patient presents you with a prescription for oxycodone. The patient's address and the prescriber's address is in Geurnsey. What is the best course of action with this prescription? - Advise the patient to see a local prescriber to get a new prescription - Call a local prescriber to ask if they will write a prescription and send it to you - Confirm the prescriber's details and that the prescription meets prescription requirements and dispense - Do not dispense the prescription as it is not a valid prescription in the UK - Speak to the prescriber in Guernsey and make an emergency supply at their request

Advise the patient to see a local prescriber to get a new prescription. Prescriptions for CDs should be written by a prescriber with an address in the UK. The GMC advises that Guernsey is not in the UK. The patient should be advised to see a local prescriber to get a prescription so that they are not left in pain.

For the following requests for emergency supplies, what would be the most appropriate course of action? An American mother on holiday in London requesting a salbutamol inhaler for her 18-year-old son which is usually prescribed by his local doctor. He has forgotten his inhaler but he is not currently having an asthma attack. - Advise the patient to see a local prescriber to get a prescription. - Advise that you must speak to the patient themselves before considering further action. - Consider making an emergency supply if it can be clarified that the item has been prescribed by a valid prescriber before . - Contact the patient's prescriber to see if you can get them to send you an NHS prescription so that the patient doesn't have to pay - Get the prescriber to fax you a prescription and use that as a private prescription - Send the patient to A&E to get a prescription - Tell the patient that there is nothing you can do for them - Tell the patient to try the non-NHS pharmacy down the road

Advise the patient to see a local prescriber to get a prescription. Emergency supplies cannot be made to American citizens. As the patient is at no immediate risk referring them to a local prescriber (rather than A&E) would be appropriate.

For the following requests for emergency supplies, what would be the most appropriate course of action? A UK patient, who is on holiday near your pharmacy, requesting morphine sulfate tablets because she will run out of these tomorrow - Advise the patient to see a local prescriber to get a prescription. - Advise that you must speak to the patient themselves before considering further action. - Consider making an emergency supply if it can be clarified that the item has been prescribed by a valid prescriber before . - Contact the patient's prescriber to see if you can get them to send you an NHS prescription so that the patient doesn't have to pay - Get the prescriber to fax you a prescription and use that as a private prescription - Send the patient to A&E to get a prescription - Tell the patient that there is nothing you can do for them - Tell the patient to try the non-NHS pharmacy down the road

Advise the patient to see a local prescriber to get a prescription. Emergency supplies of schedule 2 CDs cannot be made. The patient would need to see a local prescriber. There is no immediate urgency to this because her pain relief does not run out until the next day.

Which one of the following actions would be the most appropriate for a patient taking sulfasalazine who is experiencing a sore throat with a fever? Advise the patient to seek medical attention as the dose of the medicine may need to be increased. Advise the patient to see their GP as the medicine may need to be withdrawn. Advise the patient to seek medical attention as the dose of the medicine may need to be reduced. Advise the patient that they are experiencing a side effect of the medicine. It is safe to continue taking the medicine, but they may wish to see their GP for an alternative. Advise the patient to stop taking the medicine straight away and seek medical attention.

Advise the patient to stop taking the medicine straight away and seek medical attention BNF states that the aminosalicylates should be stopped immediately if there is suspicion of blood dyscracias. A sore throat could indicate a blood dyscrasia. Patients should be advised to report a sore throat if this occurs whilst they are being treated with this drug. The drug should be stopped and a blood test performed to check for blood dyscrasias.

Latin abbreviations are commonly seen on prescriptions in both hospital and community pharmacies. A pre-registration pharmacist notices a prescription that has the abbreviation 'p.c.' written on it and asks what this means. What is the correct meaning of the Latin abbreviation p.c.? Before food. Take in the evening. On an empty stomach. After food. With food.

After food. p.c stands for 'post cibum' which means after food.

For each of the following patients, select the single most likely cause behind their complaint. Mr K is a 31-year-old gate keeper who becomes very anxious in closed spaces such as an elevator and on the train. Acute stress disorder Agoraphobia Depression Epilepsy Generalised anxiety disorder Panic disorder Post-traumatic stress disorder Social phobia

Agoraphobia Agoraphobia is where a person becomes afraid of places or situations from which escape might be difficult.

Which of these medical terms best describes the medication side effects discussed in the scenarios below? Mrs R describes herself to you as "constantly active, unable to stop moving and feeling restless all the time", as well as having "involuntary movements of her face". You believe this to be side effects of fluphenazine decanoate she is currently using for schizophrenia. Hypertrichosis and tenesmus Hypertrichosis and tinnitus Hypertriglyceridemia and tympanosclerosis Akathisia and tardive dyskinesia Hyperhydrosis and tinnitus Dystonia and dyskinesia Akathisia and dystonia Akathisia and myoclonic jerks

Akathisia and tardive dyskinesia

For each of the following statements, select the drug that fits most closely. Osteonecrosis of the jaw is a risk in patients who receive treatment with this drug. Alendronate Disulfiram Isoniazid Isotretinoin Ketoconazole Lithium Rifampicin Sulfasalazine

Alendronate. Alendronic acid is a bisphosphonate, and as with all bisphosphonates, osteoncecrosis of the jaw is a potential side effect of treatment with this drug.

You are the pharmacist working on an orthopaedic ward. Mr G is a 71-year-old male who has been admitted with an atypical femoral fracture. You review his medical record and he tells you that he has been taking the same medications for over 8 years now. Which one of his following medications are the most likely cause of his fracture? Alendronic acid Aspirin Atenolol Metformin Amlodipine

Alendronic acid Atypical femoral fractures may occur with patients receiving long-term bisphosphonate therapy.

Which drug combination is present in the following products? Fosavance®. Ramipril and amlodipine Tegafur, gimeracil and oteracil Tegafur and gimeracil Calcium and vitamin D3 Alendronic acid and colecalciferol Ramipril and felodipine Oxycodone and naloxone Morphine and naloxone

Alendronic acid and colecalciferol

A GP rings the community pharmacy you work in to ask for advice regarding declining renal function in a patient. Which of the following drugs is contraindicated in renal function below CrCl 35 mLs/min? Alendronic acid Bisoprolol Clarithromycin Denusomab Paracetamol

Alendronic acid. Alendronic acid iscontraindicated in patients with CrCl <35 mLs/min.

When working in your community pharmacy, a patient complains about severe dental pain they have been feeling. Which of the patient's long term regular medications should you flag to the GP as a potential cause of this pain? Alendronic acid Bisoprolol Glyceryl trinitrate Ibuprofen Ranitidine

Alendronic acid. It is a CHM warning that with bisphosphonates (particularly IV) there is an increased risk of osteonecrosis of the jaw.

Chronic obstructive pulmonary disease (COPD) is a progressive obstructive airways disease. Which of the following is NOT true regarding COPD? The main causative factor is smoking. It is less common in women as it is in men. Environmental pollution can be a causative factor. COPD sufferers may also suffer swollen ankles. All smokers will eventually develop COPD.

All smokers will eventually develop COPD. Smoking is the main cause of COPD. Environmental pollution can be a causative factor as can occupational pollution (if you worked in a coal mine for example). COPD sufferers can suffer from oedema leading to swollen ankles. NOT ALL smokers will develop COPD and so it is thought genetics may also play a role in the condition.

For each of the following statements, select the drug that fits most closely. This drug may be prescribed as a monotherapy for the prophylaxis of gout. Allopurinol Azathioprine Azithromycin Cholestyramine Clindamycin Co-beneldopa Colchicine Ethosuximide

Allopurinol Allopurinol is routinely used as monotherapy for gout prophylaxis.

A 58-year-old man has been diagnosed with gout. He had a stroke 3 years ago and suffers from hay fever, but is otherwise well. He is currently taking the following medication: clopidogrel 75 mg once dailysimvastatin 20 mg once dailyamlodipine 20 mg once dailycetirizine 10 mg dailyHe has no known drug allergies. What is the most suitable drug treatment for long-term prophylaxis of his gout? Allopurinol Aspirin Colchicine Febuxostat Riluzole

Allopurinol. Aspirin and riluzole are not indicated in gout. Colchicine is not indicated for long-term prophylaxis. Allopurinol is indicated in the prophylaxis of gout and is a first-line choice. Febuxostat is indicated for gout prophylaxis but NICE recommends only using it in patients who are unable to take allopurinol or who cannot tolerate allopurinol.

Which one of the following is NOT a side-effect from long term topical use of steroid creams? Alopecia Skin atrophy Skin depigmentation Telangiectasia Adrenal suppression

Alopecia. Long term use of steroid creams can cause hypertrichosis which is overgrowth of hair and not alopecia - loss of hair.

For each of the following statements, select the drug which is correct in relation to the scenario. This drug may exacerbate oedema and because of this is cautioned for use in congestive heart failure. Trihexyphenidyl Procyclidine hydrochloride Orphenadrine hydrochloride Tolcapone Amantadine hydrochloride Selegiline hydrochloride Entacapone Rasagiline

Amantadine hydrochloride Amantadine is cautioned in congestive heart failure as it may exacerbate oedema.

What is selegiline metabolised to?

Amfetamine

For each of the following questions, select the drug which is correct in relation to the scenario. You are giving a training session on the treatment on acne vulgaris to some pre-registration trainees. A trainee asks which oral antibiotics are suitable for use in the treatment of acne vulgaris. Ceftazidime Ertapenem Tetracycline Amikacin Clarithromycin Sodium fusidate Co-trimoxazole Fidaxomicin

Amikacin. Amikacin use in obese patients requires the use of 'Ideal weight for height' to calculate dose. Serum amikacin must be closely monitored also.

For each of the following statements, select the drug which is the most applicable. Patients should be carefully evaluated clinically and consideration given to chest X-rays before starting therapy. Amiodarone Clozapine Digoxin Gentamicin Lithium Phenytoin Theophylline Warfarin

Amiodarone Amiodarone can be associated with serious and sometimes fatal lung toxicity. X-ray before starting to establish baseline, after that annually to monitor any changes.

For each of the following patients, select the drug which is contraindicated due to drug disease interactions. A patient with a dysfunctioning thyroid hormone should not be initiated on this medication due to the ability of the medication to accentuate thyroid problems Ursodeoxycholic acid Amiodarone Bendroflumethiazide Pseudoephedrine Piperacillin with tazobactam Levomemprazine Sodium hyaluronate 0.1% eye drops Colestyramine

Amiodarone Amiodarone is contraindicated in patients who suffer with thyroid dysfunction.

For each of the following patients, select the drug which is contraindicated due to drug disease interactions. You are reviewing a patient with sinoatrial block, which drug is contraindicated for use in the patient. Ursodeoxycholic acid Amiodarone Bendroflumethiazide Pseudoephedrine Piperacillin with tazobactam Levomemprazine Sodium hyaluronate 0.1% eye drops Colestyramine

Amiodarone Amiodarone is contraindicated in sinoatrial block.

Which drug combinations may be related to the following scenarios? A patient comes into your pharmacy complaining of muscle weakness and tenderness. Warfarin and fluconazole Amiodarone and simvastatin Aspirin and salbutamol Salbutamol and beclometasone Carbamazepine and erythromycin Simvastatin and St John's Wort Budesonide and formoterol Theophylline and disulfiram

Amiodarone and simvastatin There is an increased risk of myopathy when simvastatin is given with amiodarone.

Miss L is a 29-year-old bank assistant who is reported to have thyroid dysfunction. You review her current medications and you think this effect may be attributed to one of the drugs. Which one of the following drugs is most likely to have caused Miss L's thyroid dysfunction? Aminophylline Amiodarone Glucosamine Loperamide Sodium bicarbonate

Amiodarone. Amiodarone has been associated with causing thyroid dysfunction and may cause hyper- or hypothyroidism.

Excessive consumption of which of these is most likely to have caused the signs and symptoms described below. 25-year-old female with dilated pupils, dry mouth and urinary retention. Methamfetamine Alcohol Sodium oxybate Fluoxetine Atenolol Amitriptyline Paracetamol Chloroquine

Amitriptyline

Which of the drugs below have the following counselling points? This medicine may make you sleepy. If this happens, do not drive or use tools or machines. Do not drink alcohol. Tacrolimus topical Co-careldopa Ketoprofen Cefalexin Detrusitol® Sandocal® Amitriptyline Triamterene

Amitriptyline This is warning label 2 in the BNF. Detrusitol (tolterodine) can also cause drowsiness but this has warning label 3 in the BNF which does not mention alcohol.

Mr R is a 48-year-old male of African family origin. He was previously diagnosed with stage one hypertension. After trying to reduce his blood pressure through changes in diet and physical activity levels, it is still raised. Today he has brought in a prescription for a medication to reduce his blood pressure. He has no other medical conditions. Which one of the following antihypertensives is Mr R most likely to have been prescribed to manage his hypertension? Amlodipine Bisoprolol Candesartan Ramipril Spironolactone

Amlodipine For patients over 55 years, and patients of any age who are of African or Caribbean family origin and do not have diabetes step 1 in the drug management of hypertension is a calcium-channel blocker (CCB). Amlodipine is the only CCB listed in the answer options.

For each of the following patients, select the drug which is most likely to be responsible for the symptoms described. Mr A, a 53-year-old mechanic, reports a strong headache and swelling of his ankles a few days after starting a new drug. Amlodipine Atenolol Bendroflumethiazide Bumetanide Doxazosin Lisinopril Methyldopa Verapamil

Amlodipine The dihydropyridine calcium channel blockers (e.g. nifedipine, amlodipine) cause peripheral vasodilation which leads to swollen ankles, headaches and flushing. Verapamil is another type of calcium channel blocker which is more selective to the heart.

For each of the following scenarios, choose the appropriate treatment for the patient. A 55-year-old male who requires treatment for a cough. The patient is allergic to tetracyclines and macrolides. The patient does not have Covid-19. Amoxicillin Aztreonam Clarithromycin Clindamycin Chloramphenicol Doxycycline Temocillin Vancomycin

Amoxicillin Amoxicillin is an alternative first-line option for adults requiring treatment for a cough.

For each of the following scenarios, choose the appropriate treatment for the patient. A 5-year-old child who has a cough and is unwell. The patient has no known drug allergies and does not have Covid-19. Amoxicillin Aztreonam Clarithromycin Clindamycin Chloramphenicol Doxycycline Temocillin Vancomycin

Amoxicillin Amoxicillin is the first-line treatment for children requiring antibiotics for a cough.

For each of the following scenarios, choose the appropriate treatment for the patient. A 30-year-old woman who is 25 weeks pregnant and requires antibiotic treatment for a cough. The patient has no known drug allergies and does not have Covid-19. Amoxicillin Aztreonam Clarithromycin Clindamycin Chloramphenicol Doxycycline Temocillin Vancomycin

Amoxicillin Doxycycline should not be given to pregnant women; amoxicillin or erythromycin are preferred in women who are pregnant.

For each of the following patients, select the drug which is contraindicated due to drug disease interactions. A patient is admitted to the ward after suffering an acute bronchospasm. Which medication should be AVOIDED in this patient? Aciclovir Atenolol Atorvastatin Bendroflumethiazide Chloramphenicol Flucloxacillin Levomepromazine Paracetamol

Atenolol Atenolol is a beta blocker. This medication should be avoided in patients with history of acute bronchospasm.

Mrs F is a 34-year-old lady who is breast feeding her 6-month-old baby. She is diagnosed with low-severity community acquired pneumonia by her GP. The GP phones you to recommend an antibacterial for Mrs F as she wishes to continue to breast feed whilst on treatment. Which one of the following antibacterials would you recommend for Mrs F? Flucloxacillin Clarithromycin Ciprofloxacin Trimethoprim Amoxicillin

Amoxicillin. Amoxicillin is indicated in low-severity community acquired pneumonia. Amoxicillin is appropriate to use in breastfeeding according to the BNF. Although clarithromycin can be indicated in some instances for low-severity community acquired pneumonia, the BNF states that it should be avoided in breastfeeding.

For the presenting signs and symptoms described below, select the most likely condition from the list. An 11-year-old girl who has trouble breathing immediately after consuming a prawn sandwich. Rheumatoid arthritis Asthma attack Gout Osteoporosis Anaphylaxis Hernia Ectopic pregnancy Irritable bowel syndrome

Anaphylaxis. Common immediate signs of anaphylaxis are trouble breathing, reduced blood pressure and syncope. The following symptoms may also present: hives, swelling, a tingling feeling, itchiness, skin rash, nausea, vomiting, dizziness, diarrhoea and stomach cramps.

Which of the following conditions does NOT cause rectal bleeding? Anal fissure Colorectal carcinoma Diverticular disease Hiatus hernia Inflammatory bowel disease

Hiatus hernia. The other conditions are associated with rectal bleeding.

For each of the following questions, select the drug which is correct in relation to the scenario. A patient is being treated for deep vein thrombosis and is only aware of the dose of the medication they are taking. They inform you they were told to take '10 mg twice daily for 7 days, then a maintenance dose of 5 mg twice daily'. Which medication is this likely to be? Aspirin Clopidogrel Dipyridamole Apixaban Danaparoid sodium Dalteparin sodium Unfractionated heparin Dabigatran etexilate

Apixaban Apixaban is licensed for use in the treatment of DVT at a dosage regime of '10 mg twice daily for 7 days, then a maintenance of 5 mg twice daily'.

For each of the following statements, select the drug which is correct. This drug is a direct inhibitor of activated factor X. Apixaban Aspirin Atenolol Digoxin Lacidipine Methyldopa Ramipril Urokinase

Apixaban Apixaban is one of the newer oral anticoagulants and works on the activated factor X cascade.

Which of the following statements is correct for English prescriptions? - Appliances in section IX A/B/C and part X of the Drug Tariff are the only ones that can be supplied against a FP10 form. - Any appliance may be supplied against a FP10 providing an invoice accompanies the prescription when submitted for pricing. - Any appliance can be prescribed by community nurses on a FP10 form. - All appliances attract a single fee regardless of the number of actual items. - Any appliance can be prescribed by a dentist on a FP10D form

Appliances in section IX A/B/C and part X of the Drug Tariff are the only ones that can be supplied against a FP10 form.

You are working in a community pharmacy on a Saturday. Your counter assistant makes you aware that there is a lady asking for an emergency supply of an EpiPen. On questioning the patient you establish that she is allergic to bees and has been stung. She does not have an EpiPen with her. She reports feeling breathless and having a tight chest. What is the most appropriate initial step in your course of action? Administer an EpiPen to the patient in the pharmacy. Ask the assistant to call the patient's GP for them to request an emergency supply. Ask the assistant to telephone 999, request an ambulance and state anaphylaxis. Start an emergency supply procedure. Take the patient into the consultation room.

Ask the assistant to telephone 999, request an ambulance and state anaphylaxis. The patient is demonstrating signs of anaphylaxis. She will need emergency assistance so it is important to make this call asap. As you have other staff available to do this, it would be appropriate to get them to do this instantly while you then deal with the emergency. The MEP states that: "If a pharmacist administers adrenaline they must also ensure that an ambulance is called by​ dialling 999 and reporting that there is a case​ of suspected anaphylaxis." EpiPen training provides similar guidance. When managing anaphylaxis it is crucial that the ambulance gets to the patient as soon as possible. Therefore, the initial first step to get the assistant to phone 999 would be prudent while you are in the process of locating and administering the adrenaline pen in order to avoid undue delay in the patient getting the next step in their care.

For each of the following scenarios, select the drug which is correct. You are running a training session for a group of pharmacy students and one student wants to know which of the following drugs has anti-pyretic properties. Aspirin Apixaban Clopidogrel Dabigatran Dalteparin sodium Danaparoid sodium Dipyridamole Unfractionated heparin

Aspirin Aspirin can be used for its anti-pyretic properties.

For each of the following questions, select the drug which is correct in relation to the scenario. Which of the listed medications is an NSAID and as such is indicated for analgesia and pyrexia Aspirin Clopidogrel Dipyridamole Apixaban Danaparoid sodium Dalteparin sodium Unfractionated heparin Dabigatran etexilate

Aspirin Aspirin can be used to help prevent blood clots which it does through its anti-platelet activity. It is also indicated for mild-moderate pain and pyrexia.

For each of the following statements, select the drug which is correct. This drug has analgesic, antipyretic and antiplatelet properties. Apixaban Aspirin Atenolol Digoxin Lacidipine Methyldopa Ramipril Urokinase

Aspirin Aspirin causes several different effects in the body, mainly the reduction of inflammation, analgesia, the prevention of clotting and the reduction of fever.

You are working in a busy community pharmacy when a mother of a 14-year-old girl asks to speak to the pharmacist. The mother wants to buy some medication OTC for her daughter who is suffering from a headache, raised temperature, a chesty cough and nasal congestion. Which of the following would NOT be suitable to recommend for this patient? Aspirin 300 mg tablets Guaifenesin 100 mg/5 mL & levomenthol 1.1 mg/5 mL oral solution Ibuprofen 100 mg/5 mL oral suspension Paracetamol 250 mg/5 mL oral suspension Xylometazoline 0.1% nasal spray

Aspirin 300 mg tablets. Aspirin is contraindicated for use in children. Owing to an association with Reye's syndrome aspirin containing preparations should not be given to children under 16. In this instance; Paracetamol can be used for mild-moderate pain and pyrexia. Ibuprofen can be used for fever with discomfort and pain. Guaifenesin and levomenthol solution can be used as an expectorant. Xylometazoline 0.1% nasal spray can be used as a decongestant.

For each question in this section select one answer from the list of eight options above it. This medication must not be sold to patients under the age of 16. It can cause Reye's syndrome and as such is contraindicated in those under the age of 16. Aspirin 75 mg tablets Clotrimazole 1% cream Guaifenesin 100 mg/5 mL & Levomenthol 1.1 mg/5 mL oral solution Loratidine 10 mg tablets Paracetamol 500 mg tablets Pholcodine 5 mg/5 mL linctus Sumatriptan succinate 50 mg tablets Xylometazoline 0.1% nasal spray

Aspirin 75 mg tablets Aspirin and aspirin containing products are contraindicated in under 16s as it may precipitate Reye's syndrome which may be fatal.

Mrs X is a 55-year-old patient. She has recently been suffering from acute migraine attacks which have not responded to paracetamol. She has a history of ischaemic heart disease. Which of the following medications is the most suitable to treat her migraine? Almotriptan Aspirin and metoclopramide Naratriptan Rizatriptan Sumatriptan

Aspirin and metoclopramide. Aspirin and metoclopramide could be used to relieve the symptoms of a migraine: pain and vomiting, respectively. Serotonin 5HT1 agonists (i.e. the 'triptans') are contraindicated in patients with ischaemic heart disease.

You receive a faxed prescription from a dentist for co-codamol 8/500 mg on a Saturday. When the patient enters the pharmacy to collect the medicine the surgery is closed. The patient tells you it is for dental pain and he pays for his prescriptions. What is the most appropriate course of action? - Assess the patient and if appropriate sell OTC co-codamol 8/500 mg - Dispense from the fax - Make an emergency supply at the request of the patient - Make an emergency supply at the request of the prescriber - Send the patient to a new prescriber for a new prescription

Assess the patient and if appropriate sell OTC co-codamol 8/500 mg. Co-codamol 8/500 mg is licensed for dental pain so could be sold following an appropriate assessment. Faxes are not legal prescriptions, it is difficult to make an emergency supply at the request of the patient because you cannot confirm it has been prescribed before (fax is not legal), you cannot speak to the prescriber and sending a patient to another prescriber would potentially take up more appointment time.

Mrs R is a patient who has AIDS. She has been diagnosed with mild pneumocystis pneumonia. She is unable to tolerate trimethoprim. Which single medication is the most likely treatment option for her pneumonia? Atovaquone Co-trimoxazole Hydrocortisone Pentamidine isetionate Prednisolone

Atovaquone. Atovaquone is licensed for the treatment of mild to moderate pneumocystis infection in patients who cannot tolerate co-trimoxazole (co-trimoxazole is made up of sulfamethoxazole and trimethoprim - Note that Mrs R can not tolerate trimethoprim). The other options are suitable for severe pneumocystis infection.

For the patients below, select the most suitable counselling advice you will give. A 19-year-old female who is taking zonisamide for epilepsy. Drowsiness may affect your driving. This effect is enhanced with alcohol consumption. Take this medication at night. Oral solution should be swallowed as a single 100 mL dose with plenty of water while sitting or standing upright. Take this medication on an empty stomach. Avoid factors which cause overheating or dehydration like exercise. Report symptoms including sore throat, mouth ulcers, bruising, fever, tiredness or non-specific illnesses to your doctor immediately. Report immediately if a skin rash develops or if any signs or symptoms of hypersensitivity develops. Take this medication after food

Avoid factors which cause overheating or dehydration like exercise.

Mrs PC comes in to the pharmacy wanting to buy some paracetamol for her 10-year-old daughter, AT, as she has just fallen over and twisted her ankle whilst at school. Mrs PC also asks for a heat pad to ease the pain. Which one of the following is the most appropriate advice? Advise that a heat spray rather than a rub would be better as it has a quicker onset of action. Avoid heat but elevate the foot and use a cold compress. Don't do anything and go to the doctor straight away. Don't do anything and visit the local emergency department as children are more likely to have a fracture. Use heat and paracetamol but also elevate the foot.

Avoid heat but elevate the foot and use a cold compress. RICE is the standard treatment for acute injuries; heat is therefore not appropriate and rules out options A and E. The injury appears to be manageable through self-care and thus options C and D are also incorrect.

You are contacted by a carer for advice on administering medicines to a patient. What is the correct advice for the administration of Mezolar Matrix (fentanyl) 12 mcg/hour transdermal patch? Apply the next patch at the same site. Avoid touching the adhesive side of patches and wash hands after application. Replace every 48 hours. Shave prior to application. Wash the area with soap before applying.

Avoid touching the adhesive side of patches and wash hands after application. The area should be clipped (not shaved) and the area cleaned but soaps avoided.

Miss E is taking MR 250mg Slo-phyllin capsules, she is struggling to swallow the capsule and wants to know if there is another way of taking it, what is the most appropriate advice to give Miss E? A open capsule and disperse granules in full glass of water B sprinkle contents of capsule of yoghurt and eat without chewing C sprinkle contents of capsule on a biscuit and chew thoroughly D capsule must not be opened but can be chewed in mouth E capsule must not be opened and must be swallowed whole

B sprinkle contents of capsule of yoghurt and eat without chewing

A 54-year-old woman whose has a BMI of 34 kg/m2 has asked you for some diet advice as she wishes to lose weight. You explain that she should eat at least 5 portions of fruit and veg each day. She asks for examples of what a portion of fruit and vegetables consists of. What would be the most appropriate answer? A -1 heaped tablespoon of vegetables B -1 pear C -1 teaspoon of dried fruit D -30g of canned fruit E -75 mL of fruit juice

B. A portion is: ● 80 g of fresh, canned or frozen fruit and vegetables ● 30 g of dried fruit ● 150 mL glass of fruit juice Just 1 apple, banana, pear or similar sized fruit is 1 portion. A slice of pineapple or melon is also 1 portion, and 3 heaped tablespoons of vegetables is another portion. Adding a tablespoon of dried fruit, such as raisins, to your morning cereal is an easy way to get 1 portion.

When is menstruation likely to start after stopping norethisterone? A. Same day as stopping. B. 2-3 days after stopping. C. 5-7 days after stopping. D. 1-2 weeks after stopping. E. 4 weeks after stopping.

B. 2-3 days after stopping.

Mr D has been referred to a specialist who has confirmed the diagnosis of PD, he is started on pramipexole at titrating dose, which of the following is the least appropriate advice to give regarding pramipexole? A. BP should be monitored due to risk of postural hypotension B. if a rash occurs the dose should be reduced until rash resolved C. never stop treatment immediately as it carries small risk of neuroleptic malignant syndrome D. sudden onset of sleep is a possible side effect on this med E . treatment is associated with impulse control disorder

B. if a rash occurs the dose should be reduced until rash resolved. Antiparkinsonian drug therapy should never be stopped abruptly as this carries a small risk of neuroleptic malignant syndrome.

A new born baby whose father had tuberculosis two years ago is due to be vaccinated against tuberculosis. Which vaccine is indicated for this patient? BCG vaccine Hepatitis A vaccine Human papilloma virus vaccine Intradermal seasonal influenza vaccine Poliomyelitis vaccine for oral use

BCG vaccine

Mrs WB is a 77-year-old woman, who is admitted to hospital for the treatment of hallucinations. Her past medical history is multiple sclerosis. Her doctor decides to prescribe haloperidol. The next day the ward pharmacist carries out a medicines reconciliation with Mrs WB. Which ONE of the following medicines could be contributing to the reason for Mrs WB's admission to hospital? Paracetamol 1 g QDS started 5 years ago Dantrolene 100 mg four times daily started three months ago Baclofen 10 mg TDS started 3 weeks ago Bendroflumethiazide 2.5 mg OM started 20 years ago Metformin 500 mg TDS started 5 years ago

Baclofen 10 mg TDS started 3 weeks ago

For each of the following patients, select the condition they are likely to be suffering from. A patient presents in the pharmacy asking for your advice about red eyes. Both his eyes are affected and there is a purulent discharge, which made his eyelids stick together this morning. He describes his eyes as feeling 'gritty'. He does not have any eye pain and his vision is not affected. Bacterial conjunctivitis Basel cell carcinoma Blepharitis Corneal ulcer Ectropion Stye Subconjunctival haemorrhage Viral conjunctivitis

Bacterial conjunctivitis Symptoms stated indicative of bacterial conjunctivitis - sticky discharge, both eyes affected, gritty feeling in the eye.

A local doctor asks you about how to prescribe controlled drugs for a patient she has seen privately. To be a valid prescription for a schedule 2 CD from a doctor who has seen a patient privately, how must the prescription be presented? Be computer generated Be hand written Be prescribed on the NHS only Be on an approved form Be on letter-headed paper

Be on an approved form. Private prescriptions for schedule 2 and 3 CDs must be on an approved form.

A patient seeks advice regarding an oral complaint. The patient states they have a sore mouth, which on inspection shows white patches on her tongue. Which of the following medications is the most likely cause of this? Beclometasone Isonazid Itraconazole Pamidronate Rifampicin

Beclometasone inhalers can cause oral candida. Counsel patient to rinse mouth out with water after using inhaler.

You are visited by a representative from Apharma who make a drug called Nufen, a drug for pain. The representative tells you that if you can increase sales on this drug by recommending it, he will donate any profits direct to you so you can use it in the shop for other services. Which of the following professional standards gives guidance on this type of situation? Communicate effectively Behave in a professional manner Demonstrate leadership Maintain, develop and use their professional knowledge and skills Work in partnership with others

Behave in a professional manner

For each of the following patients, select the drug which is most likely to be responsible for the symptoms described. Mrs Y is a 49-year-old house wife. She has type 2 diabetes mellitus and hypertension. Her doctor has recently initiated her on a drug which she suspects is raising her blood glucose levels. Amlodipine Atenolol Bendroflumethiazide Bumetanide Doxazosin Lisinopril Methyldopa Verapamil

Bendroflumethiazide Hyperglycaemia is an adverse effects of thiazide diuretics.

For each of the following patients, select the drug which is most likely to be responsible for the symptoms described. Mr K, a 36-year-old shop assistant, was admitted into hospital after collapsing during a marathon. He was reported to have hypokalaemia, hyponatraemia and he was severely dehydrated. Amlodipine Atenolol Bendroflumethiazide Bumetanide Doxazosin Lisinopril Methyldopa Verapamil

Bendroflumethiazide Thiazide diuretics can cause electrolyte imbalance such as a decline in the levels of potassium, sodium, magnesium and chloride.

For each of the following patients, select the drug which is contraindicated due to drug disease interactions. A patient presents with poor renal function, upon calculating her renal function it is determined that her eGFR is currently 27 mL/minute. Which of the following drugs would be ineffective for use in the patient? Ursodeoxycholic acid Amiodarone Bendroflumethiazide Pseudoephedrine Piperacillin with tazobactam Levomemprazine Sodium hyaluronate 0.1% eye drops Colestyramine

Bendroflumethiazide Thiazides and related diuretics are ineffective if eGFR is less than 30 mL/minute/1.73 m2and should be avoided.

You are working alongside a pre-reg pharmacist and teaching them about hypertension, particularly about the use of bendroflumethiazide for the treatment of high blood pressure. What is the usual dose used for hypertension? Bendroflumethiazide 1 mg twice daily Bendroflumethiazide 2.5 mg once daily Bendroflumethiazide 2.5 mg twice daily Bendroflumethiazide 5 mg once daily Bendroflumethiazide 10 mg once daily

Bendroflumethiazide 2.5 mg once daily A higher dose is rarely necessary.

A patient presents with poor renal function. You calculate her eGFR as 27 mL/minute. Which of the following drugs would be ineffective for use in the patient? Amiodarone Bendroflumethiazide Piperacillin with Tazobactam Pseudoephedrine Ursodeoxycholic Acid

Bendroflumethiazide. Effectiveness of thiazide diuretics is eradicated if eGFR <30 mLs per minute.

A 33-year-old patient comes into the pharmacy with a prescription for a skin complaint. He has erythema around his face in a "butterfly" pattern, with no comedones present. He feels otherwise well and has no other symptoms. Which of the following treatment options is the LEAST suitable for this skin condition? Brimonidine tartrate Tetracycline Camouflaging make-up Erythromycin Benzoyl peroxide

Benzoyl peroxide Benzoyl peroxide is usually used to treat acne vulgaris not rosacea.

For each of the following scenarios, select the drug which is correct in relation to the scenario. A dentist wants to prescribe a medication for a patient suffering with painful inflammatory conditions of the oro-pharynx and asks can you advise accordingly. Benzydamine hydrochloride Dalteparin Duloxetine Isosorbide mononitrate Isotretinoin Pizotifen Timolol 0.25% eye drops Xylometazoline hydrochloride 0.1%

Benzydamine hydrochloride Benzydamine hydrochloride is commonly used in painful conditions affecting the oro-pharnyx and is normally administered as a mouthwash or localised spray.

A doctor contacts you asking you to suggest a drug to treat a patient who has contracted meningococcal disease. Which drug would be suitable to be given via intravenous infusion in this patient? Benzylpenicillin sodium Erythromycin Linezolid Metronidazole Phenoxymethylpenicillin

Benzylpenicillin sodium. Benzylpenicillin sodium is indicated for use in meningococcal disease and is only available in an IV preparation.

Which of the following creams/ointments could stain clothing? Betamethasone valerate/clioquinol cream Daktacort® ointment Fucibet® cream Locoid® ointment Synalar ® cream

Betamethasone valerate/clioquinol cream

Mrs C is complaining of muscle pain after being discharged from hospital following a myocardial infarction. Which ONE of the following medicines is the most likely drug-related cause? Aspirin Omeprazole Bezafibrate Bisoprolol Clopidogrel

Bezafibrate

For each of the following scenarios or statements, select the drug which is correct. This laxative is available as 5 mg gastro-resistant tablets and 10 mg suppositories. Bisacodyl Co-danthramer Docusate sodium Glycerol Ispaghula husk Lactulose Methylcellulose Senna

Bisacodyl Bisacodyl is available as the preparations listed in the question

Which of the following symptoms is most likely to be attributed to ferrous sulfate? Myositis Dry mouth Weight gain Bruising that may be heavy Black stools

Black stools

For each of the following patients, select the condition they are likely to be suffering from. A lady comes into the pharmacy and asks you to look at her eye lids. Both eye lids appear to have skins flakes around the eyelashes. She says that both eyelids feel irritated and her eyes are quite watery. Bacterial conjunctivitis Basel cell carcinoma Blepharitis Corneal ulcer Ectropion Stye Subconjunctival haemorrhage Viral conjunctivitis

Blepharitis Skin flakes around eye lashes accompanied by watery eyes is indicative of blepharitis.

Whilst handing out a patient's regular medication, the patient describes symptoms of digoxin toxicity and generally not feeling well. Which of the following symptoms is most telling of digoxin toxicity? Blurred or yellow vision. Constipation. Haematuria. Hyperhidrosis. Increased thirst.

Blurred or yellow vision. Visual disturbances, including blurred vision and photophobia, may occur. Colour vision may be affected infrequently, with objects appearing yellow.

Mr J is a 54-year-old male who comes to your pharmacy with a prescription for olive oil. You notice that the prescriber has written 'a.u' as the direction for use. Which one of the following is this an abbreviation for? As directed. As required. Both ears. Dilute before use. Apply.

Both ears. a.u from the Latin words auris utrae meaning both ears. Pharmacists should be familiar with common Latin abbreviations used in practice.

For each of the following patients described, select the most likely possible consequence of the drug interaction from the list above. A 77-year-old woman is on the following medication: atorvastatin 10 mg daily aspirin 75 mg daily digoxin 62.5 mcg daily. She has been newly prescribed atenolol 10 mg daily. Bleeding risk increased Bradycardia Constipation First dose hypotension Hypertensive crisis Reduced eGFR Serotonin syndrome QT interval prolongation

Bradycardia Beta blockers such as atenolol would increase the risk of digoxin-related bradycardia.

You are working in a community pharmacy and a prescription comes in for a drug you are not familiar with. The patient is waiting and you feel pressured to do this item quickly as it is busy. What should you do? You should dispense it and make a mental note to look it up later. You should start a CPD entry right now. Briefly read the BNF section related to the drug to familiarise yourself with it. Refuse to dispense it. Ask the patient to take it to another pharmacy.

Briefly read the BNF section related to the drug to familiarise yourself with it

For each of the following, select the most appropriate reference source that contains information relating to the topic listed. List of Preparations approved by the Secretary of State for Health which may be prescribed on form FP10(D) by Dentists for National Health Service patients. British National Formulary British Pharmacopoeia Clinicaltrials.gov Electronic Medicines Compendium Martindale - The Complete Drug Reference Medicines, Ethics and Practice - The professional guide for pharmacists National Institute for Health and Care Excellence guidelines Pubmed

British National Formulary Refer to the BNF for the list of approved preparations that can be prescribed on a dental FP10 on the NHS.

You are training your pre-registration pharmacist on food interactions with warfarin. Which of the following foods is most likely to contain a large amount of vitamin K? Artichoke Broccoli Cauliflower Green pepper Peas

Broccoli. Certain foods such as liver, broccoli, brussels sprouts and green leafy vegetables contain large amounts of vitamin K.

Which of the following symptoms is most likely to be attributed to coumarins? Myositis Constipation Weight gain Bruising that may be heavy Black stools

Bruising that may be heavy

The Misuse of Drugs Regulations 2001 (as amended) classify controlled drugs into 5 schedules according to the different levels of control attributed to each. Which schedule includes buprenorphine? Schedule 1 Schedule 2 Schedule 3 Schedule 4 Schedule 5

Buprenorphine is a Schedule 3 CD.

Which of the following drugs may antagonise analgesic effects of other opioids? Buprenorphine Codeine Ibuprofen Morphine Paracetamol

Buprenorphine. Buprenorphine is a partial agonist/partial antagonist and will block, or antagonise, the effects of other opioids.

What should the pre-dose 'trough' concentration be of vancomycin for Mrs D? A 5-10 mg/L B 10-15 mg/L C 15-20 mg/L D 20-25 mg/L E 25-30 mg/L

C 15-20 mg/L

Which of the following should be monitored during Mrs D's vancomycin treatment? A liver function, ECG and blood counts B renal function, liver function and blood counts C renal function, urinalysis and blood counts D thyroid function, liver function and renal function E urinalysis, blood counts and ECG

C renal function, urinalysis and blood counts

RE 25 female going on holiday next month and has calculated she is due to menstruate whilst on holiday. She sees her GP who has prescribed her norethisterone 5mg tablets. The prescription has use as directed and the woman has forgotten how the doctor told her to take the tablets, which is the most appropriate dosage regimen for this patient? Which is the most appropriate dosage regimen for this patient? A. 1 tablet daily B. 1 tablet BD C. 1 tablet TDS D. 2 tablets OD E. 2 tablets BD

C. 1 tablet TDS

Which of the following is the most appropriate time for her to take her norethisterone? A. 1 day before menstruation is due. B. 2 days before menstruation is due. C. 3 days before menstruation is due. D. 4 days before menstruation is due. E. 5 days before menstruation is due.

C. 3 days before menstruation is due.

Mr D's PD has progressed further and he is experiencing unpredictable 'off' periods. Apomorphine HCL is initiated at a specialist clinic. Mr D is taught how to self-administer this medication. Which of the following is the most appropriate way to administer apomorphine HCL? A. IM into inner thigh B. IM into deltoid muscle C. SC into lower abdomen D. SC into deltoid E. SC into inner thigh

C. SC into lower abdomen

You are providing training for a trainee pharmacy technician on controlled drugs. Which of the following is true with regards to cannabis-based products for medicinal use (CBPMS)? - CBPMs supplies can not be held for future use by licensed wholesalers. - CBPMs need to be recorded in the controlled drugs register. - Patients should be advised that the medicine they have been prescribed is licensed. - Private prescriptions do not need to be on a standardised form. - Take one as required is an unacceptable dose instruction for a prescription for CBPMS.

CBPMs need to be recorded in the controlled drugs register

You are updating your pharmacy SOPs with regards to the management and dispensing of gabapentin. Which of the following is NOT a requirement for gabapentin? CD register entry Dose on prescription Drug strength on prescription Drug formulation on prescription Total quantity in words and figures on prescription

CD register entry Pregabalin and gabapentin are exempt from record-keeping requirements.

A doctor asks for advice regarding dyspepsia. A patient is complaining of worsening dyspepsia. Which of the following is NOT associated with exacerbation of symptoms of dyspepsia? Bisphosphonates Calcium supplements Doxycycline Excess alcohol Prednisolone

Calcium supplements. Calcium supplements are not known to exacerbate symptoms of dyspepsia. The rest of these will and should be avoided where possible to reduce worsening dyspepsia.

A patient you estimate to be in her 30s visits you in your pharmacy. She is feeling very unwell. She says she feels dizzy, and you notice that she is pale, sweating and clammy, with rapid, shallow breathing. Her friend reports that "she's just not been right for a while". What would the most appropriate next step be for the pharmacist? Advise the patient to make an appointment with her GP. Allow her to rest in your consultation room. Call 999. Send the patient to A & E. Take her blood pressure

Call 999. The patient is displaying signs of adrenal crisis, which can be fatal if left untreated. 999 should be called.

Which one of the following medicines does NOT stimulate the pancreas to release insulin? Gliclazide Exenatide Canagliflozin Vildagliptin Repaglinide

Canagliflozin. Canagliflozin reversibly inhibits SGLT2 in the kidney to reduce glucose reabsorption and increase urinary glucose excretion.

For each of the following patients with hypertension, select the treatment that is most likely to cause the adverse effect described. Mr X is a 53-year-old patient with hypertension. He was started on an antihypertensive medication and has recently developed a persistent dry cough. Azilsartan medoxomil Bendroflumethiazide Captopril Eprosartan Furosemide Indapamide Spironolactone Telmisartan

Captopril Captopril is an angiotensin-converting enzyme (ACE) inhibitor. It prevents the breakdown of bradykinin, which in turn results in a persistent dry cough.

For each of the following patients with hypertension, select the treatment that is most likely to cause the adverse effect described. Mr A is a 45-year-old patient with hypertension. He has difficulty sleeping. Azilsartan medoxomil Bendroflumethiazide Captopril Eprosartan Furosemide Indapamide Spironolactone Telmisartan

Captopril Insomnia is a side effect of captopril.

Which drug combinations may be related to the following scenarios? A 45-year-old patient admitted on a hospital ward is suffering from dizziness, blurred vision, nausea and vomiting and ataxia. Warfarin and fluconazole Amiodarone and simvastatin Aspirin and salbutamol Salbutamol and beclometasone Carbamazepine and erythromycin Simvastatin and St John's Wort Budesonide and formoterol Theophylline and disulfiram

Carbamazepine and erythromycin Plasma concentration of carbamazepine is increased by erythromycin thus increased risk of carbamazepine toxicity.

A 34-year-old woman has come in with her prescription for carbamazepine 200 mg three times daily. She asks you what the likely reason is for her maintenance dose to be higher than the dose she was started on, as her symptoms have not worsened since the start of treatment. What is the most appropriate explanation to give to this patient? Carbamazepine itself increases how quickly the body processes it so the maintenance dose is higher than the initial dose. Carbamazepine is removed by the kidneys at a high rate and so the dose must be increased over time. Carbamazepine is removed by the liver at a high rate and so the dose must be increased over time. Disease progression is taking place despite no change in symptoms and so the dose has been increased. Drug tolerance may have developed and so the dose has been increased

Carbamazepine itself increases how quickly the body processes it so the maintenance dose is higher than the initial dose. Carbamazepine is an enzyme autoinducer and thus a lower loading dose is required. Carbamazepine induces its own metabolism and so the maintenance dose is higher than the initial doses.

A patient with newly diagnosed type 2 diabetes presents to the pharmacy. He is concerned as his GP mentioned that if his glucose levels are not well controlled he may develop secondary complications of diabetes. Which of the following are secondary complications of type 2 diabetes? Cardiovascular disease Hepatic disease Hyperglycaemia Hypoglycaemia Jaundice

Cardiovascular disease. Cardiovascular disease, diabetic nephropathy and neuropathy are secondary complications of diabetes

You are required to dispense a prescription for sodium valproate to Mrs AM who is 28 years of age. Which counselling point is best practice when dispensing this prescription? Advise the patient to avoid eating food for 30 minutes after taking the medicine. Advise the patient that they will need weekly monitoring of the plasma-drug levels. Advise the patient to take at the same time as food. Check that the patient in enrolled in a Pregnancy Prevention Programme. Tell the patient to ensure she has the same brand dispensed each time.

Check that the patient in enrolled in a Pregnancy Prevention Programme. When dispensing sodium valproate, pharmacists should remind women of child-bearing age of the risks of teratogenicity and provide with a Patient Card every time they are dispensed a valproate medicine - situations can change and a one-time conversation is not sufficient.

You are dispensing a high strength insulin for a patient. What is an appropriate course of action, specific to high strength insulin, to take? Check that the patient can attach a needle to the pen. Check that the patient can hold the pen correctly. Check the patient can read the strength of the medicine on the pen. No additional actions are required. Refer the patient to their diabetes nurse for further advice.

Check the patient can read the strength of the medicine on the pen. Pharmacists should check the patient can read the strength of the medicine on the pen.

For each of the following patients, select the condition they are likely to be suffering from. A child has a rash with very itchy small red spots that started on the head and neck before moving down to the trunk. Some of the spots appear to have crusted over. Before the spots appeared the child had a fever and sore throat. Chicken pox Glandular fever Impetigo Measles Meningitis Molluscum contagiosum Mumps Rubella

Chicken pox Symptoms indicative of chicken pox - particularly the itchy scabs.

For each of the following questions or statements, select the illness which is correct. This disease produces vesicles that contain large quantities of the virus and if these vesicles are scratched or picked then scarring can occur. Atopic eczema Chicken pox (varicella) German measles (rubella) Impetigo Measles (morbilli) Mumps (epidemic parotitis) Slap cheek syndrome (parvovirus) Whooping cough (pertussis)

Chicken pox (varicella) Chicken pox produces vesicles all over the body that carry the virus. This is why patients should be encouraged not to scratch these, which can spread the virus and also cause scarring.

For each of the following patients, select the drug which is contraindicated due to drug disease interactions. A patient is admitted to the ward suffering with acute porphyrias. Which of the following drugs is contraindicated in this patient? Aciclovir Atenolol Atorvastatin Bendroflumethiazide Chloramphenicol Flucloxacillin Levomepromazine Paracetamol

Chloramphenicol The use of chloramphenicol is contraindicated in patients with acute porphyrias.

For each of the following patients, select the drug which is contraindicated due to drug disease interactions. During a ward round, a patient is found to be pregnant. Which drug should NOT be used in pregnant women due to concerns over 'grey baby syndrome'? Aciclovir Atenolol Atorvastatin Bendroflumethiazide Chloramphenicol Flucloxacillin Levomepromazine Paracetamol

Chloramphenicol Whilst safe in pregnancy for topical use, systemic use of chloramphenicol has been linked with grey baby syndrome.

You are working in a community pharmacy and a 29-year-old woman asks for your advice about her teeth. She has started to notice a 'brown staining' on her teeth and wishes to know whether any of her medications could be the cause of this. Which of the following medications is most likely to be the causative factor of the brown staining? Chlorhexidine mouthwash Doxycycline Alendronic acid Omeprazole Methotrexate

Chlorhexidine mouthwash. Brown staining of the teeth frequently follows the use of chlorhexidine mouthwash, spray or gel, but can readily be removed by polishing. Deposition of tetracyclines in growing bone and teeth (by binding to calcium) causes staining and occasionally dental hypoplasia, and they should not be given to children under 12 years but are unlikely to cause staining in adults.

You are the pharmacist on duty in a busy community pharmacy. You are screening the prescriptions and notice that one of them has an incorrect dosage written on it. Which one of the following is most likely the error? Azithromycin 500 mg once daily. Chloroquine 310 mg once daily. Risedronate sodium 5 mg once daily. Aspirin 75 mg once a day. Metronidazole 200 mg three times a day.

Chloroquine 310 mg once daily. Chloroquine 310 mg is given once weekly.

For each of the following statements, select the drug that fits most closely. The use of higher dosages of this drug may cause photosensitisation, and as such, sunlight should be avoided. Amoxicillin Baclofen Chloramphenicol Chlorpromazine Ciprofloxacin Fludrocortisone Sodium risedronate Tamsulosin hydrochloride

Chlorpromazine Chlorpromazine may cause photosensitisation. Although ciprofloxacin can cause photosensitivity, this is not in a dose-related manner. Chlorpromazine causes photosensitivity at higher doses, which is what the question asks about so that is the appropriate answer to this question. Chloramphenicol is not associated with photosensitivity.

For each of the following statements, select the drug that fits most closely. Contact sensitisation is a risk with this drug. Amoxicillin Baclofen Chloramphenicol Chlorpromazine Ciprofloxacin Fludrocortisone Sodium risedronate Tamsulosin hydrochloride

Chlorpromazine Contact sensitisation is a risk with chlorpromazine.

For each of the following statements, select the drug that fits most closely. This drug is known as a bile acid sequestrant. Allopurinol Azathioprine Azithromycin Cholestyramine Clindamycin Co-beneldopa Colchicine Ethosuximide

Cholestyramine Bile acid sequestrants act by binding bile acids, preventing their reabsorption. Cholestyramine is a bile acid sequestrant.

For each of the following statements, select the drug that fits most closely. This drug is used to treat conditions related to bile acid in the body. Allopurinol Azathioprine Azithromycin Cholestyramine Clindamycin Co-beneldopa Colchicine Ethosuximide

Cholestyramine Cholestyramine is a bile acid sequestrant and is used to treat conditions related to bile acid.

A doctor on a cardiology ward is seeking advice regarding a patient's medication history. The doctor is wondering if any of the patient's current medication may be contributing to the QT prolongation the patient is currently presenting with. Which of the following medications may be the causative factor in the QT prolongation? Bendroflumethiazide Chlorphenamine Ciprofloxacin Paracetamol Sodium Valproate

Ciprofloxacin. Quinolones including ciprofloxacin may prolong the QT interval.

Mrs D comes into your pharmacy. She tells you that her 13-year-old child suffers from Leber's Hereditary Optic Neuropathy and asthma. Her urine is red-brown in colour. She usually takes the following medication: - Salbutamol 2 puffs when required - Beclometasone 2 puffs twice daily - Idebenone 300 mg three times daily Which of the following explanations is the most likely to be the cause of the red-brown urine? An interaction between salbutamol and idebenone. Chromaturia is a side effect of beclometasone. Chromaturia is a side effect of idebenone. Menstruation. Urinary tract infection.

Chromaturia is a side effect of idebenone The metabolites of idebenone may cause red-brown discolouration of the urine. This effect is harmless.

For the patients described, select the most likely drug from the list below which may have caused the stated side-effect. Oral candidiasis in a 12-year-old child who is using the inhaler form of this drug. Ciclesonide Ranitidine Sucralfate Docusate sodium Prochlorperazine Felodipine Glyceryl trinitrate Carbocisteine

Ciclesonide. Oral candidiasis can be caused by inhaled corticosteroids, especially if inhaler technique is poor or patients are not rinsing their mouth after using the inhaler.

For each of the following patients, select the most likely drug which results in the adverse effect mentioned. Mr A has swollen gums. Ciclosporin Diclofenac Gold Hydroxychloroquine Methotrexate Penicillamine Prednisolone Sulfasalazine

Ciclosporin Gingival hyperplasia (swollen gums) is a common side effect of ciclosporin.

You are providing a training session on food interactions with medicines. Which of the following is most likely to interact with grapefruit juice? Bisoprolol Ciclosporin Dabigatran Rosuvastatin Warfarin

Ciclosporin Grapefruit juice increases the concentration of ciclosporin. Manufacturer advises avoid.

A patient attends anticoagulation clinic for a routine INR check. The target INR for this patient is 2.5-3.5, however in clinic the INR reading is 4.4. Which of the following medications is most likely causing this deviation in INR range? Carbamazepine Cimetidine Dabrafenib Fosaprepitant Phenobarbital

Cimetidine H2-receptor antagonists increase the anticoagulant effect of warfarin.

For each of the following statements, select the drug that fits most closely. Large amounts of dairy products particularly milk or yoghurt may slow down the uptake of this drug and as such should be taken 1 to 2 hours before taking this medication. Amoxicillin Baclofen Chloramphenicol Chlorpromazine Ciprofloxacin Fludrocortisone Sodium risedronate Tamsulosin hydrochloride

Ciprofloxacin Ciprofloxacin absorption is impacted by dairy produce. As such, consumption of large amounts of dairy produce should be restricted to 1 to 2 hours before taking this medication.

For each of the following statements, select the drug that fits most closely. The CSM has warned that quinolones may induce convulsions in patients with or without a history of convulsions. Amoxicillin Baclofen Chloramphenicol Chlorpromazine Ciprofloxacin Fludrocortisone Sodium risedronate Tamsulosin hydrochloride

Ciprofloxacin Ciprofloxacin is a quinolone. The CSM has warned that quinolones may induce convulsions in patients with or without a history of convulsions; taking NSAIDs at the same time may also induce them.

For each of the following statements, select the drug that fits most closely. Tendon rupture may occur within 48 hours of starting treatment with this medication. Amoxicillin Baclofen Chloramphenicol Chlorpromazine Ciprofloxacin Fludrocortisone Sodium risedronate Tamsulosin hydrochloride

Ciprofloxacin Tendon damage (including rupture) has been reported rarely in patients receiving quinolones. Tendon rupture may occur within 48 hours of starting treatment. Ciprofloxacin is a quinolone.

You receive a phone call from a doctor on the ward who is seeking advice regarding prescribing antibiotics. The doctor is unsure which antibiotic must be used with caution in patients with a history of epilepsy. Which of the following antibiotics must be used with caution in patients with a history of epilepsy? Ciprofloxacin. Co-trimoxazole. Demeclocycline Hydrochloride. Flucloxacillin. Lymecycline.

Ciprofloxacin. Quinolones including ciprofloxacin may induce convulsions in patients with or without a history of convulsions. As such they must be used in caution in patients with a history of epilepsy.

Ola George is 56 years old and has penicillin allergy. Treated for h.pylori with metronidazole and omeprazole. What is the other antibiotic?

Clarithromycin - one week triple therapy regimen.

Which one of the following would you use treat a penicillin-allergic patient with cellulitis near the eyes or nose? Flucloxacillin Phenoxymethylpenicillin Topical fusidic acid Co-amoxiclav Clarithromycin with metronidazole

Clarithromycin with metronidazole. From summary of antibacterial therapy under skin, cellulitis needs to be treated with oral or intravenous first line if infection is near the eyes or nose. Clarithromycin with metronidazole suitable in a patient who is penicillin allergic.

For each of the following patients, select the combination of drugs that would be used for their treatment. A patient, with a penicillin allergy, requires treatment to eradicate a H. pylori infection. Abacavir, lamivudine and lopinavir Amoxicillin, clarithromycin and lansoprazole Clarithromycin, metronidazole and omeprazole Clavulanic acid and ticarcillin Efavirenz, emtricitabine and tenofovir Ethambutol, isoniazid, pyrazinamide and rifampicin. Ethambutol and rifampicin Isoniazid and rifampicin

Clarithromycin, metronidazole and omeprazole Amoxicillin, clarithromycin and lansoprazole or clarithromycin, metronidazole and omeprazole used to treat H.pylori infection, however, patient is allergic to penicillin, therefore this is the correct choice.

A 75-year-old man is taking the following medicines: bisoprolol 2.5 mg once daily clarithromycin 500 mg twice daily lansoprazole 30 mg once daily metformin m/r 500 mg twice daily atorvastatin 20 mg once daily He has type 2 diabetes and had a transient ischaemic attack last year. He is also currently being treated for a mild skin infection. He is due to have knee replacement surgery and will be prescribed dabigatran 150 mg once daily for prophylaxis of venous embolism. Which of his existing medicines should be stopped because dabigatran is started? Bisoprolol Clarithromycin Lansoprazole Metformin Atorvastatin

Clarithromycin. Dabigatran is predicted to increase the risk of bleeding events when given with clarithromycin. As the patient is only being treated for a mild skin infection the best course of action is to stop the clarithromycin whilst undergoing treatment with dabigatran.

Mr K is a 69-year-old retired engineer who takes warfarin and has chronic obstructive pulmonary disease (COPD). He has been admitted into hospital following reports of increased sputum production and a change in sputum colour (clear to green). He was started on 28% oxygen, antibiotics, steroids and regular nebulisers. After 4 days, his INR was reported to be 5.4. Which single drug is most likely to have contributed to his increased INR? Atenolol Clarithromycin Ipratropium Loperamide Salbutamol

Clarithromycin. Warfarin is metabolised via the cytochrome P450 system. Therefore, inhibitors of this system, such as clarithromycin, will potentiate anticoagulation (increase INR).

A doctor wants your advice on what to prescribe a patient with low severity community-acquired pneumonia who has a history of hypersensitivity to penicillin. Which one of the following is the most suitable option for this patient? Cefaclor Cefuroxime Clarithromycin Co-amoxiclav Co-fluampicil

Clarithromycin. Clarithromycin or doxycycline or azithromycin or erythromycin are listed as alternatives to amoxicillin for low severity community-acquired pneumonia. Clarithromycin is the only one of those options listed in the answers.

For each of the following patients, select the combination of drugs that would be used for their treatment. A patient is suffering from a respiratory tract infection caused by beta-lactamase-producing bacteria that are resistant to amoxicillin alone. Abacavir, lamivudine and lopinavir Amoxicillin, clarithromycin and lansoprazole Clarithromycin, metronidazole and omeprazole Clavulanic acid and ticarcillin Efavirenz, emtricitabine and tenofovir Ethambutol, isoniazid, pyrazinamide and rifampicin. Ethambutol and rifampicin Isoniazid and rifampicin

Clavulanic acid and ticarcillin Amoxicillin alone normally used but when bacteria are resistant then it is combined with clavulanic acid allowing it to work more effectively.

Your dispenser is measuring out the daily methadone liquid prescriptions. As she returns the bottle to the cupboard she drops it and it smashes on the floor. What is the most appropriate way to clear up the spillage? Clear up with tissue and denature. Clear up with tissue and store in a bag in the CD cupboard. Clear up with tissue and throw in the bin. Mop up the floor and pour waste water down the drain. Mop up the floor and pour waste water into denaturing kit.

Clear up with tissue and store in a bag in the CD cupboard. Destruction of stock of a CD should be witnessed by an authorised person. This includes spillages. Therefore, the spillage should be cleaned up and kept in the CD cupboard until destruction can be witnessed.

You attend a CPD seminar on the 2001 regulations for the classification of controlled drugs. Which of the following controlled drugs can be classified as a Schedule 4 Part II drug? Diamorphine Diazepam Clenbuterol LSD Zopiclone

Clenbuterol is classified as a Schedule 4 Part II (CD Anab POM).

For each of the following questions, select the drug which is correct in relation to the scenario. Whilst running a training session for pharmacy technicians, you are asked a question relating to the interaction of omeprazole and a certain drug. Omeprazole is said to 'reduce the antiplatelet activity' of this drug if given concurrently. Which drug does this apply to? Aspirin Clopidogrel Dipyridamole Apixaban Danaparoid sodium Dalteparin sodium Unfractionated heparin Dabigatran etexilate

Clopidogrel Both esomeprazole and omeprazole reduce antiplatelet activity of clopidogrel.

For each of the following questions, select the drug which is correct in relation to the scenario. A patient is suffering with a transient ischaemic attack and is intolerant of aspirin and had headaches with Persantin®. A doctor queries which drug is a suitable alternative. Aspirin Clopidogrel Dipyridamole Apixaban Danaparoid sodium Dalteparin sodium Unfractionated heparin Dabigatran etexilate

Clopidogrel Clopidogrel is indicated for use in those patients suffering with a transient ischaemic attack only in patients 'with aspirin hypersensitivity, or those intolerant of aspirin despite the addition of a proton pump inhibitor.' It is given at a dose of 75 mg daily in this condition.

For each of the following scenarios, select the drug which is correct. The antiplatelet effect of this drug is affected by some proton pump inhibitors. Omeprazole is said to reduce the antiplatelet activity of this drug if administered concurrently. Aspirin Apixaban Clopidogrel Dabigatran Dalteparin sodium Danaparoid sodium Dipyridamole Unfractionated heparin

Clopidogrel Omeprazole reduces the antiplatelet effect of clopidogrel and should not be prescribed concurrently. The interaction between clopidogrel and omeprazole is well documented. Any interaction between clopidogrel and dipyridamol is theoretical.

A patient is admitted with a upper GI bleed. Which agent is the most likely cause of the bleed in this patient? Alendronic acid Citalopram Clopidogrel Lansoprazole Ramipril

Clopidogrel has the highest risk of causing a GI bleed of the drugs listed.

Whilst running a training session for pharmacy technicians you are asked a question about the interaction of omeprazole and a certain drug. Omeprazole is said to 'reduce the antiplatelet activity' of this drug if given concurrently. What drug interacts with omeprazole in this way? Apixaban. Aspirin. Clopidogrel. Dalteparin sodium. Dipyridamole.

Clopidogrel. Omeprazole is known to reduce the anti-platelet effect of clopidogrel and as such these drugs should not be co-prescribed.

For each question in this section select one answer from the list of eight options above it. A patient presents to the pharmacy you work at complaining of vulvo-vaginal itching and pain with no discharge. She has previously been treated for this via her GP but is on vacation and wishes to purchase something over the counter. What is the most suitable OTC medication to help alleviate the pain and itching? Aspirin 75 mg tablets Clotrimazole 1% cream Guaifenesin 100 mg/5 mL & Levomenthol 1.1 mg/5 mL oral solution Loratidine 10 mg tablets Paracetamol 500 mg tablets Pholcodine 5 mg/5 mL linctus Sumatriptan succinate 50 mg tablets Xylometazoline 0.1% nasal spray

Clotrimazole 1% cream Clotrimazole 1% cream is indicated for use in vaginal thrush. It is licensed for sale for this condition.

For each question in this section select one answer from the list of eight options above it. A patient walks into the pharmacy in which you work. They wish to discuss symptoms with the pharmacist. Upon consulting with the patient it is apparent the patient has red, scaly and cracked feet. The patient asks if you can recommend some treatment to help resolve the symptoms. Aspirin 75 mg tablets Clotrimazole 1% cream Guaifenesin 100 mg/5 mL & Levomenthol 1.1 mg/5 mL oral solution Loratidine 10 mg tablets Paracetamol 500 mg tablets Pholcodine 5 mg/5 mL linctus Sumatriptan succinate 50 mg tablets Xylometazoline 0.1% nasal spray

Clotrimazole 1% cream These symptoms suggest that the patient has athlete's foot (tinea pedis). Clotrimazole cream can be sold OTC for this indication.

For each of the following statements, select the drug which is the most applicable. The patient should have their white blood cell (WBC) counts and absolute neutrophil count (ANC) checked weekly for the first 18 weeks and at least 4-week intervals thereafter. Amiodarone Clozapine Digoxin Gentamicin Lithium Phenytoin Theophylline Warfarin

Clozapine Neutropenia and potentially fatal agranulocytosis reported with clozapine. Leucocyte and differential blood counts must be normal before starting; monitor counts every week for 18 weeks then at least every 2 weeks and if clozapine continued and blood count stable after 1 year at least every 4 weeks (and 4 weeks after discontinuation); if leucocyte count below 3000 /mm3 or if absolute neutrophil count below 1500 /mm3 discontinue permanently and refer to haematologist.

Whilst working on a hospital acute medicines ward you determine that a patient had a low level of leukocytes in their blood. Which of the following medications may be causing this? Amiloride Clozapine Naloxone Paracetamol Sodium docusate

Clozapine. Leucopenia is a common or very common side effect of clozapine treatment. Leukocytes are white blood cells and leucopenia is a reduction in white blood cells.

For each of the following descriptions, select the condition that is most accurate. Is associated with nasal congestion and excruciating attacks of pain one side of the head. Cluster headache Medication-overuse headache Meningitis Migraine Sinusitis Sub-arachnoid haemorrhage Temporal arteritis Tension-type headache

Cluster headache Accompanying symptoms with headache are useful in helping with a differential diagnosis. Nasal congestion associated with headache is not common and the only conditions listed where this would be seen is in sinusitis and cluster headache. As the pain is described as severe, then this points to cluster headache being the most likely diagnosis.

For each of the following patients, select the most likely option for their presentations. A 23-year-old female has suffered from rapid onset headaches around her right eye every night for the past week. She describes it as excruciating and tells you that she suffers from photophobia in that eye. Cluster headache Epilepsy Meningitis Migraine Rebound headache Sinus headache Tension headache Vertigo

Cluster headache Her signs are typical of a cluster headache: brief headaches around an eye with possible associated symptoms of unilateral photophobia, redness, lid swelling and nasal stuffiness. These are recurrent and could last for a couple of weeks.

Asif is 25 years old. He was bitten by a dog and needs antibacterial prophylaxis. What is given?

Co-amoxiclav. If bitten then cleanse wound thoroughly. Provide vaccine if tetanus prone wound. Rabies prophylaxis should be considered. Assess risk of blood borne viruses e.g. HIV Alternative is doxycycline or metronidazole.

For each of the following statements, select the drug that fits most closely. Indications for this drug include parkinsonism and tremors related to Parkinson's disease. Allopurinol Azathioprine Azithromycin Cholestyramine Clindamycin Co-beneldopa Colchicine Ethosuximide

Co-beneldopa Co-beneldopa is a mixture of benserazide hydrochloride and levodopa. It is indicated for parkinsonism and tremors related to Parkinson's disease.

For each of the following statements, select the drug that fits most closely. This drug contains two active ingredients - levodopa and benserazide. Allopurinol Azathioprine Azithromycin Cholestyramine Clindamycin Co-beneldopa Colchicine Ethosuximide

Co-beneldopa Madopar is also knowns as co-beneldopa. Co-beneldopa contains two active ingredients which are levodopa and benserazide.

For each of the following statements, select the drug that fits most closely. Use of this drug may cause the urine of the patient to turn reddish in colour. Allopurinol Azathioprine Azithromycin Cholestyramine Clindamycin Co-beneldopa Colchicine Ethosuximide

Co-beneldopa Patients who are using co-beneldopa may find that during treatment their urine may turn reddish in colour.

Which of the drugs below have the following counselling points? This medicine may colour the urine dark red. Tacrolimus topical Co-careldopa Ketoprofen Cefalexin Detrusitol® Sandocal® Amitriptyline Triamterene

Co-careldopa Note that triamterene can also colour the urine - it may look slightly blue in some lights.

Which of the following laxative agents is only indicated for constipation in terminally ill patients? Lactulose Liquid paraffin Co-danthramer Sodium docusate Movicol®

Co-danthramer

For each of the following scenarios or statements, select the drug which is correct. You are running a workshop training junior doctors on drugs and side effects. One of the attendees wishes to discuss laxatives and their potential to colour urine and stools. Which laxative can colour the urine red? Bisacodyl Co-danthramer Docusate sodium Glycerol Ispaghula husk Lactulose Methylcellulose Senna

Co-danthramer Co-danthramer is known to colour the urine red. Senna may also colour the urine, but the colour is described as 'yellow or red-brown' rather than red.

For each of the following questions, select the drug which is correct in relation to the scenario. A terminally ill patient is very constipated and a doctor asks you to recommend a suitable laxative. Which one of the following is licensed for use in this situation? Isphaghula husk Methylcellulose Bisacodyl Co-danthramer Docusate sodium Senna Lactulose Glycerol

Co-danthramer Co-danthramer is licensed for use 'only in constipation in terminally patients of all ages'.

A patient aged 46 is requesting a laxative to help reduce the occasional constipation that she suffers with. Which of the following will NOT be suitable for use in this patient? Co-danthramer Isphagula husk Lactulose Senna Sodium docusate

Co-danthramer is only suitable for use palliative care patients.

A terminally ill patient is very constipated and a doctor asks you to recommend a suitable laxative. Which one of the following is licensed only for use in terminally ill patients? Bisacodyl Co-danthramer Docusate sodium Lactulose Senna

Co-danthramer. All of these laxatives may be suitable for use in a patient who is palliative. However, only Co-danthramer is specifically indicated for use in patients who are in palliative care or terminally ill.

For each of the following questions, select the drug which is correct in relation to the scenario. You are running a workshop training healthcare professionals in medicines and some side effects to look out for. One of the attendees questions you on which laxatives can colour the urine. Isphaghula husk Methylcellulose Bisacodyl Co-danthramer Docusate sodium Senna Lactulose Glycerol

Co-danthramer. Co-danthramer may colour urine red.

A patient comes into your pharmacy and is concerned about a change in the colour of their urine. Which is the LEAST likely to have caused this? Co-beneldopa Co-careldopa Co-danthramer Co-danthrusate Co-fluampicil

Co-fluampicil. The other drugs listed can all cause a discolouration of the urine.

A patient who is HIV positive is deemed to be at risk of opportunistic infections and requires prophylactic treatment for Pneumocystis jirovecii pneumonia. Which antibiotic is most appropriate to prescribe for this indication? Amikacin Ceftazidim Co-trimoxazole Ertapenem Tetracycline

Co-trimoxazole Co-trimoxazole is first line therapy for both PCP treatment in HIV patients and also PCP prophylaxis.

For each of the following questions, select the drug which is correct in relation to the scenario. You are working as the on-call pharmacist. The on-call doctor calls to inform you that an in-patient has been diagnosed with Pneumocystis jirovecii pneumonia. The doctor is seeking advice regarding which antibiotic to prescribe. What should you recommend? Ceftazidime Ertapenem Tetracycline Amikacin Clarithromycin Sodium fusidate Co-trimoxazole Fidaxomicin

Co-trimoxazole Co-trimoxazole is the drug of choice in the treatment and prophylaxis ofPneumocystis jiroveciipneumonia.

For each of the following questions, select the drug which is correct in relation to the scenario. A patient who is HIV positive is deemed to be at risk of opportunistic infections and requires prophylactic treatment for Pneumocystis jirovecii pneumonia. Which antibiotic is most appropriate? Ceftazidime Ertapenem Tetracycline Amikacin Clarithromycin Sodium fusidate Co-trimoxazole Fidaxomicin

Co-trimoxazole. Co-trimoxazole is the drug of choice in the treatment and prophylaxis ofPneumocystis jiroveciipneumonia.

For each of the following patients, select the most likely cause for the weight loss described in the scenario. Mrs X is a 40-year-old patient who presents with abdominal pain, foul smelling steatorrhoea, weight loss and dermatitis herpetiformis on her elbows and buttocks. Anorexia nervosa Bulimia nervosa Cancer Coeliac disease Crohn's disease Diabetes mellitus Thyrotoxicosis Tuberculosis

Coeliac disease Her symptoms are indicative of coeliac disease (autoimmune response to gluten, a dietary protein found in various foods such as cereal, bread and pasta). Her symptoms are abdominal pain, foul smelling steatorrhoea (fatty stools due to malabsorption), weight loss and dermatitis herpetiformis (a type of skin rash that is itchy and has blisters that burst when scratched).

For each of the following patients, select the single most likely cause behind their complaint. Master P, a 33-month-old boy, has had poor growth since the age of 9 months. His parents report recurrent abdominal pain with loose stools that are pale and difficult to flush. Coeliac disease Constipation Crohn's disease Diarrhoea Gastroenteritis Lactose intolerance Peptic ulcer disease Pneumonia

Coeliac disease The recurrent abdominal pain, loose and pale stools, together with poor growth, are suggestive of coeliac disease. Further specialist investigations are required for this patient.

The following patient is presenting with diarrhoea. Select the single most likely cause behind this complaint. Mr R, a 14-year-old boy, has type I diabetes mellitus. He is presenting with oily loose stools, abdominal pain and recurrent episodes of hypoglycaemia. He tells you that he enjoys eating different types of breads. Coeliac disease Constipation Crohn's disease Dyspepsia Gastro-oesophageal reflux disease Lactose intolerance Pneumonia Sepsis

Coeliac disease. Coeliac disease is caused by an adverse immune response to gluten, a protein found in wheat, barley and rye. Like type 1 diabetes mellitus, coeliac disease is also an autoimmune condition.

For each of the following statements, select the drug that fits most closely. This drug is used in the treatment of acute gout. Allopurinol Azathioprine Azithromycin Cholestyramine Clindamycin Co-beneldopa Colchicine Ethosuximide

Colchicine Colchicine is used in the treatment of acute gout. Allopurinol is only used in the prophylaxis of gout.

A patient requires calcium supplements. What will facilitate the absorption of calcium? Ascorbic acid Colecalciferol Hydroxocobalamin Pyridoxine Phytomenadione

Colecalciferol

For each of the following patients, select the drug which is contraindicated due to drug disease interactions. You receive a phone call from a GP regarding the suitability of medicines in a patient suffering with complete biliary obstruction. Which one of the listed drugs is contraindicated in a patient suffering with complete biliary obstruction? Ursodeoxycholic acid Amiodarone Bendroflumethiazide Pseudoephedrine Piperacillin with tazobactam Levomemprazine Sodium hyaluronate 0.1% eye drops Colestyramine

Colestyramine Colestyramine is contraindicated in complete biliary obstruction as it NOT likely to be effective.

For each of the following scenarios, select the prescriber which is correct in relation to the scenario. These prescribers are restricted to prescribing dressings, appliances and licensed medicines which are listed in the Nurse Prescribers' Formulary. Community practitioner nurse Dentist registered in the UK Doctor registered in the UK EEA or Swiss doctor or dentist Nurse independent prescriber Optometrist independent prescriber Pharmacist independent prescriber Supplementary prescriber

Community practitioner nurse. Community Practitioner Nurses must only prescribe dressings, appliances and licensed medicines which are listed in the Nurse Prescribers' Formulary.

From the list below select the most suitable counselling points you would give to the following patients. A 67-year-old male who has been prescribed topical alprostadil (prostaglandin E1) for erectile dysfunction. - Immediately contact the hospital emergency department if you experience abnormal heart rhythms. - Do not take this medication with any herbal product - Condoms should be used to avoid local reactions and exposure of this agent to women of childbearing age or those who are pregnant - Apply liberally to scalp - If you experience numbness or tingling of your finger or toes, stop your medication immediately and contact your doctor. - Using a nit comb with this is effective - If you wash your hands reapply treatment - This medication is most effective at night

Condoms should be used to avoid local reactions and exposure of this agent to women of childbearing age or those who are pregnant. Prostaglandin E1 and E2 induce labour and are contraindicated in women of childbearing age, pregnant, or those who are breast feeding.

You are working as the responsible pharmacist in a community pharmacy. A patient presents you with a prescription for lercanidipine. The patient's address and the prescriber's address is in Jersey. What is the best course of action with this prescription? Advise the patient to see a local prescriber to get a new prescription. Call a local prescriber to ask if they will write a prescription and send it to you. Confirm the prescriber's details and that the prescription meets prescription requirements and dispense. Do not dispense the prescription as it is not a valid prescription in the UK. Speak to the prescriber in Jersey and make an emergency supply at their request.

Confirm the prescriber's details and that the prescription meets prescription requirements and dispense Pharmacists may dispense prescriptions from Jersey if the prescription meets legal requirements.

For the following prescriptions, select what would be the most appropriate course of action if presented with one in a community pharmacy. An NHS prescription from Northern Ireland presented for dispensing in England for lisinopril. - Consider dispensing the prescription if the appropriate prescriber checks can be made - Dispense the prescription without making any checks - Dispense the prescription, you can import that product in to the UK - Do NOT dispense the prescription because it is not an NHS prescription - Ask the patient to confirm the indication for the medicine and dispense the UK generic equivalent - Do NOT dispense the prescription, prescriptions from prescribers in this country are not valid in the UK - Do NOT dispense the prescription, the prescriber is not allowed to prescribe whilst in the UK - Do NOT dispense the prescription, the product cannot be supplied in the UK

Consider dispensing the prescription if the appropriate prescriber checks can be made Northern Ireland is part of the UK and as such all prescriptions are valid to be dispensed if they meet the legal requirements.

For the following requests for emergency supplies, what would be the most appropriate course of action? A French patient on holiday requesting citalopram usually prescribed by his local doctor. - Advise the patient to see a local prescriber to get a prescription. - Advise that you must speak to the patient themselves before considering further action. - Consider making an emergency supply if it can be clarified that the item has been prescribed by a valid prescriber before . - Contact the patient's prescriber to see if you can get them to send you an NHS prescription so that the patient doesn't have to pay - Get the prescriber to fax you a prescription and use that as a private prescription - Send the patient to A&E to get a prescription - Tell the patient that there is nothing you can do for them - Tell the patient to try the non-NHS pharmacy down the road

Consider making an emergency supply if it can be clarified that the item has been prescribed by a valid prescriber before. Emergency supplies at the request of an EEA or Swiss patient can be made.

Counselling for medicines often involves advising patients to look out for key side effects. Which of the following side effects of clozapine should be reported due to the risk of fatality? Blurred vision Constipation Headache Nausea Tiredness

Constipation. Clozapine has been associated with varying degrees of impairment of intestinal peristalsis - see Cautions and Contraindications for further information. Patients and their carers should be advised to seek immediate medical advice before taking the next dose of clozapine if constipation develops.

A 24-year-old girl comes into your pharmacy and asks for advice regarding itchy, red, dry skin around her ring finger. She recently started wearing a ring on that finger. There's no broken skin or signs of infection and on further enquiry she tells you that she also suffers from hay fever. What is the likely cause of this patient's concerns? Psoriasis Eczema Contact dermatitis Scabies Insect bite

Contact dermatitis The patient is likely to have contact dermatitis caused by an allergy to the metal in her ring.

A 65-year-old man presents to your pharmacy with the following prescription from his GP: simvastatin 40 mg; amlodipine 10 mg; aspirin 75 mg. What is the most appropriate action? - Dispense the prescription, it is clinically appropriate for this patient. - Refuse to dispense the prescription as it is clinically inappropriate - Contact the GP as there is a possible interaction between amlodipine and simvastatin, whereby the dose of amlodipine needs to be reduced to 5mg od - Contact the GP as there is a possible interaction between amlodipine and simvastatin, whereby the dose of simvastatin needs to be reduced to 20mg od - Contact the GP as there is a possible interaction between amlodipine and simvastatin, whereby the dose of simvastatin needs to be increased to 80mg od

Contact the GP as there is a possible interaction between amlodipine and simvastatin, whereby the dose of simvastatin needs to be reduced to 20mg od. Possible increased risk of myopathy due to interaction between simvastatin and amlodipine which is thought to increase the plasma concentration of simvastatin. Maximum of 20 mg allowable with concomitant amlodipine.

For each of the following scenarios, select the most appropriate action to take. You have received a hand written FP10 prescription from a GP. It is not for a controlled drug. The prescription is missing the quantity. The GP surgery is currently open. What is the best course of action? Ask the doctor's receptionist how many to dispense. Ask the patient to return to the prescriber and get the quantity added. Contact the prescriber for guidance and add a 'PC' endorsement to the prescription. Dispense 5 days worth of treatment and add a 'PNC' endorsement to the prescription . Dispense the largest pack size available. Dispense the smallest pack size available. Dispense 30 days without taking further action. Use your professional judgement to decide how many to dispense.

Contact the prescriber for guidance and add a 'PC' endorsement to the prescription The quantity to dispense is not a legal requirement on this prescription, but appropriate actions should be taken to dispense the correct amount. The prescriber should be contacted where this is possible for further clarification. If the prescriber is not available 5 days worth of treatment should be dispensed.

You have received a hand written FP10 prescription from a GP. It is not for a controlled drug. The doctor has failed to include the strength on the prescription. The product comes in more than one strength. What is your best course of action? - Ask the dispenser what you've got on the shelf and dispense that - Ask the doctor's receptionist what strength to give - Ask the patient to return to the prescriber to get the strength added - Ask the patient what strength they usually have - Contact the prescriber to clarify the strength and add a 'PC' endorsement to the prescription - Dispense the prescription with no additional actions - Dispense the strength available and endorse the prescription accordingly - Use your professional judgement about which strength to dispense

Contact the prescriber to clarify the strength and add a 'PC' endorsement to the prescription. The product strength is not a legal requirement on this prescription, but appropriate actions should be taken to dispense the correct product. Where more than one strength is available the prescriber should be contacted for clarification and the prescription endorsed accordingly.

You discover some out of date morphine sulfate tablets 20 mg in your CD cupboard. What is the most appropriate course of action for destruction of the CD? - Contact your authorised person for destruction to be witnessed - Contact your superintendent pharmacist for destruction to be witnessed - Denature with your technician as a witness and put in CD cupboard - Denature yourself and put in CD cupboard - Denature yourself and put in the doop bin

Contact your authorised person for destruction to be witnessed Destruction of CD stock must be witnessed by an authorised person.

Mrs PJ comes into the pharmacy. She asks your advice on the abdominal discomfort she is experiencing. She says she often suffers with diarrhoea and cramps, and that these symptoms seem to be worse when she feels anxious or stressed. Her Doctor has mentioned it is likely to be IBS. Which one of the following would NOT be an appropriate recommendation? Colpermin IBS Relief (peppermint oil) Loperamide Paracetamol Buscopan Cramps (hyoscine butylbromide) Crampex (colecalciferol, calcium gluconate and nicotinic acid)

Crampex (colecalciferol, calcium gluconate and nicotinic acid). All the other four options are indicated or suitable for the treatment of various symptoms of IBS - Crampex isn't.

Mr T is 55 years old and has come to your pharmacy to collect his first prescription for warfarin. Which one of the following is most likely to increase the anticoagulant effect of warfarin? Beer Cranberry juice Grapefruit juice Green tea Spinach, kale, ginger and apple juice

Cranberry juice. Cranberry juice is an enzyme inhibitor. It slows the metabolism of warfarin, therefore, increasing its anticoagulant effect. The other answers are enzyme inducers and/or high in vitamin k so will reduce the anticoagulant effect of warfarin.

For each of the following patients, select the single most likely cause behind their complaint. Miss G, a 9-year-old girl, is reporting lethargy, poor appetite, weight loss and abdominal pain on her right side for the past 5 months. She has loose stools and her serum C-reactive protein is raised. Coeliac disease Constipation Crohn's disease Diarrhoea Gastroenteritis Lactose intolerance Peptic ulcer disease Pneumonia

Crohn's disease The chronic history, loose stools, weight loss and raised inflammatory markers are suggestive of Crohn's disease. Further specialist investigations are required for this patient.

The following patient is presenting with diarrhoea. Select the single most likely cause behind this complaint. Miss D, a 7-year-old girl, presents with loose stools and abdominal pain. Her parents say that she has lost weight and generally feels 'low'. Her blood results reveal an elevated C-reactive protein and erythrocyte sedimentation rate. Coeliac disease Constipation Crohn's disease Dyspepsia Gastro-oesophageal reflux disease Lactose intolerance Pneumonia Sepsis

Crohn's disease. Diarrhoea, abdominal pain and elevated CPR/ESR are suggestive of Crohn's disease.

Mrs L brings her 6-year-old child into your pharmacy. The child has scabies. The mother would like to purchase a cream, which might stop the child from itching and causing a secondary infection. Which of the following creams could you recommend for non-prescription supply? Crotamiton cream 10% (Eurax®) Titanium dioxide cream (Metanium®) Cod liver oil and zinc oxide cream (Morhulin®) Hydrocortisone cream 1% Clotrimazole 1% cream

Crotamiton cream 10% (Eurax®). Crotamiton cream is recommended for treating scabies infection, other creams are not. B and C are nappy rash creams and hydrocortisone being a steroid is not suitable for children in this setting. Neither is the antifungal agent clotrimazole.

For each of the following children, select the single most likely cause behind their complaint. Mr K, an 11-month-old boy, presents with a hoarse barking cough. He has had a cold for the past 2 days and his body temperature is 37.6°C. Anaphylaxis Bacterial tracheitis Chronic obstructive pulmonary disease Croup Cystic fibrosis Dyspepsia Gastro-oesphageal reflux disease Peptic ulcer

Croup Croup (or laryngotracheobronchitis) is a common condition which affects infants between the ages of 3 months and 6 years. Typically, the symptoms include a barking cough.

Cytotoxic drugs are used in conditions in which there malignancies. Which of the following drugs is cytotoxic? Ciclosporin Co-careldopa Cyclophosphamide Etoricoxib Methadone

Cyclophosphamide

Ms J, a 23-year-old female customer, comes into your pharmacy and complains that she is experiencing some unpleasant symptoms. Over the past couple of days she has found it painful to urinate. She describes it as being a burning, stinging type of pain. She feels like she needs to urinate frequently but is only passing small amounts of urine. She says that her urine is quite dark and smells strongly. Which one of the following is Ms J most likely to be suffering from? Atrophic vaginitis Bacterial vaginosis Cystitis Trichmoniasis Vaginal thrush

Cystitis. Symptoms described are indicative of cystitis. Other conditions are associated with accompanying symptoms e.g. fever, discharge.

The doctor asks your advice on which adverse effects to monitor with vancomycin. Which one of the following is NOT an adverse effect of vancomycin? Nephrotoxicity 'Red man' syndrome High platelet count Low neutrophil count Tinnitus

High platelet count

Mr D has been told that he will need to see his specialist regularly for reviews, after PD diagnosis how often should diagnosis be reviewed? A. 1-2 weeks B. 1-2 months C. 3-6 months D. 6-12 months E. 1-2 years

D. 6-12 months

2 years later, Mr D's PD has advanced and he now takes co-beneldopa 50mg/200mg caps 1 TDS, which is not a common side effect of co-beneldopa? A. abnormal dreams B. depression C. dementia D. hypertension E. taste disturbances

D. hypertension

The Misuse of Medicines Act,1971 (as amended) governs the supply of Controlled Drugs. Which of the following is NOT a legal requirement for a requisition of a controlled drug? Date of birth of the recipient Name of the recipient Purpose of the requisition Signature of the recipient Total quantity of the drug

Date of birth of the recipient. Date of birth of the recipient of a controlled drug requisition is not required.

You receive a requisition for a controlled drug. Which one of the following is NOT a legal requirement for the requisition? Address of recipient. Date of requisition. Occupation. Purpose of the requisition. Signature of the recipient.

Date of requisition. The legal requirements for a controlled requisition are: signature of the recipient, name of recipient, address of recipient, profession/occupation, total quantity of drug and purpose of requisition.

Which of the following requires a dose reduction with concomitant use of verapamil? Warfarin Rivaroxaban Dabigatran Apixaban Phenindione

Dabigatran Verapamil may increase the plasma concentration of dabigatran therefore increasing its effects. For this reason, the manufacturer advises that the dose is adjusted in patients taking verapamil.

For each of the following questions, select the drug which is correct in relation to the scenario. A consultant is seeking to treat a patient for deep vein thrombosis with a direct thrombin inhibitor which has a rapid onset of action and asks for advice in choosing one. Aspirin Clopidogrel Dipyridamole Apixaban Danaparoid sodium Dalteparin sodium Unfractionated heparin Dabigatran etexilate

Dabigatran etexilate Dabigatran etexilate is direct thrombin inhibitor with a rapid onset of action.

Which of the following non-prescription topical preparations is to be used with caution in patients taking phenytoin? HC45® hydrocortisone 1% cream Clotrimazole 2% cream Daktarin® oral gel Zovirax® 5% cream Benzoyl peroxide 4% cream

Daktarin® oral gel. Miconazole enhances anticonvulsant effect of phenytoin (plasma concentration enhanced) although with topical use, the risk is lower.

For each of the following scenarios, select the drug which is correct in relation to the scenario. This medication is routinely prescribed to help prevent the formation of thrombus. Benzydamine hydrochloride Dalteparin Duloxetine Isosorbide mononitrate Isotretinoin Pizotifen Timolol 0.25% eye drops Xylometazoline hydrochloride 0.1%

Dalteparin Dalteparin is commonly used as a prophylactic treatment to prevent thrombus formation in patients who are sedentary such as post-operative patients.

For each of the following scenarios, select the drug which is correct in relation to the scenario. You are working on a cardiology ward as the ward based pharmacist and you receive a phone call from a junior doctor on the ward. The doctor is concerned that the patient is suffering with a subcutaneous haematoma and queries which drug may be the causative factor. Benzydamine hydrochloride Dalteparin Duloxetine Isosorbide mononitrate Isotretinoin Pizotifen Timolol 0.25% eye drops Xylometazoline hydrochloride 0.1%

Dalteparin Dalteparin is known to cause injection site reactions including subcutaenous haematomas.

You are the pharmacist explaining to a pharmacy summer placement student which medicines are not suitable for use in children, including those in their mother's womb. Select the reason why this drug should not be used in children. Oxytetracycline in a lady who is 8 weeks pregnant. Bone marrow toxicity Damage to skeletal development Discolouration of teeth Drug toxicity Reye's syndrome Stunted growth The medicine is teratogenic The risk of using the medicine outweighs the benefits

Damage to skeletal development Effects on skeletal development have been documented in the first trimester in animal studies and tetracyclines should be avoided.

For each of the following questions, select the drug which is correct in relation to the scenario. You receive a phone call from a GP who is seeking to initiate a 32-year-old man on medication for premature ejaculation. The GP explains that this man has had poor control over ejaculations for seven months and this is affecting his relationship, the man is distressed as a result and the GP asks for advice on choosing a suitable drug. Sildenafil Dapoxetine Oxytocin Carbetocin Misoprostol Clindamycin Clotrimazole Estriol

Dapoxetine Dapoxetine is a short acting selective serotonin reuptake inhibitor which is indicated for premature ejaculation in men who meet a set list of criteria.

A patient uses simvastatin 40 mg nocte and presents with a prescription for an antibiotic. Which of the following antibiotics may precipitate rhabdomyolosis if given concomitantly with simvastatin? Amoxicillin Ciprofloxacin Co-amoxiclav Daptomycin Doxycycline

Daptomycin. Statins are predicted to increase the risk of rhabdomyolosis when given with daptomycin.

A patient returns a bag of items to you. Select the most appropriate way to dispose of them. Tramadol capsules 50 mg Denature and dispose of in the designated medicines 'Doop' disposal bin Denature and pour down the sink Down the toilet In the normal waste bin In the wholesaler returns tray Place in the designated medicines disposal 'Doop' bin Place in the sharps bin Put in the confidential waste bin

Denature and dispose of in the designated medicines 'Doop' disposal bin

A 61-year-old man has been prescribed finasteride 5 mg tablets for the treatment of benign prostatic hyperplasia. For which of the following adverse effects should you advise him to immediately stop his finasteride tablets and refer him back to the prescriber? Depression Erectile dysfunction Low libido Pruritis Rash

Depression MHRA/CHM (May 2017) advice regarding reports of depression and suicidal ideation is to stop finasteride immediately and inform a healthcare professional if patient develops depression.

Mr J comes into your pharmacy to ask you about dermatophytic onychomycosis. What part of the body does this condition affect? Genitals Nails Oral cavity Soles of the feet Scalp

Dermatophytic onychomycosis is a fungal infection of the nails.

Select from the list below the most likely method of removing the poisons described below. A 27-year-old female who has ingested an excessive amount of iron in the past 30 minutes. The extract below is from the BNF. Induction of emesis Desferrioxamine administration Repeat administration of activated charcoal Single administration of activated charcoal Alkalinisation of the urine Whole bowel irrigation Haemodialysis Administration of pH neutraliser

Desferrioxamine administration

For each of the following statements or scenarios, select the drug which is correct. This drug is licensed for use in narcolepsy. Dexamfetamine sulfate Lisdexamfetamine mesilate Atomoxetine Methylphenidate hydrochloride Valproic acid Asenapine Lithium carbonate Lithium citrate

Dexamfetamine sulfate. Dexamfetamine sulfate is indicated and licensed for narcolepsy. Methylphenidate is indicated for use in narcolepsy but it is an off-licence use. No methylphenidate product is licensed for use in narcolepsy.

For each of the following statements, select the drug that fits most closely. This drug, when prescribed, should only be for treatment courses lasting for 2 to 4 weeks. This indication includes '....anxiety that is severe or disabling...' Desmopressin Diazepam Flucloxacillin Lamotrogine Misoprostol Paroxetine Salbutamol Vigabatrin

Diazepam Benzodiazepines can be prescribed for an array of conditions, however, they should not be prescribed for any longer than 2 to 4 weeks. This is due to the addictive nature of this drug class.

Different schedules of controlled drugs have varying prescription requirements and legalities. Of the following controlled drugs, which is allowed to be issued via a repeat prescription? For example the prescriber may write on the prescription 'Repeat x 3'? Buprenorphine Diamorphine Diazepam Methadone Oxycodone

Diazepam Diazepam, a schedule 4 CD, is allowed on repeats.

Podiatrists can become independent prescribers (IP) and prescribe a limited range of POMs. Which of the following medicines can a podiatrist IP prescribe for oral administration? Diazepam. Morphine sulfate concentrated oral solution. Morphine sulfate tablets. Oxycodone normal release. Oxycodone prolonged release.

Diazepam. Podiatrists can prescribe diazepam, dihydrocodeine, lorazepam and temazepam only for oral administration.

A 13-year-old child diagnosed with proctitis requires an anti-inflammatory agent. Which diclofenac preparation is the LEAST appropriate for this child? Diclofenac 25 mg suppositories Diclofenac 25 mg tablets Diclofenac 50 mg dispersible tablets Diclofenac 50 mg tablets Diclofenac 75 mg MR tablets

Diclofenac 25 mg suppositories. Diclofenac suppositores (Voltarol) can cause rectal irritation (see contraindications for diclofenac sodium).

For each of the following statements, select the drug which is correct. Hypokalaemia will predispose the patient to toxicity of this drug. Apixaban Aspirin Atenolol Digoxin Lacidipine Methyldopa Ramipril Urokinase

Digoxin Patient's taking digoxin should have their potassium levels monitored, especially when therapy changes which can adversely affect potassium levels.

You are giving a seminar to accredited checking technicians working in your pharmacy on serious drug interactions. Select the most likely drug combination from the list which results in the serious effects described below. This combination causes nausea and vomiting. Digoxin and quinine Digoxin and cefalexin Ketoprofen and paracetamol Sildenafil and isosorbide mononitrate Atenolol and ibuprofen Clonidine and propranolol Pimozide and ketoconazole Metronidazole and cimetidine

Digoxin and quinine The BNF states: plasma concentration of digoxin increased by quinine. Nausea and vomiting may be early signs of an excessive dose of digoxin.

Mr D is a 70-year-old patient who has severe heart failure due to left ventricular systolic dysfunction for which he has been prescribed a medicine. He was recently admitted into A & E for a systemic fungal infection and he was given amphotericin intravenously. He is currently exhibiting signs of nausea, visual disturbances and arrhythmia. Which heart failure medication is most likely to be the cause of the symptoms Mr D is experiencing? Bisoprolol Candesartan Digoxin Nebivolol Valsartan

Digoxin. Digoxin has a narrow therapeutic window. Amphotericin is predicted to increase the risk of digoxin toxicity when given with digoxin. The signs of digoxin toxicity include: confusion, arrhythmias, nausea and visual disturbances. None of the remaining listed drugs interact with amphotericin.

You are the pharmacist explaining to a pharmacy summer placement student which medicines are not suitable for use in children, including those in their mother's womb. Select the reason why this drug should not be used in children. Doxycycline in a mother breastfeeding a 3-month-old child. Bone marrow toxicity Damage to skeletal development Discolouration of teeth Drug toxicity Reye's syndrome Stunted growth The medicine is teratogenic The risk of using the medicine outweighs the benefits

Discolouration of teeth Doxycycline can pass into breastmilk may cause discoloration of teeth.

For each of the following scenarios, select the most appropriate action to take. You have received a hand written FP10 prescription from a GP. It is not for a controlled drug. The prescription is missing the quantity. The GP surgery is currently closed. What is the best course of action? Ask the doctor's receptionist how many to dispense. Ask the patient to return to the prescriber and get the quantity added. Contact the prescriber for guidance and add a 'PC' endorsement to the prescription. Dispense 5 days worth of treatment and add a 'PNC' endorsement to the prescription . Dispense the largest pack size available. Dispense the smallest pack size available. Dispense 30 days without taking further action. Use your professional judgement to decide how many to dispense.

Dispense 5 days worth of treatment and add a 'PNC' endorsement to the prescription The quantity to dispense is not a legal requirement on this prescription, but appropriate actions should be taken to dispense the correct amount. Since the surgery is closed, the prescriber is not available, so 5 days worth of treatment should be dispensed.

You are dispensing a prescription for sodium valproate for Mrs DS who is 26 years old. When dispensing the prescription, you ask if she has signed a risk acknowledgement form, but she says that she hasn't. What should you do? Dispense the prescription, but advise the patient to see her GP as soon as possible. Dispense the prescription without taking any further steps. Do not dispense the prescription and advise the patient to see her GP as soon as possible. Do not dispense the prescription and alert the GP that you have not dispensed it. Do not dispense the prescription as this form should be signed before the prescription can be dispensed.

Dispense the prescription, but advise the patient to see her GP as soon as possible. When dispensing sodium valproate pharmacists should check whether women are enrolled in the Pregnancy Prevention Programme and have signed a Risk Acknowledgement Form - if not, dispense the prescription and advise the patient to speak to her GP as soon as possible (including by contacting the GP directly if necessary) for a specialist referral.

You have received a hand written FP10 prescription from a GP. It is not for a controlled drug. The doctor has failed to include the strength on the prescription. The product comes in one strength. What is your best course of action? - Ask the dispenser what you've got on the shelf and dispense that - Ask the doctor's receptionist what strength to give - Ask the patient to return to the prescriber to get the strength added - Ask the patient what strength they usually have - Contact the prescriber to clarify the strength and add a 'PC' endorsement to the prescription - Dispense the prescription with no additional actions - Dispense the strength available and endorse the prescription accordingly - Use your professional judgement about which strength to dispense

Dispense the strength available and endorse the prescription accordingly. The product strength is not a legal requirement on this prescription, but appropriate actions should be taken to dispense the correct product. Where only one strength is available that can be dispensed and the prescription endorsed accordingly.

In the morning delivery at the pharmacy you receive the following products. What would be the most appropriate way, with regards to legal requirements or Home Office recommendations, to pro cess each of the products in this delivery? Co-codamol 8/500 mg effervescent tablets pack of 32. Display the product behind the counter Immediately place in the CD cupboard Place on the shelves in the dispensary Place the product on the shelves in the shop Record the delivery in the CD register Record the delivery in the prescription-only medicine (POM) register Store in the fridge not visible from the shop Store in the fridge visible from the shop

Display the product behind the counter Co-codamol 8/500 mg is a schedule 5 CD and in a pack size of 32 tablets can be sold to the public if it has a P licence.

For each of the following statements, select the drug that fits most closely. This drug may be used as an adjunct in the treatment of alcohol dependence. Alendronate Disulfiram Isoniazid Isotretinoin Ketoconazole Lithium Rifampicin Sulfasalazine

Disulfiram Disulfiram is used in the treatment of alcohol dependency.

For the following prescriptions, select what would be the most appropriate course of action if presented with one in a community pharmacy. A computer generated and hand signed prescription for salbutamol from a doctor who is registered in Australia, presented by a British patient. - Consider dispensing the prescription if the appropriate prescriber checks can be made - Dispense the prescription without making any checks - Dispense the prescription, you can import that product in to the UK - Do NOT dispense the prescription because it is not an NHS prescription - Ask the patient to confirm the indication for the medicine and dispense the UK generic equivalent - Do NOT dispense the prescription, prescriptions from prescribers in this country are not valid in the UK - Do NOT dispense the prescription, the prescriber is not allowed to prescribe whilst in the UK - Do NOT dispense the prescription, the product cannot be supplied in the UK

Do NOT dispense the prescription, prescriptions from prescribers in this country are not valid in the UK Australian prescribers are not valid prescribers in the UK unless they are also registered with the GMC or another EEA or Swiss regulator.

Miss T is a 22-year-old female who regularly comes to your pharmacy to collect her prescription for fluoxetine. Today she presents you her prescription with a new additional drug, phenelzine. Which one of the following is the best course of action to take? - Dispense both medications and advise Miss T to take phenelzine after food - Dispense both medications and advise Miss T to avoid eating foods rich in tyramine - Dispense both medications and advise Miss T to take both of them on an empty stomach - Dispense both medications and advise Miss T to take them at least 4 hours apart - Do not dispense her medication and contact her prescriber to advise him of the serious adverse effects associated with this combination of drugs.

Do not dispense her medication and contact her prescriber to advise him of the serious adverse effects associated with this combination of drugs. The combination of these two drugs may result in central serotonin syndrome which is characterised by changes in mental status, agitation, diaphoresis, tachycardia and death.

You dispense a prescription for a 70-year-old lady for a Durogesic DTrans (fentanyl) 12 mcg/hour transdermal patch. What is an appropriate counselling point for the patient? Apply the next patch at the same site Do not have a long hot bath whilst wearing the patch Replace every 48 hours Shave prior to application Wash the area with soap before applying

Do not have a long hot bath whilst wearing the patch. Exposure of fentanyl patches to heat sources can increase concentrations of fentanyl which have been associated with serious adverse reactions and death.

For each of the following scenarios, select the prescriber which is correct in relation to the scenario. If prescribing cocaine for the purpose of treating addiction this prescriber requires a Home Office license. Community practitioner nurse Dentist registered in the UK Doctor registered in the UK EEA or Swiss doctor or dentist Nurse independent prescriber Optometrist independent prescriber Pharmacist independent prescriber Supplementary prescriber

Doctor registered in the UK Doctors registered in the UK require a Home Office license to prescribe this controlled drug.

For each of the following questions, select the drug which is correct in relation to the scenario. This medication can be used at a dose of up to 500 mg daily in divided doses in adult patients suffering with chronic constipation. Isphaghula husk Methylcellulose Bisacodyl Co-danthramer Docusate sodium Senna Lactulose Glycerol

Docusate sodium. Docusate sodium can be used up to 500 mg daily in divided doses for those adult patients suffering with chronic constipation.

Cautionary and advisory labels must be clear on all medication you dispense. Which of the following tablets or capsules must contain the advice: 'Take with a full glass of water' on the dispensing label? Dutasteride Dabigatran Doxycycline Ethambutol Imipramine

Doxycycline

For each of the following scenarios, choose the appropriate treatment for the patient. A 25-year-old male who is unwell and has had a cough for 3 weeks. The patient has no known drug allergies and does not have Covid-19. Amoxicillin Aztreonam Clarithromycin Clindamycin Chloramphenicol Doxycycline Temocillin Vancomycin

Doxycycline Doxycycline is the first-line drug in adults requiring antibiotics for an acute cough.

Recommended cautionary and advisory labels are usually added to dispensing labels to inform the patient of key points about their medication. One advisory label tells patients to avoid sunlight and protect skin from exposure to sunlight. Which of the following medications would usually carry the warning to avoid sunlight when taking this medication? Amoxicillin Remifentanyl Flucloxacillin Doxycycline Flurbiprofen

Doxycyline comes with advisory warnings to protect the skin from sunlight or sunlamps even on a cloudy day. Doxycyline makes the skin more susceptible to UV light and as such more susceptible to UV light damage.

For the patients below, select the most suitable counselling advice you will give. A 39-year-old male who is taking baclofen for multiple sclerosis. Drowsiness may affect your driving. This effect is enhanced with alcohol consumption. Take this medication at night. Oral solution should be swallowed as a single 100 mL dose with plenty of water while sitting or standing upright. Take this medication on an empty stomach. Avoid factors which cause overheating or dehydration like exercise. Report symptoms including sore throat, mouth ulcers, bruising, fever, tiredness or non-specific illnesses to your doctor immediately. Report immediately if a skin rash develops or if any signs or symptoms of hypersensitivity develops. Take this medication after food

Drowsiness may affect your driving. This effect is enhanced with alcohol consumption

You a running a workshop educating pharmacy technicians on medications used in asthma. The focus of the workshop is primarily on beta-2 agonists used to treat asthma. Which of the following medications is a beta-2 agonist used to treat asthma? Flupentixol Formoterol fumarate Itraconazole Olodaterol Umeclidinium

Formoterol fumarate Formoterol is used asinhalation treatment in asthma.

You are the pharmacist explaining to a pharmacy summer placement student which medicines are not suitable for use in children, including those in their mother's womb. Select the reason why this drug should not be used in children. Codeine for a broken wrist in a 10-year-old child. Bone marrow toxicity Damage to skeletal development Discolouration of teeth Drug toxicity Reye's syndrome Stunted growth The medicine is teratogenic The risk of using the medicine outweighs the benefits

Drug toxicity Children taking codeine are at risk of morphine toxicity.

For each of the following patients, select one answer from which would it be most important to monitor. A 30-year-old woman has been prescribed gentamicin intravenous injection for the treatment of endocarditis. Alanine transferase C-reactive protein Drug-serum levels Blood glucose International normalised ratio Potassium Red blood cell count Sodium

Drug-serum levels Gentamicin has a narrow therapeutic index, thus drug-serum concentrations must be monitored.

For each of the following patients, select one answer from which would it be most important to monitor. A 45-year-old man prescribed lithium carbonate 400 mg daily. He does not have any other medical conditions and has no known allergies. Alanine transferase C-reactive protein Drug-serum levels Blood glucose International normalised ratio Potassium Red blood cell count Sodium

Drug-serum levels Lithium has a narrow therapeutic index, thus drug-serum concentrations must be monitored.

Mr Singh, a 76-year-old man, asks to speak with the pharmacist. He has a sore right eye. He says that his eye is watering and irritated. The symptoms have been present for quite some time. Upon examination his eye appears normal. Based on this information, what is the most likely diagnosis? Blepharitis Conjunctivitis Dry eye syndrome Entropion Stye

Dry eye syndrome. Entropion, blepharitis and dry eye syndrome can all cause excessive tearing. However, an eye examination would generally show other symptoms of redness and skin flaking in conjunctivitis and blepharitis. Dry eye syndrome seems the most likely diagnosis. Supporting this is that Mr Singh's age also fits with this diagnosis.

A patient is prescribed amitriptyline at night and is worried about side effects. Which side effect is LEAST likely to be caused by amitriptyline? Dry eyes Constipation Dry mouth QT prolongation Drowsiness

Dry eyes Dry eyes is not listed as a side effect of amitriptyline.

For each of the following scenarios, select the drug which is correct in relation to the scenario. A urologist calls the medicines formulary team to discuss stress urinary incontinence and treatment options available in the hospital in which you work. The urologist is unhappy with the current treatment outcomes and wants to prescribe this drug for a patient suffering with stress urinary incontinence. Benzydamine hydrochloride Dalteparin Duloxetine Isosorbide mononitrate Isotretinoin Pizotifen Timolol 0.25% eye drops Xylometazoline hydrochloride 0.1%

Duloxetine Duloxetine is indicated for stress urinary incontinence.

Which of the drugs below requires the following counselling points according to the BNF? Brush teeth for one minute before spitting out. Avoid drinking or rinsing mouth for 30 minutes after use. Rotigotine Alendronic acid Olsalazine sodium Duraphat® Chlorhexidine Pancreatin Felodipine Amiodarone

Duraphat®

RE Miss F, 32 year old female, takes theophylline, which of the following factors will decrease theophylline concentration? A heart failure B hepatic impairment C viral infection D starting smoking E increased alcohol consumption

E increased alcohol consumption

48) 45 female, commenced on tamoxifen 20mg tabs OD for breast cancer treatment, she has presented in pharmacy today asking for a pain killer to help with pain in her left calf, which of the following is the most appropriate action to take? A. recommend a cool pack to apply to affected area when required B. recommend co-codamol 8/500mg tabs C. recommend ibuprofen tabs D. recommend paracetamol tabs E. recommend she see her GP ASAP

E. recommend she see her GP ASAP Patients should be made aware of the symptoms of thromboembolism and advised to report sudden breathlessness and any pain in the calf of one leg.

29 female, takes fludrocortisone tablets at a dose of 50mcg tabs daily and levo 25mcg tabs at a dose of 3 tabs daily. She has realised that for the past week she has confused the directions of her medications and has accidentally been taking 3 fludro tabs daily. Which of the following is the most likely effect she will experience? A. hypercalcaemia B. hyperkalaemia C. hyponatremia D. hypotension E. water retention

E. water retention

The local community nurse phones you to say her patient has run out of medicines. The nurse would like to know which of the patient's medicines she can prescribe for the patient. She is not an independent or supplementary prescriber. Which of the patient's medicines could the nurse prescribe? Atenolol Bendroflumethiazide E45 cream Timodine cream® Morphine sulphate 20 mg tablets

E45 cream. Community nurse prescribers have a restricted list of items that can be prescribed and in this case all the nurse could prescribe is E45 cream.

For each of the following scenarios, select the prescriber which is correct in relation to the scenario. If prescribing controlled drugs, this set of prescribers are only allowed to prescribe Schedule 4 or 5 controlled drugs only. Community practitioner nurse Dentist registered in the UK Doctor registered in the UK EEA or Swiss doctor or dentist Nurse independent prescriber Optometrist independent prescriber Pharmacist independent prescriber Supplementary prescriber

EEA or Swiss doctor or dentist EEA or Swiss doctor or dentist if prescribing scheduled drugs must only prescribe drugs from schedule 4 or 5.

For each of the following scenarios, select the prescriber which is correct in relation to the scenario. This prescriber must prescribe a medicine that is available in the UK. Community practitioner nurse Dentist registered in the UK Doctor registered in the UK EEA or Swiss doctor or dentist Nurse independent prescriber Optometrist independent prescriber Pharmacist independent prescriber Supplementary prescriber

EEA or Swiss doctor or dentist EEA or Swiss doctors or dentists must only prescribe licensed UK medications if seeking to prescribe within the UK.

A 42-year-old woman comes into the pharmacy stating that she has been seeing flashing lights followed by a throbbing headache on her right side. Her symptoms first begun yesterday morning and she reports no change in vision. She has no other medical conditions and is not on any regular medication. Which one of the following is a possible trigger of this woman's symptoms? Deep breathing Doing yoga Drinking water Eating chocolate Meditating

Eating chocolate. The woman is showing symptoms of a migraine with an aura. Common triggers for this include hormonal changes, emotional triggers, fatigue, dietary triggers such as eating cheese, chocolate or drinking alcohol and taking certain medication.

For the presenting signs and symptoms described below, select the most likely condition from the list. A 17-year-old female who experiences lower abdominal pain 8 weeks after having unprotected sex. Rheumatoid arthritis Asthma attack Gout Osteoporosis Anaphylaxis Hernia Ectopic pregnancy Irritable bowel syndrome

Ectopic pregnancy. Lower abdominal pain after unprotected sex maybe indicative of an ectopic pregnancy.

You are a pharmacist working in a GP surgery creating a prescribing guideline for anticoagulant drugs. Which direct acting oral anticoagulant is NOT likely to be recommended by NICE for the prevention of VTE after hip surgery? Apixaban Dabigatran Edoxaban Rivaroxaban Warfarin

Edoxaban. Edoxaban is not recommended by NICE as it is not licensed for that indication. Warfarin is not a DOAC.

For each of the following patients, select the combination of drugs that would be used for their treatment. A patient with HIV infection has been stabilised on antiretroviral therapy for more than 3 months and now needs an add-on therapy. Abacavir, lamivudine and lopinavir Amoxicillin, clarithromycin and lansoprazole Clarithromycin, metronidazole and omeprazole Clavulanic acid and ticarcillin Efavirenz, emtricitabine and tenofovir Ethambutol, isoniazid, pyrazinamide and rifampicin. Ethambutol and rifampicin Isoniazid and rifampicin

Efavirenz, emtricitabine and tenofovir Efavirenz, emtricitabine and tenofovir is a treatment choice in patient with HIV once stabilised on other medication.

For each of the following, select the most appropriate reference source that contains information relating to the topic listed. Up-to-date, approved and regulated patient information for licensed medicines. British National Formulary British Pharmacopoeia Clinicaltrials.gov Electronic Medicines Compendium Martindale - The Complete Drug Reference Medicines, Ethics and Practice - The professional guide for pharmacists National Institute for Health and Care Excellence guidelines Pubmed

Electronic Medicines Compendium. Up-to-date, approved and regulated patient information for licensed medicines, in the form of Summary Product Characteristics and Patient Information Leaflet, can be found on the eMC.

A prescription for levonorgestrol 1500 microgram is received into the dispensary. What is the indication for this prescription? Combined oral contraceptive Dysmenorrhoea Emergency hormonal contraception Follicular stimulating agent Intrauterine contraception

Emergency hormonal contraception. Levonorgestrel at this strength is used for emergency hormonal contraception.

Which of the following can be a side-effect of lisinopril? The SPC is provided here: Hyperglycaemia Hypernatraemia Hyperkalaemia Hypomagnesaemia Hypophosphataemia

Hyperkalaemia

Mrs T comes into the hospital with a potassium level of 7 mmol/L (range 3.5-5.3) and is diagnosed with hyperkalaemia. Which drug is most likely to have caused the hyperkalaemia? Citalopram Enoxaparin Ferrous sulphate Naproxen Salbutamol

Enoxaparin can raise potassium levels due to it inhibiting aldosterone secretion. This is particularly in patients with diabetes mellitus, chronic renal failure, pre-existing metabolic acidosis, taking medicinal products known to increase potassium. Plasma potassium should be monitored regularly especially in patients at risk.

An auditor arrives at your pharmacy to review the electronic pharmacy record. Which of the following actions is INCORRECT? Ensure that any alterations made to the record identify when and by whom the alterations where made. Ensure that the record is available at the registered pharmacy for inspection by the responsible pharmacist, the pharmacy staff and the inspector. Ensure that the record is backed up and there is an audit trail so that in the event of any incident or error it shows who is accountable. Ensure that the record is kept for three years from the day the record was created. Ensure that the responsible pharmacist maintains the pharmacy record.

Ensure that the record is kept for three years from the day the record was created. The electronic pharmacy record must be kept for at least 5 years from the day the record was recreated.

You are working on a Saturday as a locum and you find one error made by the regular pharmacist. The patient has not received the item and has not come to harm from it. It is appropriate to take which of the following actions? Report without delay to the superintendent pharmacist. Report to your immediate supervisor. Report the error to the General Pharmaceutical Council. Ensure the regular pharmacist is aware of their error. Report the pharmacist to the newspaper.

Ensure the regular pharmacist is aware of their error. In practice, you should look at the situation and make your own decisions. In this case you should put yourself in the position of a locum pharmacist (on a Saturday the locum is likely to be the most senior member of staff). You would expect the locum pharmacist to correct the error, fill out any near-miss logs (as it has not reached the patient) and report to the regular pharmacist.

For each of the following statements, select the drug which is correct in relation to the scenario. This drug may colour the urine reddish-brown. Trihexyphenidyl Procyclidine hydrochloride Orphenadrine hydrochloride Tolcapone Amantadine hydrochloride Selegiline hydrochloride Entacapone Rasagiline

Entacapone Entacapone may colour the urine reddish-brown.

You are running a workshop for pre-registration pharmacists regarding controlled drugs and clinical governance. A student has a question regarding the recording of controlled substances in the controlled drug register. Which of the following is NOT true regarding entries into the controlled drug register? A separate register is used for each class of drugs Entries must be in chronological order Entries must be in indelible ink Entries must be made on day of the transaction or within the following five days The register must not be removed from the premises at any time during use

Entries must be made on day of the transaction or within the following five days. Entries into the CD register must be made either on the day of transaction or the FOLLOWING day.

Morphine is a Schedule 2 controlled drug and is subject to strict regulations, including making records in the controlled drug register. Which of the following is NOT true regarding entries in the controlled drug register? The register must not be kept for any other purpose. Entries must be in chronological order. Entries must be made on day of the transaction or within the following five days. The register must not be removed from the premises at any time during use. A separate register is used for each class of drugs.

Entries must be made on day of the transaction or within the following five days. When making an entry into the controlled drugs register the entry must be made on the day of transaction or the FOLLOWING DAY, no later.

A patient with which of the following conditions would most likely be required to notify the DVLA? Asthma Depression Epilepsy Hyperkalaemia Hypertension

Epilepsy. Patients with epilepsy are required to notify the DVLA if they have experienced a seizure. Patients with other conditions may be required to notify the DVLA if they are taking certain medicines.

Which of these medical terms best describes the medication side effect described in the scenarios below? Mrs D administers two sprays of ipratropium bromide into each nostril twice daily for rhinorrhoea associated with allergic rhinitis. She is currently suffering from nose-bleeds secondary to the use of this medication. Hyperhidrosis Haematoma Hyperaesthesia Melaena Haematuria Hypertrichosis Rhinosinusitis Epistaxis

Epistaxis

A mother of a 5-year-old daughter wants some advice. Her daughter has recently developed a red rash on her face. Based solely on this information, which is the most likely condition causing the rash? Atopic dermatitis Erythema infectiosum Pertussis Pityriasis rosea Psoriasis

Erythema infectiosum. Pertussis does not have facial rash; psoriasis can have scalp involvement but lesions away from the hairline are rare; pityriasis rosea is associated with a rash on the body. This leaves atopic dermatitis and erythema infectiosum - both could show facial lesions. Based on probabilities erythema infectiosum is more likely. Its alternative name is 'slapped cheek disease'.

A patient reports finding a piece of plastic in their sachet of medicine. What is the most appropriate first course of action? Complete internal monitoring systems Complete a Yellow Card Establish if the patient has taken the medicine Notify the patient's GP Notify the manufacturer

Establish if the patient has taken the medicine. In a potential medicines safety situation, the most important thing to do is establish if the patient is at any risk.

A patient newly started on tuberculosis treatment comes into your pharmacy with a bowel complaint. What is the most likely cause of flatulence? Ethambutol Isoniazid Pyrazinamide Rifampicin Streptomycin

Ethambutol

For each of the following patients, select the combination of drugs that would be used for their treatment. A patient is admitted to hospital with tuberculosis and requires the combination of drugs used in the initial phase of tuberculosis treatment. Abacavir, lamivudine and lopinavir Amoxicillin, clarithromycin and lansoprazole Clarithromycin, metronidazole and omeprazole Clavulanic acid and ticarcillin Efavirenz, emtricitabine and tenofovir Ethambutol, isoniazid, pyrazinamide and rifampicin. Ethambutol and rifampicin Isoniazid and rifampicin

Ethambutol, isoniazid, pyrazinamide and rifampicin Recommended drug regimen for initial phase of tuberculosis treatment.

For each of the following statements, select the drug that fits most closely. This drug is commonly used in the treatment of seizure-related medical conditions. Allopurinol Azathioprine Azithromycin Cholestyramine Clindamycin Co-beneldopa Colchicine Ethosuximide

Ethosuximide Ethosuximide is indicated in the treatment of absence seizures, atypical absence seizures (adjunct) and myoclonic seizures.

NSAIDs are commonly used for both their analgesic and anti-inflammatory properties. Which of the following NSAIDs has a maximum daily dose of 600 mg? Dexketoprofen Etodolac Etoricoxib Felbinac Ibuprofen

Etodolac As per the BNF totalmaximum daily dose of etodolac is 600 mg.

You are a hospital pharmacist responsible for monitoring the results of patients taking clozaril. How often should the following patient's white blood cell count be monitored? A patient who has been taking clozaril for 9 months. Daily Once weekly Every 2 weeks Every 3 weeks Every 4 weeks Every 8 weeks Every 12 weeks Yearly

Every 2 weeks In the UK, a white cell count with a differential count must be monitored: At least weekly for the first 18 weeks of treatment At least at 2 week intervals between weeks 18 and 52 After 1 year of treatment with stable neutrophil counts, patients may be monitored at least at 4 week intervals Monitoring must continue throughout treatment and for at least 4 weeks after discontinuation

You are a hospital pharmacist responsible for monitoring the results of patients taking clozaril. How often should the following patient's white blood cell count be monitored? A patient who has been taking clozaril for 2 years and has always had stable results. Daily Once weekly Every 2 weeks Every 3 weeks Every 4 weeks Every 8 weeks Every 12 weeks Yearly

Every 4 weeks. In the UK, a white cell count with a differential count must be monitored: At least weekly for the first 18 weeks of treatment At least at 2 week intervals between weeks 18 and 52 After 1 year of treatment with stable neutrophil counts, patients may be monitored at least at 4 week intervals Monitoring must continue throughout treatment and for at least 4 weeks after discontinuation

You have been working on a cytotoxic rotation within your trust and are required to learn the eight side effects of cytotoxic drugs. Which of the following is NOT a side effect of cytotoxic drugs? Alopecia Excessive bone marrow production Extravasation of intravenous drugs Hyperuricaemia Oral mucositis

Excessive bone marrow production is incorrect - cytotoxic drugs can cause bone marrow suppression.

A patient presents to the community pharmacy asking for advice. They are complaining of dyspeptic symptoms. Which of the following symptoms in patients would NOT warrant referral to their GP when presenting along with dyspeptic symptoms? Anorexia Excessive gas Persistent vomiting Referred pain Weight loss

Excessive gas Excessive gas does not warrant referral to the GP. The rest of the symptoms alongside dyspeptic symptoms must be referred to the GP.

You are explaining the mechanism of action of different anti-diabetes medicines to pharmacy students. Link the medicine to the mechanism of action. Which medicine binds to and activates the GLP-1 (glucagon-like peptide-1) receptor to increase insulin secretion, suppress glucagon secretion and slow gastric emptying? Acarbose Empagliflozin Exenatide Gliclazide Metformin Repaglinide Saxagliptin Short-acting insulin

Exenatide. Exenatide binds to and activates the GLP-1 (glucagon-like peptide-1) receptor to increase insulin secretion, suppress glucagon secretion and slow gastric emptying.

For each of the following patients, select the most relevant safety concern associated with the patient's medication. A 64-year-old woman prescribed empagliflozin 10 mg once daily. metformin 500 mg twice daily aspirin 75 mg daily amlodipine 10 mg daily She has been newly prescribed pioglitazone 15 mg once daily. Atypical femoral fractures Benign idiopathic osteonecrosis Fatal diabetic ketoacidosis Increased incidence of heart failure Risk of severe harm and death Subacute cutaneous lupus erythematosus Suicidal ideation Uterine perforation

Fatal diabetic ketoacidosis. MHRA has issued a safety alert for increased risk of diabetic ketoacidosis linked with empagliflozin.

Nora smith has gestational diabetes. Usual first line treatment has not been successful. What is the next treatment option?

First line is metformin Alternative is glibenclanide if metformin is contraindicated. Insulin should be initiated if oral therapy has failed after 1-2 weeks. Should start with either lispro or aspart.

Which ONE of the following antihypertensive drugs would be the first choice for a newly diagnosed 60 year old male Caucasian patient with no other comorbidities? Felodipine M/R 5 mg tablets Bisoprolol 2.5 mg tablets Captopril 25 mg tablets Indapamide 2.5 mg tablets Losartan 25 mg tablets

Felodipine M/R 5 mg tablets Step 1 hypertension therapy in patients over 55 years old is calcium channel blocker, felodipine is the only calcium channel blocker listed in the options above.

A patient has temporarily lost the ability to swallow. The patient takes morphine for pain and the doctor would like to prescribe them a fentanyl patch. The patient currently receives 120 mg of morphine salt daily. What is a suitable dose of the fentanyl patch for this patient? Fentanyl '12' patch Fentanyl '25' patch Fentanyl '50' patch Fentanyl '75' patch Fentanyl '100' patch

Fentanyl '50' patch Fentanyl '50' patch is approximately equivalent to 120 mg daily of morphine salt.

Dermatological conditions are very common and can require specialist input to help control the conditions. A patient walks into the pharmacy you are responsible for and complains that they are suffering from psoriasis. Which of the following is NOT a sign or symptom of plaque psoriasis? Affects mainly knees, hands, elbow and scalp. Fever. Plaque formation. Red scaly lesions. Silver scales on skin.

Fever

Select the best answer from the following question... Psoriasis is a chronic condition affecting the skin and joints. The aim of the treatment is to keep the condition controlled as there is currently no known cure. Which of the following is NOT a characteristic or symptom of plaque psoriasis? Red scaly lesions Silver scales on skin Plaque formation on skin Affects mainly knees, hands, elbow and scalp Fever

Fever. Psoriasis is a chronic condition which has many aggravating factors such as stress, drugs and infections. Plaque psoriasis is characterised by red scaly lesions on the skin, silver scales on the skin, plaque formation on the skin and most commonly affects the knees, hands, elbows and scalp. Fever is NOT a normal sign or symptom of psoriasis (it may indicate a medical emergency such as pustular psoriasis).

Which of the following antibiotics does NOT cause Clostridium difficile infection? Ampicillin Clindamycin Ciprofloxacin Fidaxomicin Cefuroxime

Fidaxomicin

For each of the following questions, select the drug which is correct in relation to the scenario. A patient has been diagnosed with Clostridium difficile infection that is moderate in severity. A hospital consultant asks which is the most appropriate antibiotic to prescribe. Ceftazidime Ertapenem Tetracycline Amikacin Clarithromycin Sodium fusidate Co-trimoxazole Fidaxomicin

Fidaxomicin. Fidoxamicin is indicated for use inC. difficileinfection but cautioned for use in severe or life threatening C.difficile infection.

You are a pre-registration pharmacist in a community pharmacy. You have observed, on more than one occasion, the pharmacist taking codeine tablets from the shelf while working in the pharmacy. What would be the first step in raising a concern about the pharmacist? Alert the pharmacy technician Contact the superintendent pharmacist Find out the organisation's whistleblowing policy Report to the GPhC Speak to the police

Find out the organisation's whistleblowing policy. Pharmacy professionals have a professional responsibility to raise genuine concerns. Pharmacy professionals should find out their employer's policy on raising concerns.

For each of the following patients described, select the most likely possible consequence of the drug interaction from the list above. An 80-year-old man is on the following medication: isosorbide mononitrate 50 mg m/r daily aspirin 75 mg daily amlodipine 10 mg daily He has been newly prescribed lisinopril 10 mg daily. Bleeding risk increased Bradycardia Constipation First dose hypotension Hypertensive crisis Reduced eGFR Serotonin syndrome QT interval prolongation

First dose hypotension. Isosorbide, amlodipine and lisinopril together would potentiate the first dose hypotension.

For each of the following scenarios, select the drug which is correct in relation to the scenario. On the hospital ward you are carrying out a medicines reconciliation for a patient who has been admitted with yellowing of the eyes, dark urine and pale stools. The patient informs you that five weeks ago they completed a 2 week course of an antibiotic which they had never taken before. Which antibiotic is most likely to cause these symptoms? Aztreonam Benzylpenicllin sodium Flucloxacillin Linezolid Metronidazole Nalidixic acid Phenoxymethylpenicillin Tinidazole

Flucloxacillin The symptoms suggest cholestatic jaundice. Flucloxacillin has been reported by the MHRA to cause cholestatic jaundice and this may be delayed by up to two months post treatment.

For each of the following statements, select the drug that fits most closely. This medication is commonly used in the treatment of impetigo. Desmopressin Diazepam Flucloxacillin Lamotrigine Misoprostol Paroxetine Salbutamol Vigabatrin

Flucloxacillin. Flucloxacillin is an antibiotic. It is commonly used in the treatment of impetigo.

A doctor approaches you on the ward to discuss side effects of antibiotics and in particular antibiotics that may affect the functioning of the liver. Which of the following antibiotics may cause cholestatic jaundice to occur, up to two months after treatment with this antibiotic has stopped? Benzylpenicillin sodium Flucloxacillin Linezolid Nalidixic Acid Phenoxymethylpenicillin

Flucloxacillin. MHRA warning stipulates that flucloxacillin may cause cholestatic jaundice at any time during treatment and up to two months post treatment. Patients should be adequately counselled and report any concerns to a healthcare professional.

A patient is initiated on flucloxacillin as follows: '500 mg q.d.s for 14/7'. The patient queries whether there is any lasting side effects and what they should look out for. Which of the following is a significant counselling point that you must inform the patient of as you counsel them on their medication? Flucloxacillin is an antibiotic drug that is used to treat an array of infectious soft tissue. Flucoxacillin may cause issues with liver function and may cause cholesatatic jaundice up to 8 weeks post treatment. Flucloxacillin may in very rare cases cause hair loss. Flucloxacillin must be taken with food. Nerve issues are a side effect of flucloxacillin

Flucoxacillin may cause issues with liver function and may cause cholesatatic jaundice up to 8 weeks post treatment. Flucloxacillin use has been linked to hepatic disorders and patients should be counselled on hepatic risks.

For each of the following statements, select the drug that fits most closely. This drug may be used to treat postural hypotension. Amoxicillin Baclofen Chloramphenicol Chlorpromazine Ciprofloxacin Fludrocortisone Sodium risedronate Tamsulosin hydrochloride

Fludrocortisone Fludrocortisone is indicated for treatment of postural hypotension.

For each of the following patients, select the most appropriate antidote. A 64-year-old man is prescribed alprazolam 500 mcg three times daily. He has consumed 5 mg today. Acetylcysteine Aspirin Digoxin-specific antibody European viper snake venom antiserum Flumazenil Methaemoglobinaemia Naloxone hydrochloride Protamine sulfate

Flumazenil The antidote for benzodiazepine toxicity is flumazenil.

Which of the following drug combinations is known to introduce a risk of rhabdomyolysis? Ibuprofen with paracetamol Digoxin with amiodarone Beclomethasone with salbutamol Fluvastatin with bezafibrate Phenytoin with sodium valproate

Fluvastatin with bezafibrate

For each of the following questions, select the vitamin or mineral which is the correct answer. Used in the prevention of a methotrexate side-effect. Ascorbic acid Calcium Folic acid Iron Retinol Vitamin D Vitamin K Zinc

Folic acid Methotrexate suppresses cells which are metabolically active. One of the ways it does this is to antagonise an enzyme involved in the metabolism of folic acid. Folic acid should be taken on the day or days when the patient does not take methotrexate.

Miss G is a 6-year-old child who presents with the following signs and symptoms: vomiting, malaise and sudden stomach ache which comes and goes. Which one of the following is the most likely cause of these effects? Dyspepsia Encephalitis Hypoxia Migraine Food poisoning

Food poisoning. This patient is presenting with possible signs of food poisoning. Signs and symptoms include; poison in the stool, vomit and urine of the patient, as well as, malaise and repeated attacks of GI irritation.

Steroid creams are routinely sold over the counter. In which of the following situations can hydrocortisone 1% cream be recommended over the counter? In a 9-year-old child with an insect bite on the arm. For contact dermatitis on the neck line of a post-menopausal female. For acne on the back of a 14-year-old male. For the external treatment of chicken pox in an adult. For a patient who has tried it for 7 days and would like to continue for a further 7 days.

For contact dermatitis on the neck line of a post-menopausal female. This question refers to OTC supply - not prescription. It can be sold to treat allergic contact dermatitis, irritant dermatitis, insect bite reactions and mild to moderate eczema. However it may not be sold to children under 10 or in pregnancy except under medical advice. Contraindications include use on eye or face, anogenital region, broken or infected skin, acne or athletes foot. Use is restricted to a max. of 1 week.

A 76-year-old man with COPD and ischaemic heart disease is admitted to hospital for an exacerbation of his COPD. He has no known drug allergies and his current medication is listed below on his hospital inpatient prescription chart. - aspirin 75 mg, oral, daily - Clexane® (enoxaparin sodium) 40 mg, SC, daily - Fostair NEXThaler® (beclometasone diproprionate 100 micrograms/ fluticasone furmurate 6 micrograms/ inhalation), 1 puff, inhaled, twice a day - isosorbide dinitrate m/r, 40 mg, oral, daily - lisinopril, 20 mg, oral, daily - salbutamol 100 micrograms/inhalation, 2 puffs, inhaled, prn Which of the following drugs would you advise him to rinse his mouth after taking? Aspirin Fostair NEXThaler Isosorbide dinitrate m/r Lisinopril Salbutamol

Fostair NEXThaler. Steroid containing inhalers may increase the risk of oral candidiasis if left in contact with the oral mucosa for a prolonged period.

Mr XX visits your pharmacy to ask about getting some sleep aids as he has been struggling to get a good night of sleep. Which of the following conditions is most likely to be disturbing his sleep pattern? Frozen shoulder Housemaid's knee Lateral epicondylitis (tennis elbow) Medial epicondylitis (golfer's elbow) Rotator cuff syndrome

Frozen shoulder. Pain is observed in all conditions. Epicondylitis are not associated with night pain, and neither is housemaid's knee. Rotator cuff and frozen shoulder can wake the patient, but it is frozen shoulder that is the more likely of the two to cause this problem.

Which cream would be suitable for treatment of a child over the age of 8 with infected eczema because it contains an antibacterial with the steroid? Trimovate® (clobetasone butyrate, oxytetracycline, nystatin) Fucidin® H (hydrocortisone acetate, fusidic acid) Terra-Cortril® (hydrocortisone, oxytetracycline) Alphaderm® (hydrocortisone, urea) Canesten® HC (hydrocortisone, clotrimazole)

Fucidin® H (hydrocortisone acetate, fusidic acid). Trimovate® and Canesten® HC contains antifungal which is not necessary in this case. Alphaderm® does not contain an antibacterial. Terra-Cortril® is not recommended for children.

Whilst working in a community pharmacy, a parent brings in their child with a small rash on their arm. Upon examining the rash you notice a ring-like red rash and you suspect a ringworm infection. What is the cause of a ring worm infection? Bacterial infection Fungal infection Parasite infection Stress induce rash Virus infection

Fungal infection

Mrs BV is taking GTN tablets for her angina. Which of the following counselling points is FALSE with regards to GTN tablets? If the patient experiences chest pain, they should take a tablet immediately by putting it under the tongue. GTN tablets should be discarded 4 weeks after opening them. The tablets should preferably be taken sitting down. The tablets should not be transferred to another container. Facial flushing may occur after taking the tablets

GTN tablets should be discarded 4 weeks after opening them. GTN tablets should be discarded 8 weeks after opening them, not 4.

A middle aged man presents to the pharmacy complaining of epigastric pain. Which condition is most closely associated with pain seen in this region? Appendicitis. Diverticulitis. Gastric ulcer. Irritable bowel syndrome. Renal colic.

Gastric ulcer. Appendicitis has central or right lower quadrant pain; diverticulitis and irritable bowel syndrome are associated with left lower quadrant pain; renal colic pain is usually in the loin area.

For each of the following children, select the single most likely cause behind their complaint. Mr L, a 9-year-old boy, presents with halitosis and dysphagia. His parents tell you that he coughs at night and is complaining of a burning sensation in his upper abdomen. Anaphylaxis Bacterial tracheitis Chronic obstructive pulmonary disease Croup Cystic fibrosis Dyspepsia Gastro-oesphageal reflux disease Peptic ulcer

Gastro-oesphageal reflux disease Gastro-oesophageal reflux disease (GORD) is a clinical condition which results from episodes of reflux of acid. It is characterised by heartburn. Further specialist investigations are required for this patient.

Monoclonal antibodies are sometimes used in malignant diseases. To which monoclonal antibody, does the following statement relate to. '....... is a monoclonal antibody that binds to CD33-expressing tumour cells to induce cell cycle arrest and apopptotic cell death.' Daratumumab Elotuzumab Gemtuzumab ozogamicin Inotuzumab ozogamicin Ipilimumab

Gemtuzumab ozogamicin The drug action for gemtuzumab ozogamicin states: 'Gemtuzumab ozogamicin Is a monoclonal antibody that binds to CD33-expressing tumour cells to induce cell cycle arrest and apopptotic cell death.'

For each of the following patients, select the single most likely cause behind their complaint. Mrs J is a 39-year-old bank assistant who has overwhelming worries about everyday aspects and is unable to relax. She reports periods where her fears worsen and she feels dizzy, gets palpitations, shortness of breath and flushes. She has had these signs and symptoms for the past 9 months. Acute stress disorder Agoraphobia Depression Epilepsy Generalised anxiety disorder Panic disorder Post-traumatic stress disorder Social phobia

Generalised anxiety disorder Generalised anxiety disorder is characterised by persistent excessive and inappropriate anxiety on most days for at least 6 months. The anxiety is not restricted to a specific situation.

You attend a group CPD session on ear problems. The group discusses a case of eczematous inflammation in otitis externa. Which one of the following is indicated in this condition? Almond oil ear drops Gentamicin with hydrocortisone ear drops Olive oil ear drops Oral aciclovir Oral chlorphenamine

Gentamicin with hydrocortisone ear drops. From the list provided, gentamicin with hydrocortisone ear drops are indicated in patients with eczematous inflammation in otitis externa.

For each of the following questions or statements, select the illness which is correct. Pregnant women have to be extra cautious to ensure that they will not come into contact with this disease as there is an increased risk of congenital abnormalities. Atopic eczema Chicken pox (varicella) German measles (rubella) Impetigo Measles (morbilli) Mumps (epidemic parotitis) Slap cheek syndrome (parvovirus) Whooping cough (pertussis)

German measles (rubella) Rubella usually only becomes a serious concern if a pregnant woman catches the infection during the first 20 weeks of her pregnancy. This is because the rubella virus can disrupt the development of the baby and cause a wide range of health problems.

For each of the following patients, select the condition they are likely to be suffering from. A lady is worried about her 17-year-old son. He is very run down and has a headache, sore throat and swollen/tender lymph glands. He has been spending a lot of time with his new girlfriend but isn't well enough to see her at the moment. Chicken pox Glandular fever Impetigo Measles Meningitis Molluscum contagiosum Mumps Rubella

Glandular fever Also termed kissing disease - glandular fever associated with painful swollen glands in teenagers.

An 18-year-old girl enters your pharmacy and asks for your advice regarding a sore throat, tiredness and swollen glands in her neck. What do you suspect may be her condition and most likely cause? Glandular fever caused by Epstein-Barr virus Quinsy secondary to tonsillitis Drug-induced blood dyscrasias Glandular fever caused by varicella-zoster virus Tonsillitis caused by rhinovirus

Glandular fever caused by Epstein-Barr virus.

You are running a clinic for patients with diabetes. You need to be vigilant for signs of diabetic ketoacidosis when withdrawing insulin on patients also taking which medicine? Dipeptidylpeptidase-4 inhibitor Glucagon-like peptide-1 receptor agonist Metformin Sodium glucose co-transporter 2 inhibitor Sulfonylurea

Glucagon-like peptide-1 receptor agonist. Diabetic ketoacidosis has been reported in patients with type 2 diabetes on a combination of a GLP-1 receptor agonist and insulin who had doses of concomitant insulin rapidly reduced or discontinued.

For the presenting signs and symptoms described below, select the most likely condition from the list. A 38-year-old male who presents with excruciating pain in his right ankle at night. He described the ankle as being 'red, inflamed and feels warm'. Rheumatoid arthritis Asthma attack Gout Osteoporosis Anaphylaxis Hernia Ectopic pregnancy Irritable bowel syndrome

Gout. Gout commonly affects the large toe but may also affect the ankles, heels, knees, wrists, fingers and elbows. It commonly occurs at night and may last 3-10 days.

You are counselling a patient on the use of warfarin. Which hot drink would you most likely advise her to avoid? Chamomile tea Coffee Earl grey tea Green tea Mint tea

Green tea. The effects of coumarins can be reduced or abolished by vitamin K, including that found in health foods, food supplements, enteral feeds, or large amounts of some green vegetables or green tea.

For each of the following patients, select the most appropriate treatment option for the scenario described. Mrs D comes into your pharmacy with her 5-year-old son. She tells you that her husband was diagnosed with ringworm one month ago and shows you a picture of a large swelling on a section of his scalp. Betamethasone Clobetasone butyrate Griseofulvin Hydrocortisone Ketoconazole shampoo Miconazole Rest and ibuprofen Tetracycline

Griseofulvin. Scalp infection requires systemic treatment. The swelling on his scalp is a kerion, which could develop in severe ringworm infections, in this case tinea capitis. If left untreated it could result in hair loss and permanent scarring. It's treated using griseofulvin.

The Misuse of Drugs Regulations 2001 (as amended) classify controlled drugs into 5 schedules according to the different levels of control attributed to each. Which schedule includes growth hormones? Schedule 1 Schedule 2 Schedule 3 Schedule 4 Part 1 Schedule 4 Part 2

Growth hormones are Schedule 4 (part 2) CDs.

Which of the following is NOT a risk factor for coronary heart disease? Diabetes Dyslipidemia Haematoma Hypertension Lack of exercise

Haematoma

Mr Y comes into your pharmacy. He mentions that he also has a sore-throat. On further enquiry, he tells you that he takes the following medication: methotrexate, folic acid and paracetamol. What is the most appropriate advice for the patient? - The symptoms described are not related to his medications. - Folic acid minimises the adverse effects of methotrexate, so he should not worry about his symptoms and continue as prescribed. - Advice could be given for oral hygiene and chlorhexidine mouthwash could be used to manage the ulcers. - He should be warned to report this immediately to his GP, as stomatitis may be the first sign of gastro-intestinal toxicity and methotrexate treatment may need to be withdrawn. - He is experiencing a side-effect of methotrexate and it is safe to continue taking this drug

He should be warned to report this immediately to his GP, as stomatitis may be the first sign of gastro-intestinal toxicity and methotrexate treatment may need to be withdrawn Any onset of features of blood disorders (e.g. sore throat and mouth ulcers) should be reported immediately.

You are dispensing a prescription for a patient for an emollient which contains 40% paraffin. The patient tells you he's seen on the television that such emollients can catch fire. What advice would you give him? - He should not smoke or go near naked flames when clothing or bedding has been in contact with an emollient. - There is only a risk of fire if you're a smoker. - There is no risk of fire with paraffin-containing products. - The product contains less than 50% paraffin so there is no fire risk. - Washing his clothing and bedding regularly will ensure there is no emollient present.

He should not smoke or go near naked flames when clothing or bedding has been in contact with an emollient. When recommending, dispensing or selling emollient products to patients, instruct them not to smoke or go near naked flames because clothing or fabric such as bedding or bandages that have been in contact with an emollient or emollient-treated skin can rapidly ignite. Washing clothing or fabric at a high temperature may reduce emollient build-up but not totally remove it.

Regarding head lice, which one of the following statements is true? Head lice are able to jump from head to head. Everybody in the same household must be treated. Head lice can be detected using a fine tooth comb and a piece of white paper. Head lice only affects children aged 4-11 years. Head lice infestation is caused by poor hygiene.

Head lice can be detected using a fine tooth comb and a piece of white paper.

A parenteral drug misuser is seen in the clinic. Which vaccine is indicated for this patient? BCG vaccine Hepatitis A vaccine Human papilloma virus vaccine Intradermal seasonal influenza vaccine Poliomyelitis vaccine for oral use

Hepatitis A vaccine

You are gaining a patient's consent to disclose their information to another pharmacy. Which one of the following things regarding patient information are NOT relevant for the patient to understand? What information will be disclosed Why information will be disclosed Who the information will be disclosed to Consequences of non-disclosure How to contact you if they change their mind

How to contact you if they change their mind

A 12-year-old girl is taken to see her GP by her mother to discuss prevention of cervical cancer. Which vaccine is indicated for this patient? BCG vaccine Hepatitis A vaccine Human papilloma virus vaccine Intradermal seasonal influenza vaccine Poliomyelitis vaccine for oral use

Human papilloma virus vaccine

Mrs F tells you she's getting married next month. She worries that the "bumps" on her hands are something contagious. On closer inspection, you see rough, raised papules with a cauliflower-like appearance and black dots on their surface. What is the cause of this condition? Human papillomavirus Coxsackie virus Varicella zosta virus Herpes simplex virus type 1 Herpes simplex virus type 2

Human papillomavirus

Mr B brings his 8-year-old son into your pharmacy. After examination, it is clear that he is suffering from athlete's foot. Which of the following treatments would NOT be recommended for the 8 year old for OTC supply? Hydrocortisone 0.1% Clotrimazole 1% cream Miconazole 2% cream Canesten® dermatological spray Canesten® cream

Hydrocortisone 0.1%. Steroid creams are not recommended in this setting as they can worsen the fungal infection with immunosuppressive effects.

For each of the following patients, select the most likely drug which results in the adverse effect mentioned. Mrs U tells you that she has been suffering from blurred vision after taking this medicine orally. Ciclosporin Diclofenac Gold Hydroxychloroquine Methotrexate Penicillamine Prednisolone Sulfasalazine

Hydroxychloroquine Hydroyxchloroquine is associated with retinopathy.

Which of the following medications are listed in the BNF as suitable for treatment of pruritus? Carbamazepine Cinnarizine Hydroxyzine hydrochloride Ketotifen Loratadine

Hydroxyzine hydrochloride. Hydroxyzine hydrochloride is indicated for use in pruritus.

Mrs T comes into your pharmacy. Her family is going on a cruise in two weeks time and she is concerned in case any of them experiences travel sickness. She has three children aged 4-8 and she wishes to purchase something which will suit the whole family. Which of the following would be suitable for Mrs T to buy for her children? Domperidone Hyoscine butylbromide Hyoscine hydrobromide Metoclopramide Nabilone

Hyoscine hydrobromide. Domperidone, metoclopramide and nabilone are prescription only medicines. Hyoscine butylbromide (Buscopan) is not indicated for travel sickness, hyoscine hydrobromide (Joy-Rides or Kwells) can be sold to the public and is indicated for travel sickness. References: BNFC, Central nervous system

Which of these medical terms best describes the medication side effect described in the scenarios below? A patient is suffering from excessive skin sensitivity to non-noxious stimuli secondary to the use of nilotinib for the treatment of chronic myeloid leukaemia. Hyperhidrosis Haematoma Hyperaesthesia Melaena Haematuria Hypertrichosis Rhinosinusitis Epistaxis

Hyperaesthesia

Which of the following can be a side-effect of intravenous salbutamol? Hyperphosphataemia Hypermagnesaemia Hypercalcaemia Hypernatraemia Hyperglycaemia

Hyperglycaemia

You are the pharmacist on duty in a community pharmacy that offers emergency contraception. Miss CJ presents to your pharmacy for emergency contraception. She is 18 years old and would like advice on the most effective form of contraception going forward. What is the most appropriate answer? Combined hormonal oral contraceptive Diaphragm IUD Male condom Natural family planning

IUD. An Intrauterine device is more than 99% effective. With typical use the combined oral contraceptive is around 91% effective. With typical use male condoms are around 82% effective. With typical use diaphragms are around 71-88% effective, the natural family planning method is around 76% effective.

Which drug is used to treat pain accompanied by inflammation in children e.g. dental pain? Buprenorphine Codeine Ibuprofen Morphine Paracetamol

Ibuprofen is the only agent with clinically relevant anti-inflammatory effects.

A 9-year-old child has broken his arm when he fell over in the playground. He has been taken to hospital and his arm has been put in a cast. He has no allergies and is not taking any other medicines or has any other medical conditions. He is in moderate pain. What is the most appropriate drug to manage the pain and inflammation? Aspirin Codeine Ibuprofen Paracetamol Tramadol

Ibuprofen. Aspirin has both properties but has a higher risk (Reyes syndrome) compared with ibuprofen. Paracetamol has no clinically relevant anti-inflammatory properties. Codeine and tramadol are analgesics but not anti-inflammatories.

Quinolones may induce convulsions in patients that have not previously experienced them. Taking which medicine concomitantly with quinolones could also induce convulsions? Co-codamol Codeine Ibuprofen Paracetamol Tramadol

Ibuprofen. The CSM has warned that quinolones may induce convulsions in patients with or without a history of convulsions; taking NSAIDs at the same time may also induce them.

A patient with bipolar disorder, treated with lithium, requires analgesia. Which is the LEAST appropriate analgesic for this patient? Codeine Ibuprofen Morphine Paracetamol Tramadol

Ibuprofen. A serious interaction between lithium and NSAIDs can occur leading to reduced excretion of lithium and subsequent toxicity. CNS side effects of opioid analgesics are not desired but the risk of toxicity is lower. The pharmacokinetic interaction is more important than the pharmacodynamic interactions.

For each of the following medicines, select the correct point for the patient. A 45-year-old lady receiving lamotrigine for the treatment of epilepsy. - If the patient suffers a severe skin reaction in the first 8 weeks, the medicine should be stopped immediately. - The patient should be advised that they require routine monitoring of blood levels of the medicine. - The patient should be asked to sign the back of the prescription form as confirmation of collection. - The patient should be provided with a Patient Card. - The patient should immediately report a sore throat to a healthcare professional. - The patient should take the medicine once weekly. - The patient should stop taking this medicine if they have a seizure and seek further medical advice. - The same brand must always be supplied.

If the patient suffers a severe skin reaction in the first 8 weeks, the medicine should be stopped immediately. Serious skin reactions including Stevens-Johnson syndrome and toxic epidermal necrolysis have developed (especially in children); most rashes occur in the first 8 weeks. Avoid abrupt withdrawal (taper off over 2 weeks or longer) unless serious skin reaction occurs.

From the list below select the most suitable counselling points you would give to the following patients. A 17-year-old girl who has been prescribed permethrin for scabies. - Immediately contact the hospital emergency department if you experience abnormal heart rhythms. - Do not take this medication with any herbal product - Condoms should be used to avoid local reactions and exposure of this agent to women of childbearing age or those who are pregnant - Apply liberally to scalp - If you experience numbness or tingling of your finger or toes, stop your medication immediately and contact your doctor. - Using a nit comb with this is effective - If you wash your hands reapply treatment - This medication is most effective at night

If you wash your hands with soap within 8 hours of application, they should be treated again with cream

For the described patients, select the most likely counselling advice you would give them from the list below. A 24-year-old with severe eczema who is prescribed azathioprine. - Take this medication on an empty stomach. - Take this medication after food. - You may experience sudden onset of sleep. Please exercise caution when performing skilled tasks. - Report any signs of visual impairment immediately to your prescriber. - Your medication may colour your urine blue. This is harmless. - Do not take this medication at the same time as antacids. - Immediately report any signs or symptoms of anaemia, bruising or infection to your doctor. - Take this medication at night

Immediately report any signs or symptoms of anaemia, bruising or infection to your doctor Anaemia, bruising or infection are signs of bone marrow suppression.

For the described patients, select the most likely counselling advice you would give them from the list below. A 32-year-old male who has been prescribed lamotrigine for the first time. - Take this medication on an empty stomach. - Take this medication after food. - You may experience sudden onset of sleep. Please exercise caution when performing skilled tasks. - Report any signs of visual impairment immediately to your prescriber. - Your medication may colour your urine blue. This is harmless. - Do not take this medication at the same time as antacids. - Immediately report any signs or symptoms of anaemia, bruising or infection to your doctor. - Take this medication at night

Immediately report any signs or symptoms of anaemia, bruising or infection to your doctor Anaemia, bruising or infection are signs of bone marrow suppression.

For each of the following patients, select the condition they are likely to be suffering from. A 3-year-old child has an area of red itchy skin around his mouth and nose.He also has vesicles in the same area that have ruptured and are weeping. Some of the exudate has dried to a brown, yellow sticky crust. Chicken pox Glandular fever Impetigo Measles Meningitis Molluscum contagiosum Mumps Rubella

Impetigo Location of rash and age of child indicative of impetigo.

You are working on the neonatal unit and the nurse asks you to dispense a prescription for prophylactic sodium feredetate 1 mL daily for baby Tanya who was born 15 days ago, weighing 2.4 kg and is breastfed by her mother. When should you advise the baby's mother to start administering the sodium feredetate? Immediately In one weeks time In two to four weeks time In three weeks time In four to six weeks time

In two to four weeks time. Supplementation is started in 4-6 weeks after birth. Baby Tanya is already 15 days old at the time that you are advising her mother. This needs to be factored in to the advice you give so the answer 2-4 weeks time.

For each of the following patients, select the most relevant safety concern associated with the patient's medication. A 43-year-old woman is on the following medication: metformin 500 mg twice daily aspirin 75 mg daily amlodipine 10 mg daily She has been newly prescribed pioglitazone 15 mg once daily. Atypical femoral fractures Benign idiopathic osteonecrosis Fatal diabetic ketoacidosis Increased incidence of heart failure Risk of severe harm and death Subacute cutaneous lupus erythematosus Suicidal ideation Uterine perforation

Increased incidence of heart failure MHRA has issued a safety alert for increased risk of heart failure linked with pioglitazone.

This is your first shift at a registered pharmacy that prepares unlicensed medicines. With respect to the ingredients used in the preparation of an unlicensed medicine, which of the following is NOT a requirement for the record keeping process? Batch number. Certificate of analysis. Certificate of conformity. Ingredient manufacture date. Manufacturer, brand and wholesaler/distributor

Ingredient manufacture date. You should keep detailed records of the preparation of unlicensed medicines in order to safeguard patients (i.e. If there is an incident where a patient's safety is affected, or the medicine should be recalled, the method of preparation could be reconstructed). For the ingredients used in the preparation, the following should be recorded: - source (manufacturer, brand, wholesaler/distributor) - certificate of conformity - certificate of analysis - batch number - expiry date - quantity used and details of person measuring and person double-checking quantities. TSE guidance should be followed (where ingredient or product contact material is of animal origin) description of container/closure used (for example, were they were glass/plastic)

Jayden is 12 years old and presents with non blanching rash, fever and sensitivity to light. Suspects meningitis. What is initial treatment?

Initial empirical therapy is benzypenicillin sodium before transfer to hospital. Cefotaxime can be given in penicillin allergy. Chloramphenicol can be used if history of immediate hypersensitivity to penicillins or cephalosporins. Once patient is in hospital adjunctive treatment with dexamethasone is given no later than 12 hours after starting antibacterial therapy. However avoid this if they have septic shock, immunocompromised or following surgery.

You receive a prescription for phenytoin injection 250 mg/5 mL, 100 mg three times daily for one of your patients. The trainee dispensing technician comments that she has never seen phenytoin prescribed in injection form before and asks you why this is. What is the best explanation to give her? Injectable phenytoin decreases adherence Injectable phenytoin increases the risk of adverse events Injectable phenytoin is more error-prone with a risk of death Injectable phenytoin provides a slower response Injectable phenytoin reduces efficacy

Injectable phenytoin is more error-prone with a risk of death. Although the other points may be factually correct, in a practice the reason for phenytoin not being prescribed as an injection is primarily that it is more error-prone than in oral form and this can lead to an increased risk of patient deaths.

A patient complains of tinnitus and deafness. Based on this information only, what part of the ear is most likely affected? Inner ear Inner and middle ear Either middle or outer ear Middle ear Outer ear

Inner ear. Tinnitus is a symptom most closely associated with the inner ear so inner ear or inner and middle ear option could be correct, however middle ear problems are not as frequently associated with tinnitus so inner ear is the most likely answer.

A dispenser asks you about high strength insulin. Which of the following medicines are considered a high strength insulin? Insulin aspart 100 units/mL Insulin degludec 100 units/mL Insulin glargine 100 units/mL Insulin glulisine 100 units/mL Insulin lispro 200 units/mL

Insulin lispro 200 units/mL. Insulin strengths greater than 100 units/mL are considered high strength insulins.

For each of the following patients, select one answer from which would it be most important to monitor. A 64-year-old man prescribed warfarin 1 mg daily. He does not have any other medical conditions and has no known allergies. Alanine transferase C-reactive protein Drug-serum levels Blood glucose International normalised ratio Potassium Red blood cell count Sodium

International normalised ratio INR levels must be monitored in patients who take warfarin.

A 16-year-old patient is admitted into hospital with confusion, recent weight loss and 'sweet smelling' breath. On admission, it is found that the patient is not taking any regular medication and has a blood glucose level of 24 mmol/L. Which of the following is the most appropriate initial treatment for this patient? Gliclazide Intravenous hydrocortisone Intravenous soluble insulin Metformin Zoledronic acid

Intravenous soluble insulin The signs and symptoms indicate that the patient is suffering from diabetic ketoacidosis. The BNF states to start an intravenous insulin infusion.

For each of the following questions, select the vitamin or mineral which is the correct answer. When a patient has an overload of this substance, deferasirox should be administered to the patient. Ascorbic acid Calcium Folic acid Iron Retinol Vitamin D Vitamin K Zinc

Iron Indicated for the treatment of chronic iron overload when desferrioxamine is contraindicated or inadequate (with non-transfusion-dependent thalassaemia syndromes).

For each of the following questions, select the vitamin or mineral which is the correct answer. Which of the following has a side effect that can cause stools to be darkened? Ascorbic acid Calcium Folic acid Iron Retinol Vitamin D Vitamin K Zinc

Iron Iron (ferrous) can cause GI upset with one of the side effects being darkened stools.

Young children are prone to otitis media. Which of the following symptom groups best describe the signs and symptoms of otitis media in this age group? Irritability and crying Irritability and fever Irritability and pain Irritability, crying and pain Irritability, crying, fever and pain

Irritability, crying, fever and pain. Pain is the most common symptom and so eliminates options A and B. Crying is also prominent and thus eliminates option C. D and E are both possible but fever is often seen; thus E is the correct option.

Primary dysmenorrhoea is a common presenting symptom seen by community pharmacists and has a well described set of symptoms. Which of the following statements is FALSE with regard to primary dysmenorrhoea? Back pain is often experienced. Is always caused by an underlying condition. Pain is described as dull and continuous. Pain tends to decrease once the period has begun. Pain typically starts just before the period starts.

Is always caused by an underlying condition. Period pain usually begins around the onset of menstruation; pain is seen prior to the period starting and lessens once it has started; not all, but a substantial number of women, experience back pain. It is painful cramping and usually throbbing.

For each of the following patients, select the combination of drugs that would be used for their treatment. A patient is admitted to hospital with tuberculosis and requires the combination of drugs used in the continuation phase of tuberculosis treatment. Abacavir, lamivudine and lopinavir Amoxicillin, clarithromycin and lansoprazole Clarithromycin, metronidazole and omeprazole Clavulanic acid and ticarcillin Efavirenz, emtricitabine and tenofovir Ethambutol, isoniazid, pyrazinamide and rifampicin. Ethambutol and rifampicin Isoniazid and Rifampicin and pyridoxine

Isoniazid and rifampicin, pyridoxine hydrochloride Recommended drug regimen for continuation phase of tuberculosis treatment.

A patient with tuberculosis requires antibiotic treatment. Which antibiotic is the primary reason for pyridoxine supplementation? Capreomycin Ethambutol Isoniazid Pyrazinamide Rifampicin

Isoniazid. Peripheral neuropathy can occur as a result of isoniazid especially in those at highest risk (see BNF) and in these patients pyridoxine should be prescribed when treatment with isoniazid is started.

A 52-year-old patient is diagnosed with angina after complaining of chest pain and has also been recently diagnosed with T2DM. After taking his newly prescribed medication, he comes into your community pharmacy to complain of severe headaches. Which of the following newly prescribed medication is most likely causing these headaches? Aspirin Gliclazide Isosorbide mononitrate Paracetamol Simvastatin

Isosorbide mononitrate A well-known side of nitrates is 'throbbing headache'. The only nitrate in the list above is isosorbide mononitrate.

For each of the following scenarios, select the drug which is correct in relation to the scenario. An elderly patient is admitted to the ward with recurrent falls and reports episodes of light headedness prior to falling. The consultant asks which of his medications could be the most likely to be contributing to this. The patient has a history of angina. Benzydamine hydrochloride Dalteparin Duloxetine Isosorbide mononitrate Isotretinoin Pizotifen Timolol 0.25% eye drops Xylometazoline hydrochloride 0.1%

Isosorbide mononitrate Isosorbide mononitrate exerts its mechanism of action by dilating blood vessels to reduce pressure build up. As a result this may cause episodes of dizziness and light headedness which can result in falls.

A patient has been newly started on a number of medicines for a cardiovascular condition. Which agent is most likely to cause headaches in this patient? Bisoprolol Diltiazem Isosorbide mononitrate Ramipril Spironolactone

Isosorbide mononitrate. The patient is newly started on the medicines and isosorbide mononitrate is known to have the potential to cause headaches, particularly on initiation. In this situation, although other medicines may cause a headache, this would be the most likely cause and this should be excluded before looking at other causes for the headaches.

For each of the following statements, select the drug that fits most closely. For women of childbearing potential, each prescription for oral treatment with this drug should be limited to a supply of up to 30 days' treatment. Alendronate Disulfiram Isoniazid Isotretinoin Ketoconazole Lithium Rifampicin Sulfasalazine

Isotretinoin Given the highly teratogenic nature of isotretinoin, supply to women of child bearing age should be limited to a maximum of 30 days.

For each of the following scenarios, select the drug which is correct in relation to the scenario. A patient walks into the community pharmacy you are working in with a complaint of dry skin and in particular dry chapped lips. They want to know if any of the medications they are regularly taking could be contributing to or causing this. Benzydamine hydrochloride Dalteparin Duloxetine Isosorbide mononitrate Isotretinoin Pizotifen Timolol 0.25% eye drops Xylometazoline hydrochloride 0.1%

Isotretinoin Isotretonoin commonly causes skin thinning, drying of the skin and chapped skin. As such patients should be adequately counselled on how to manage these side effects and how to take precautions in the sun as this medication also leaves their skin more susceptible to the harmful UV sun rays.

For each of the following scenarios, select the drug which is correct in relation to the scenario. Each supply of this medication to women of childbearing age should be limited to 30 days only. Benzydamine hydrochloride Dalteparin Duloxetine Isosorbide mononitrate Isotretinoin Pizotifen Timolol 0.25% eye drops Xylometazoline hydrochloride 0.1%

Isotretinoin Isotretonoin is subject to many safety and legal precautions, one of which is the supply that is allowed must not exceed 30 days.

For each of the following scenarios, select the drug which is correct in relation to the scenario. Before dispensing this medication you must ensure the patient is not pregnant and repeat the pregnancy test each month. If at any time during therapy the patient falls pregnant they must inform a clinician immediately. Benzydamine hydrochloride Dalteparin Duloxetine Isosorbide mononitrate Isotretinoin Pizotifen Timolol 0.25% eye drops Xylometazoline hydrochloride 0.1%

Isotretinoin Isotretonoin is subject to many safety precautions, given that it is teratogenic females must undergo a pregnancy test within three days of a prescription to rule pregnancy and prevent any harm to an unborn child.

For each of the following scenarios, select the drug which is correct in relation to the scenario. Certain medications are restricted with regards to amounts that can be prescribed at any one time. Repeat prescriptions of this medication are not allowed and should not be authorised. Benzydamine hydrochloride Dalteparin Duloxetine Isosorbide mononitrate Isotretinoin Pizotifen Timolol 0.25% eye drops Xylometazoline hydrochloride 0.1%

Isotretinoin Isotretonoin is subject to many safety precautions, one of which is that the medication is NOT allowed to be dispensed on a repeat prescription.

For each of the following scenarios or statements, select the drug which is correct. This laxative is dispensed with BNF advisory label 13 'Dissolve or mix with water before taking'. Bisacodyl Co-danthramer Docusate sodium Glycerol Ispaghula husk Lactulose Methylcellulose Senna

Ispaghula husk. Isphagula husk is dispensed as individual sachets of powder. It requires reconstitution with water prior to administration.

For each of the following questions, select the drug which is correct in relation to the scenario. This is a bulk-forming laxative and should be mixed with water. Isphaghula husk Methylcellulose Bisacodyl Co-danthramer Docusate sodium Senna Lactulose Glycerol

Isphaghula husk. Isphagula husk is a bulk-forming laxative which relieves constipation by increasing faecal mass which stimulates peristalsis.

You are working in a community pharmacy. A patient presents to you with a cough and runny nose. They have had this for two days. The patient asks which antibiotics are best to take for their symptoms. The patient has no other symptoms. What would the most appropriate advice be to give to the patient? - A swab of the nose would identify the most appropriate antibiotic. - A swab of the throat would identify the most appropriate antibiotic. - It is likely that they are suffering from a virus and that antibiotics will be ineffective. Self help would be most appropriate currently. - Their GP will be able to make that decision for them based on their reported symptoms - They should ask their GP for amoxicillin

It is likely that they are suffering from a virus and that antibiotics will be ineffective. Self help would be most appropriate currently. Pharmacists should demonstrate concepts of health promotion, health education and health improvement programmes based on national and local health priorities and parameters. Public Health England runs campaigns on preventing antibiotic resistance. A patient who has had a cough and runny nose for 2 days is unlikely to need antibiotic treatment at this point.

Which one of the following statements best describes the role of linezolid in the treatment of infections caused by methicillin-resistant Staphylococcus aureus (MRSA)? It does NOT have a role in the treatment of infections caused by MRSA It is not active against Gram-negative organisms It must be given with another antibacterial where infection involves Gram positive bacteria It may be used for bronchiectasis caused by MRSA It is first line choice for all infections caused by MRSA

It is not active against Gram-negative organisms

A patient is prescribed methotrexate for rheumatoid arthritis. Which statement is the LEAST appropriate for this patient? It is taken once a week. It may cause respiratory effects. It may cause liver problems. It may cause hearing disturbances. It may cause blood problems.

It may cause hearing disturbances. Other options are important counselling points for patients taking methotrexate.

A registered midwife calls and asks for advice. The midwife wishes to use a midwife supply order to order an opiate based analgesic. The opiate they wish to obtain is pethidine. Which of the following is INCORRECT regarding the supply order? It must contain the name and address of the midwife It must contain the occupation of the midwife It must contain name of the patient for which the controlled drug is being obtained It must contain the total quantity of drug to be obtained It must contain the signature of an appropriate medical officer

It must contain the name and address of the midwife. It is not a pre-requisite for a midwife to state their address on a midwife supply order.

Which of the following laxative agents is safest for a pregnant woman? Lactulose Liquid paraffin Co-danthramer Sodium docusate Sodium picosulphate

Lactulose. BNF monograph for lactulose says not known to be harmful in pregnancy. Other options are not recommended in pregnancy or to be used with caution. Liquid paraffin is not recommended for use as a laxative.

A doctor in A&E is seeing a patient with a history of alcohol misuse and is querying which laxative can be used for hepatic encephalopathy. Which of the following laxatives can be used for this indication? Bisacodyl Co-danthramer Docusate sodium Lactulose Senna

Lactulose. Through its mechanism of action lactulose prevents the reabsorption of ammonia and other toxins from faecal matter in the gut which can cause hepatic encephalopathy.

Select the best answer from the following question... You are dispensing an antiepileptic drug for a patient with epilepsy. For which of the following antiepileptic drugs should you consult the prescriber to establish if the same brand is to be dispensed each time? Carbamazepine Gabapentin Lamotrigine Levetiracetam Vigabatrin

Lamotrigine. Carbemazepine is incorrect as patients should always be kept on the same brand of that.

For each of the following statements, select the drug that fits most closely. Patients being treated with this medication may need to be maintained on a specific manufacturer's branded or generic product. Desmopressin Diazepam Flucloxacillin Lamotrigine Misoprostol Paroxetine Salbutamol Vigabatrin

Lamotrigine. Different brands of this drug have varying bioavailabilities. As such, they must be prescribed brand specific formulations for individual patients.

For each of the following statements, select the drug that fits most closely. When using this drug in the treatment of epilepsy, care should be taken when switching between oral formulations. Desmopressin Diazepam Flucloxacillin Lamotrigine Misoprostol Paroxetine Salbutamol Vigabatrin

Lamotrigine. Different brands of this drug have varying bioavailabilities. As such, they must be prescribed brand specific formulations for individual patients. Category 2 - For these drugs, the need for continued supply of a particular manufacturer's product should be based on clinical judgement and consultation with the patient and/or carer taking into account factors such as seizure frequency, treatment history, and potential implications to the patient of having a breakthrough seizure.

Which of the following antisecretory and mucosal protectant drugs is a proton pump inhibitor? Cimetidine Misoprostol Lansoprazole Ranitidine Tripotassium dicitratobismuthate

Lansoprazole.

Some medicines are subject to additional monitoring requirements co-ordinated by the manufacturer, including clozapine. Which of the following are monitored under the clozapine patient monitoring service? Red blood cells. Full blood count. Leucocyte count only. Leucocyte and neutrophil counts. Neutrophil count only.

Leucocyte and neutrophil counts. The UK Clozaril Patient Monitoring Service (CPMS) was developed in order to manage the risk of agranulocytosis associated with clozapine. Patients are centrally monitored for their leucocyte and neutrophil counts.

Mrs W is a 75-year-old lady with a leg ulcer who is on your ward. She has acute pain over the weekend in her knee and the doctor thinks it might be gout.You check her renal function on Monday and it has considerably deteriorated. Her doctor asks you to have a look at her medication history and determine which one, if any, of her medicines is most likely to be related to a decline in renal function. - Ibuprofen 400 mg QDS - Flucloxacillin 500 mg QDS - Penicillin VK 500 mg QDS - Paracetamol 1 g QDS What would be the most appropriate response? Not likely to be related to any of the medicines. Related to a drug interaction between the medicines. Likely to be caused by the ibuprofen. Likely to be caused by the penicillin VK. Likely to be caused by the paracetamol.

Likely to be caused by the ibuprofen Ibuprofen is associated with renal impairment.

You are teaching a pharmacy student about medicines used in diabetes. Which of the following medicines is a dipeptidylpeptidase-4 inhibitor? Canagliflozin Dulaglutide Linagliptin Metformin Nateglinide

Linagliptin

For each of the following scenarios, select the drug which is correct in relation to the scenario. Severe optic neuropathy may occur with the use of this antibiotic, and as such, visual function should be monitored regularly if treatment is required for longer than 28 days. Aztreonam Benzylpenicllin sodium Flucloxacillin Linezolid Metronidazole Nalidixic acid Phenoxymethylpenicillin Tinidazole

Linezolid Linezolid is known to cause severe optic neuropathy so patients should be advised to report any changes in eyesight to a medical practitioner. All patients should be advised to report symptoms of visual impairment, such as changes in visual acuity, changes in colour vision, blurred vision or any visual field defect. In such cases, prompt evaluation is recommended with referral to an ophthalmologist as necessary. If any patients are taking linezolid for longer than the recommended 28 days, their visual function should be regularly monitored.

A 76-year-old man with COPD and ischaemic heart disease is admitted to hospital for an exacerbation of his COPD. He has no known drug allergies and his current medication is listed below on his hospital inpatient prescription chart. - aspirin 75 mg, oral, daily - Clexane® (enoxaparin sodium) 40 mg, SC, daily - Fostair NEXThaler® (beclometasone diproprionate 100 - micrograms/ fluticasone furmurate 6 micrograms/ inhalation), 1 puff, inhaled, twice a day - isosorbide dinitrate m/r, 40 mg, oral, daily - lisinopril, 20 mg, oral, daily - salbutamol 100 micrograms/inhalation, 2 puffs, inhaled, prn. Which of the following drugs is most likely to be responsible for his dry cough? Clexane Fostair NEXThaler Isosorbide dinitrate m/r Lisinopril Salbutamol

Lisinopril. ACE inhibitors such as lisinopril are associated with a dry, persistent cough.

For each of the following patients, select the drug which is most likely to be responsible for the symptoms described. Mr J, who is 77-years-old, was recently started on a new drug. Three weeks following initiation, Mr J was admitted into hospital and is found to have an abrupt decline in his glomerular filtration rate. He is otherwise well. Amlodipine Atenolol Bendroflumethiazide Bumetanide Doxazosin Lisinopril Methyldopa Verapamil

Lisinopril. Renal impairment may occur in patients with bilateral renal artery stenosis who have been initiated on an ACEI. Such patients rely on angiotensin II to maintain glomerular capillary pressure via vasoconstriction on the efferent arteriole. This is blocked by the ACEI, lisinopril, causing an abrupt decline in the glomerular filtration rate.

For each of the following statements, select the drug which is the most applicable. Symptoms of mild overdose may include blurred vision, light headedness, fine resting tremor, muscular weakness and drowsiness. Amiodarone Clozapine Digoxin Gentamicin Lithium Phenytoin Theophylline Warfarin

Lithium Blurred vision, resting tremor, muscular weakness etc are characteristics to lithium intoxication.

For each of the following statements or scenarios, select the drug which is correct. This drug requires dose adjustment because it has a narrow therapeutic index of 0.4-1 mmol/litre. Dexamfetamine sulfate Lisdexamfetamine mesilate Atomoxetine Methylphenidate hydrochloride Valproic acid Asenapine Lithium carbonate Lithium citrate

Lithium carbonate. Lithium carbonate requires dose adjustment to achieve a serum-lithium concentration of 0.4-1 mmol/litre.

For each of the following statements, select the drug that fits most closely. This drug has several indications including treatment of bipolar disorder. Alendronate Disulfiram Isoniazid Isotretinoin Ketoconazole Lithium Rifampicin Sulfasalazine

Lithium. Lithium is used in an array of mental health conditions, of which bipolar disorder is one of these

For each of the following statements, select the drug that fits most closely. This drug has a narrow therapeutic range. The usual range should be between 0.4 mmol/L and 1 mmol/L. Alendronate Disulfiram Isoniazid Isotretinoin Ketoconazole Lithium Rifampicin Sulfasalazine

Lithium. Treatment with lithium requires routine blood monitoring. Therapeutic levels of lithium are narrow and should be between 0.4 mmol/L and 1 mmol/L.

For each of the following descriptions, select the condition that is most accurate. Usually is precipitated by new or additional activity. Bursitis Chondromalacia Lateral epicondylitis Lower back pain Medial epicondylitis Rotator cuff syndrome Sciatica

Lower back pain New activity is associated with acute presentation, and therefore rules out most listed conditions as they develop over time and/or through repeated activity. Bursitis and lower back pain are most likely, but the most common cause of bursitis is due to prolonged pressure.

For each of the following descriptions, select the condition that is most accurate. Increased pain is observed when sedentary and relieved with exercise. Bursitis Chondromalacia Lateral epicondylitis Lower back pain Medial epicondylitis Rotator cuff syndrome Sciatica

Lower back pain Only lower back pain and chondromalacia are affected by rest, both of which can worsen. However, lower back pain worsens in chondromalacia on exercise (runner's knee), whereas back pain is eased.

You are training your pharmacy staff on different types of skin rashes. Which of the following conditions is associated with a rash which is not itchy? Ringworm. Impetigo. Dermatitis herpetiformis. Nappy rash. Lyme disease.

Lyme disease. Lyme disease is caused by a bacterial infection from tick bites. An early symptom of this condition is a characteristic "bulls eye" rash at the site of the bite.

You are a hospital pharmacist working in ICU. Mr K, who is a pharmacy summer student, comes to ask you about the acronym MDRD which he saw in a patient's medical record. Regarding this acronym, which one of the following is correct? MDRD stands for Malnutrition disease related to diabetes MDRD stands for Malnutrition doubles the risk of developing disease MDRD stands for Malnutrition of diet in renal disease MDRD is used for patients in the ICU who have special dietary requirements MDRD is used to measure eGFR of a patient

MDRD is used to measure eGFR of a patient. MDRD stands for 'modification of diet in renal disease'. It is used to calculate the eGFR of a patient.

Which of the following laxative agents is first line for faecal impaction in a patient with hard stools? Lactulose Liquid paraffin Co-danthramer Sodium docusate Macrogols

Macrogols If there are hard stools, consider prescribing a high dose of an oral macrogol. If there are soft stools, or ongoing hard stools after a few days of treatment with an oral macrogol, consider starting or adding an oral stimulant laxative.

You receive a prescription for tramadol capsules 50 mg on 1 February 2021. The prescription is signed and dated 1 January 2021. What would be the most appropriate course of action with this prescription? Dispense the prescription. Make the GP aware the patient needs a new prescription. Sell the patient co-codamol over the counter. Send the patient to another pharmacy to get it dispensed. Supply the tramadol to the patient and ask the prescriber for a new prescription.

Make the GP aware the patient needs a new prescription Prescriptions for tramadol are valid for 28 days and a new prescription would be required.

For each of the following patients, select the condition they are likely to be suffering from. A 5-year-old child has a rash that started around her ears and moved down to her trunk and limbs. In addition, she has had a fever, head cold and a cough. On her inner cheeks and gums she has small white spots. Chicken pox Glandular fever Impetigo Measles Meningitis Molluscum contagiosum Mumps Rubella

Measles White koplick spots inside mouth are indicative of measles.

For each of the following questions or statements, select the illness which is correct. Which illness is characterised by white spots surrounded by a red ring on the inner cheek and gums? Atopic eczema Chicken pox (varicella) German measles (rubella) Impetigo Measles (morbilli) Mumps (epidemic parotitis) Slap cheek syndrome (parvovirus) Whooping cough (pertussis)

Measles (morbilli) The difference between German measles and measles (morbilli) is that morbilli manifests itself with white spots in the mouth.

You are training your pre-registration pharmacist on immunisations. Which of the following disease, that could be prevented by vaccination, is the most contagious? Influenza Measles Mumps Pertussis Smallpox

Measles. A person with measles is likely to infect an average of 12-18 people, the rate is 1.4-4 for influenza, 4-7 for mumps, 12-17 for pertussis and 5-7 for smallpox

Mr F comes into your pharmacy with his 2-year-old son. His son has a rash which extends from the face down to the chest. The rash is made up of small red-brown, slightly raised spots that join together into larger blotchy patches. He has a runny nose, conjunctivitis and has greyish-white spots with a red ring in the mouth. Which of the following childhood infections are these symptoms most likely to represent? Chicken pox Measles Meningitis Mumps Rubella

Measles. The symptoms are those of measles and could also include pyrexia, loss of appetite, aches and pains.

The Misuse of Drugs Regulations 2001 (as amended) classify controlled drugs into 5 schedules according to the different levels of control attributed to each. Which schedule includes midazolam? Schedule 1 Schedule 2 Schedule 3 Schedule 4 Schedule 5

Midazolam is a Schedule 3 CD.

All pharmacies in England are expected to offer the disposal of unwanted medicines to the public. However, for the safety of the public and pharmacy staff there are certain items that are not taken back by pharmacies under this scheme. Which ONE of the following products may be accepted from the public as medicinal waste, under this essential service? Mercury thermometers Solvents Batteries Medicinal ampoules Garden chemicals

Medicinal ampoules. The pharmacy will, if required by NHS England or the waste contractor, sort them into solids (including ampoules and vials), liquids and aerosols, and the local NHS England team will make arrangements for a waste contractor to collect the medicines from pharmacies at regular intervals.

Which one of the following pieces of information relating to a patient's medication is NOT normally required when completing a medication history? Medicines required only on a prn basis Dietary supplements Medicines purchased OTC Herbal medicines Medicines stopped 6 months ago

Medicines stopped 6 months ago. Medication history should include medicines that have recently stopped or changed and although the MEP does not define how recent it would have to be, it is unlikely that medicines taken 6 months ago would be relevant to a current medication history.

For each of the following, select the most appropriate reference source that contains information relating to the topic listed. Destruction of controlled drugs. British National Formulary British Pharmacopoeia Clinicaltrials.gov Electronic Medicines Compendium Martindale - The Complete Drug Reference Medicines, Ethics and Practice - The professional guide for pharmacists National Institute for Health and Care Excellence guidelines Pubmed

Medicines, Ethics and Practice - The professional guide for pharmacists Refer to MEP for destruction of controlled drugs.

You want to obtain guidance on prescribing restrictions for podiatrist independent prescribers. Which of the following is the most appropriate reference source to use to find this information? British National Formulary. British Pharmacopoeia Martindale: The Complete Drug Reference Medicines, Ethics and Practice Stockley's Drug Interactions

Medicines, Ethics and Practice. Restrictions by prescriber types are legal requirements and can be found in the MEP.

Which of these medical terms best describes the medication side effect described in the scenarios below? A patient is prescribed ertapenem for the treatment of a diabetic foot infection and has black, tarry stools as a side effect of this medication. Hyperhidrosis Haematoma Hyperaesthesia Melaena Haematuria Hypertrichosis Rhinosinusitis Epistaxis

Melaena

For each of the following patients, select the condition they are likely to be suffering from. A 7-year-old child has a severe headache, stiff neck and a dislike a light. In addition, she has a non-blanching rash as well as a fever and she feels nauseous. Chicken pox Glandular fever Impetigo Measles Meningitis Molluscum contagiosum Mumps Rubella

Meningitis Non-blanching rash, stiff neck and dislike of light all indicative of meningitis.

For each of the following descriptions, select the condition that is most accurate. Is most frequently seen in young children. Cluster headache Medication-overuse headache Meningitis Migraine Sinusitis Sub-arachnoid haemorrhage Temporal arteritis Tension-type headache

Meningitis Option G can be eliminated as it is a condition associated with advancing age; cluster headache, tension-type headache are associated with onset in adulthood; sinusitis is a complication of colds and shows no real pattern with regards to age of onset, as is the case with medication-overuse headache; migraine can start in adolescence, but meningitis is most closely associated with young children.

Elise shah is 14 years old. Has been diagnosed with type 2 diabetes. Her HBA1C is 58. What should be recommended?

Metformin

You are explaining the mechanism of action of different anti-diabetes medicines to pharmacy students. Link the medicine to the mechanism of action. Which medicine decreases gluconeiogenesis by increasing peripheral utilisation of glucose? Acarbose Empagliflozin Exenatide Gliclazide Metformin Repaglinide Saxagliptin Short-acting insulin

Metformin Metformin decreases gluconeiogenesis by increasing peripheral utilisation of glucose.

Mrs H is a 82-year-old retired pilot. She has chronic kidney disease (CKD) with the following reported values: eGFR: 25 ml/min/1.73 m2, ACR: 17 mg/mmol. Which one of the following drugs is LEAST suitable for use in Mrs H? Allopurinol Atenolol Metformin Paracetamol Simvastatin

Metformin. Manufacturer advises avoid if eGFR is less than 30 mL/minute/1.73 m2.

An 86-year-old patient is admitted to the acute medical ward in hospital. It is determined that the patient has a stage one acute kidney injury. Which of the following medications will need to be held until this has resolved? Bisacodyl Metformin Paracetamol Simvastatin Tramadol

Metformin. Metformin should be withheld because of an increased risk of lactic acidosis for patients with AKI.

You are planning to start a local methotrexate clinic. According to BNF safety information, which of the following is NOT a recommendation to avoid dosing errors? Methotrexate should be prescribed by brand Only one strength of table should be prescribed The dispensing label should clearly display the dose of methotrexate The dispensing label should clearly display the frequency of administration The patient should be warned to report a sore throat immediately

Methotrexate should be prescribed by brand. To avoid error with low-dose methotrexate, it is recommended that: - the patient is carefully advised of the dose and frequency and the reason for taking methotrexate and any other prescribed medicine (e.g. folic acid); - only one strength of methotrexate tablet (usually 2.5 mg) is prescribed and dispensed; - the prescription and the dispensing label clearly show the dose and frequency of methotrexate administration; - the patient is warned to report immediately the onset of any feature of blood disorders (e.g. sore throat, bruising, and mouth ulcers), liver toxicity (e.g. nausea, vomiting, abdominal discomfort, and dark urine), and respiratory effects (e.g. shortness of breath).

For each of the following statements, select the drug which is correct. This drug is considered safe to treat hypertension during pregnancy. Apixaban Aspirin Atenolol Digoxin Lacidipine Methyldopa Ramipril Urokinase

Methyldopa Methyldopa is often used to treat high blood pressure that is diagnosed during pregnancy.

Miss K is a 21-year-old female who presents with a round bloated, moon-like face, facial acne and stretch marks on her abdomen. These signs are associated with high doses of which one of the following drugs? Clindamycin Folic acid Aspirin Methylprednisolone Magnesium trisilicate

Methylprednisolone. High doses of any corticosteroid is associated with Cushing's syndrome which presents with moon face, striae and acne.

Miss T comes into your Pharmacy. She asks you about contraception. She's on a combined oral contraceptive and wonders whether this is sufficient or if she should take further contraceptive precautions based on her medication. Which of the following medications do NOT result in the need for additional contraceptive precautions? Carbamazepine Metoclopramide Rifampicin St John's wort Topiramate

Metoclopramide. Metoclopramide does not affect combined oral contraceptives. Drugs which are liver enzyme inducers could reduce the efficacy of combined oral contraceptives and hence additional contraceptive precautions must be taken whilst on these medications. Examples of drugs which induce liver enzymes are: - antibacterials e.g. rifampicin - antiepileptics - antiretrovirals - St John's wort.

For each of the following scenarios, select the drug which is correct in relation to the scenario. This antibiotic is commonly used in infections where anaerobic bacteria are the suspected cause and it is commonly used in acute pericoronitis. Aztreonam Benzylpenicllin sodium Flucloxacillin Linezolid Metronidazole Nalidixic acid Phenoxymethylpenicillin Tinidazole

Metronidazole Metronidazole is commonly used in dental conditions such as acute oral infections due to the spectrum of bacteria it covers which includes anaerobes.

For each of the following scenarios, select the drug which is correct in relation to the scenario. A patient contacts you at the local community pharmacy to discuss symptoms of flushing, headache and vomiting. The patient is unsure if an antibiotic is the causative factor. Upon further questioning, she informs you that she has had two days treatment of the antibiotic and on the second day of treatment drank three large glasses of wine. What is the likely antibiotic the patient is taking? Aztreonam Benzylpenicllin sodium Flucloxacillin Linezolid Metronidazole Nalidixic acid Phenoxymethylpenicillin Tinidazole

Metronidazole Metronidazole must not be taken with alcohol due to the disulfiram-like reaction with alcohol.

A patient, who has a history of alcoholism, has been admitted into hospital after collapsing. Upon screening the patients drug chart, you suspect a potential interaction. Which of the following medications would you flag to the medical team as having a potential interaction? Amoxicillin 500 mg TDS Metronidazole 400 mg TDS Omeprazole 20 mg OD Paracetamol 1 g QDS Thiamine 100 mg OD

Metronidazole 400 mg TDS. A disulfiram-like reaction occurs when metronidazole is taken with alcohol.

Metronidazole is an antimicrobial drug with a high activity against anaerobic organisms. It is important that when taking this medication patients are advised to avoid alcohol until after treatment completion. What is the reason for advising patients to avoid alcohol when taking metronidazole? Alcohol should be avoided when taking any course of antibiotics. Alcohol renders the use of metronidazole ineffective. Alcohol reduces the efficacy of metronidazole. Metronidazole produces a disulfiram like reaction with alcohol. Alcohol increases the potency of metronidazole.

Metronidazole produces a disulfiram like reaction with alcohol. Metronidazole produces a disulfiram like reaction with alcohol resulting in facial flushing, nausea and vomiting.

A patient presents you with a dental prescription for metronidazole and wants to discuss the medication with you. He informs you it is the weekend and he wanted to have a few alcoholic drinks. What is the reason that alcohol must be avoided when this patient is taking his course of metronidazole? Alcohol reduces the efficacy of metronidazole. Alcohol renders the use of metronidazole ineffective. Alcohol should be avoided when taking any course of antibiotics. Metronidazole produces a disulfiram like reaction with alcohol. Metronidazole toxicity can occur with alcohol.

Metronidazole produces a disulfiram like reaction with alcohol. Metronidazole produces a disulfiram like reaction with alcohol. This can result in facial flushing, dizziness, nausea and vomiting.

A patient experiencing an epileptic seizure is brought into Accident and Emergency for treatment. What is the most appropriate treatment for patients to be prescribed for treatment of status epilepticus should they experience this in their day-to-day life? Clobazam tablets Clonazepam tablets Diazepam intravenously Diazepam rectally Midazolam buccally

Midazolam buccally. Diazepam IV would be unusual for epilepsy with rectal diazepam being preferred as it is a safer and rapidly acting option. However, in day-to-day life this may be difficult to administer. Midazolam buccal would be the drug of choice in practice as it is also well absorbed. It is placed between the gum and the teeth along the side of the tongue and may be split so that a dose is given into each cheek to avoid accidental swallowing.

For each of the following patients, select the most likely option for their presentations. A 28-year-old male has suffered from several episodes of a type of headache throughout the year. He describes it as a throbbing unilateral headache which lasts 3 hours and is associated with photophobia, phonophobia and vomiting. Cluster headache Epilepsy Meningitis Migraine Rebound headache Sinus headache Tension headache Vertigo

Migraine His signs are typical of a migraine. The patient could also experience an aura prior to the start of the headache, although this does not occur in some patients.

Mrs Jones presents to the pharmacy complaining of headaches. She tells you that the pain has been present for the last 24 hours and is at the front of her head, and she has felt a little sick. Based on this information what would be the most likely diagnosis? Haematoma Migraine Sinusitis Temporal arteritis Tension-type headache

Migraine. Unilateral headache associated with sickness in women is highly suggestive of migraine. Haematoma can cause sickness but the duration of symptoms is too short for this to be considered. Other conditions listed do not have sickness as a symptom, although temporal arteritis can cause malaise.

A 14-year-old boy and his mum ask for your advice as he has a sore mouth. A visual inspection reveals two small circular lesions on the inside of the gums. He says they are painful. Based on this information, what is the most likely diagnosis? Leukoplakia Lichen planus Major aphthous ulcers Minor aphthous ulcers Trauma-related ulcers

Minor aphthous ulcers. Leuoplakia and lichen planus are seen as patches rather than circular lesions; major ulcers are large; and trauma ulcers are irregular shaped.

For each of the following statements, select the drug that fits most closely. This drug is a potent uterine stimulant and may be teratogenic. Desmopressin Diazepam Flucloxacillin Lamotrogine Misoprostol Paroxetine Salbutamol Vigabatrin

Misoprostol

Which of the following antisecretory and mucosal protectant drugs is a potent uterine stimulant? Cimetidine Misoprostol Lansoprazole Ranitidine Tripotassium dicitratobismuthate

Misoprostol

Which of the following antisecretory and mucosal protectant drugs may cause severe diarrhoea which may require withdrawal of this drug? Cimetidine Misoprostol Lansoprazole Ranitidine Tripotassium dicitratobismuthate

Misoprostol Diarrhoea may occasionally be severe and require withdrawal, reduced by giving single doses not exceeding 200 micrograms and by avoiding magnesium-containing antacids.

For each of the following questions, select the drug which is correct in relation to the scenario. This medication is used to terminate pregnancy following mifepristone administration. Sildenafil Dapoxetine Oxytocin Carbetocin Misoprostol Clindamycin Clotrimazole Estriol

Misoprostol Misoprostol is indicated for the termination of pregnancy following mifepristone. Only in those up to 49 days pregnant.

You are supporting a hospital outpatient service for patients with Parkinson's disease. Which of the following is the most appropriate treatment option for daytime sleepiness in Parkinson's patients? Amantadine Domperidone Levodopa Midodrine Modafinil

Modafinil If reversible pharmacological and physical causes have been excluded, modafinil should be considered to treat excessive daytime sleepiness, and treatment should be reviewed at least every 12 months.

Mrs Sloane, a 61-year-old woman, presents with a bright red left eye. Which symptom cluster associated with this presentation do you feel most likely warrants referral to an optician? Moderate pain with slight discharge and vision unaffected. Slight eye discomfort but no discharge with vision unaffected. Slight pain with little discharge and blurred vision due to tears. Slight pain with little discharge and vision unaffected. Slight pain with mucopurulent discharge and vision unaffected.

Moderate pain with slight discharge and vision unaffected. True eye pain should be referred for further evaluation but pain is subjective. 'Slight' eye pain is, in most cases, manageable in the pharmacy. In this case only option A warrants referral.

Miss C is joining your pharmacy team as a work experience student. You receive the morning order of medications and preparations and she is keen to help put them out on the pharmacy shelves. She asks you which one of the following substances is a prescription only medicine? Bisacodyl 10 mg suppositories Guaifenesin liquid Mometasone furoate 0.1% cream Dimeticone cutaneous solution Mepyramine maleate 2% cream

Mometasone furoate 0.1% cream

Which of the following statements regarding side effects is correct? Digoxin causes alopecia. Amiodarone causes pin point pupils. Propranolol causes hypothermia. Ferrous sulphate causes raised intraocular pressure. Morphine causes dry mouth.

Morphine causes dry mouth Dry mouth is a common side effect of morphine.

Mr N is worried about the abdominal pain, nausea and vomiting he has recently been suffering from. You know he is usually prescribed sodium valproate for the management of his epilepsy. What is the most appropriate advice you should give Mr N? There is no connection between his medication and the symptoms he's describing, you could suggest over the counter symptomatic relief. These are normal side effects of the medication, nothing to worry about. Seek immediate medical attention. Advise Mr N to go to his GP, as the dose of sodium valproate might need to be increased. Advice Mr N to go to his GP, as the dose of sodium valproate might need to be decreased.

Mr N should seek immediate medical attention, as the symptoms he describes may be indicative of sodium valproate-induced pancreatitis.

A patient has recently been diagnosed with tuberculosis and recently started on isoniazid as part of his treatment regimen. He has also been prescribed pyridoxine. He has been prescribed pyridoxine to reduce a common adverse effect of isoniazid. Which adverse effect can be prevented by the prophylactic use of pyridoxine? Agranulocytosis Nausea Neuropathy QT prolongation Skin rash

Neuropathy. A common side effect of isoniazid is peripheral neuropathy and the BNF monograph for pyridoxine includes a dose for prevention of isoniazid-induced neuropathy (10-20 mg daily).

A 27-year-old pregnant woman comes into the pharmacy to purchase some nicotine replacement therapy (NRT) to help her to quit smoking. She is otherwise healthy, with no other medical conditions, and she is not taking any other medication. She is, however, suffering from pregnancy-related nausea and vomiting. Which one of the following is the most suitable formulation of NRT for this patient? Nicotine gum Nicotine lozenge Nicotine mouth spray Nicotine nasal spray Nicotine patch

Nicotine patch. Normally intermittent therapy recommended for pregnant women e.g gum, lozenge etc, but with morning sickness a patch is more appropriate - taken off before bed and put on first thing in morning.

Your dispenser is measuring out the daily methadone oral liquid prescriptions. As she returns the bottle to the cupboard she drops it and it smashes on the floor. What is the most appropriate course of action for documenting the spillage? It does not need to be recorded. Make a note in the CD register. Make a record in the POM register. Make a record on the patient's PMR. Notify your authorised person and annotate the CD register.

Notify your authorised person and annotate the CD register. The authorised person should be aware so that they can observe destruction. As this may take time it would be appropriate to annotate the CD register to account for the balance discrepancy.

Mrs XR is a 25-year-old, weighing 50 kg and has type 1 diabetes. She is admitted to hospital with diabetic ketoacidosis. Mrs XR is currently taking NovoRapid® FlexPen 10 units three times daily and Levemir® FlexPen 15 units at night and is allergic to Tazocin®. She is also breastfeeding her 6-month-old baby. Which one of the following medicines is NOT routinely used in the management of diabetic ketoacidosis? Sodium chloride 0.9% Potassium chloride adjusted according to plasma-potassium concentration NovoMix® mixed with sodium chloride 0.9% Actrapid® mixed with sodium chloride 0.9% Insulin detemir

NovoMix® mixed with sodium chloride 0.9% Soluble insulin should be diluted (and mixed thoroughly) with sodium chloride 0.9% intravenous infusion to a concentration of 1 unit/mL; infuse at a fixed rate of 0.1 units/kg/hour. Novomix is not a soluble insulin

Mr Q, a 21 year old who weighs 55 kg, is admitted to hospital seriously ill with diabetic ketoacidosis. Mr Q is diagnosed with type 1 diabetes. The doctor decides that Mr Q will be prescribed a 'basal bolus regimen'. What is the most appropriate regimen for this patient? Humalog (insulin lispro) three times daily plus Novorapid (insulin aspart) once daily. Actrapid (insulin soluble) three times daily plus Novomix 30 (insulin aspart) once daily. Novorapid (insulin aspart) three times daily plus Lantus (insulin glargine) once daily. Humalog Mix25 (insulin lispro) twice daily. Novorapid (insulin aspart) three times daily plus Actrapid (insulin soluble) once daily.

Novorapid (insulin aspart) three times daily plus Lantus (insulin glargine) once daily

Ms VL is taking regular prednisolone. Which of the following is NOT a recognised side effect of long-term corticosteroid usage? Weight gain Glaucoma Osteoporosis Nystagmus Diabetes

Nystagmus is not a recognised side effect of long-term corticosteroid use.

Mrs J, a 29-year-old school teacher presents with extremely dry cracked hands. She is well hydrated, however, she tells you that she constantly cleans the surfaces of her house. Which one of the following is the most likely cause of these effects? Acute stress disorder Agoraphobia Obsessive-compulsive disorder Panic disorder Social phobia

Obsessive-compulsive disorder. Persistent thoughts, impulses or images (obsessions) that are intrusive and cause distress. The person attempts to get rid of these obsessions by completing repetitive time-consuming actions (compulsions). Common obsessions include contamination while the compulsion may be repetitive washing or cleaning.

Ms Bridges, who is 28 years old, asks for a cough medicine for her chesty cough. From the following symptoms, which one is most likely to make you refer the patient to the GP? Associated frontal headaches. Cough present for 10 days. Failure of symptoms to respond to guaifenesin. Occasional episodes of shortness of breath. Sputum that is clear in colour.

Occasional episodes of shortness of breath. Associated headaches with cough is a relatively common symptom; duration is classed as acute until 3 weeks has elapsed; failure to respond to treatment doesn't necessarily require referral but reassessment of symptoms would be advisable; clear sputum colour is not a referral point. Shortness of breath however needs further evaluation.

A patient presents in your community pharmacy stating that they think they are suffering from a side effect of amlodipine which they started take for the first time last week. What is a common side effect of amlodipine? Cold extremities Dry mouth Oedema Rash Tremor

Oedema is common side effect of amlodipine.

Mr T, is a 29 year-old technician who comes into your pharmacy to ask for advice. He tells you that he is having difficulty hearing people. He has seen his doctor who told him that he has a build up of wax in his ear canals. He tells you that he is not allergic to any medications but is allergic to almonds. He is otherwise well. Which one of the following is the suitable treatment option for Mr T's complaint? Cotton buds to be used to remove the wax, Almond oil ear drops. EarCalm® (acetic acid 2%) spray. Gentamicin and hydrocortisone ear drops. Olive oil ear drops.

Olive oil ear drops NHS choices recommend olive oil ear drops or almond oil ear drops for ear wax build up. The patient is allergic to almonds so almond oil would not be appropriate.

Which of the drugs below requires the following counselling points according to the BNF? Patients should be advised to report any unexplained bleeding, bruising and sore throat. Rotigotine Alendronic acid Olsalazine sodium Duraphat® Chlorhexidine Pancreatin Felodipine Amiodarone

Olsalazine sodium

Ritonavir is a protease Inhibitor prescribed for use in HIV. Which of the following drugs will NOT interact with ritonavir? Omeprazole Repaglinide Tacrolimus Tadalafil Zopiclon

Omeprazole

You are a hospital pharmacist responsible for monitoring the results of patients taking clozaril. How often should the following patient's white blood cell count be monitored? A patient newly initiated on clozaril. Daily Once weekly Every 2 weeks Every 3 weeks Every 4 weeks Every 8 weeks Every 12 weeks Yearly

Once weekly In the UK, a white cell count with a differential count must be monitored: At least weekly for the first 18 weeks of treatment At least at 2 week intervals between weeks 18 and 52 After 1 year of treatment with stable neutrophil counts, patients may be monitored at least at 4 week intervals Monitoring must continue throughout treatment and for at least 4 weeks after discontinuation

You receive a prescription for a schedule 3 CD in tablet form. Which of the following is a legally appropriate wording for the directions on the prescription? As directed As per chart Every day One PRN Weekly

One PRN For a direction to be legally appropriate it must contain the number of doses to be taken/used.

A 55-year-old man presents his prescription for oxycodone 20 mg m/r tablets. Which of the following directions would be legally acceptable on a prescription for this medicine? As directed Decrease dose by one every fortnight One as directed PRN Weekly

One as directed As a minimum the directions for a control drug must include the quantity of the medicine to be taken at each dose

You receive a prescription for a schedule 2 CD in tablet form. Which of the following is a legally appropriate wording for the directions on the prescription? As directed Every day One tablet as per chart PRN When required

One tablet as per chart. For a direction to be legally appropriate it must contain the number of doses to be taken/used.

A 20-year-old woman is going on holiday to Spain. Which of the following would be suitable advice to give her to help prevent her contracting travellers' diarrhoea? Avoid eating peelable fruits. Drink tap water only. Eat plenty of fresh salad. Only consume pasteurised milk. Uncooked vegetables are ideal for consumption.

Only consume pasteurised milk. All fruits that can be peeled e.g. bananas are ideal for consumption as they are less likely to carry the bacteria that could lead to travellers' diarrhoea. Uncooked vegetables and fresh salads have a higher chance of carrying bacteria that could lead to travellers' diarrhoea. Tap water in many countries poses the same risk of bacteria, thus bottled mineral water is recommended for holiday makers. Pasteurised milk is safer to drink than unpasteurised milk as there is a lower risk of bacteria thriving in the milk and causing infective diarrhoea.

You are the Responsible Pharmacist for a community pharmacy. For each of the scenarios select the correct response. You are on the bus which is stuck in traffic and you will be late. Your pharmacy has an electronic responsible pharmacist register that allows you to log in remotely. Considering that you feel confident to log in remotely, what is the correct operating procedure to tell your staff when contacting them? - Close the pharmacy and only compliance aids can be made up - Normal service, the dispenser and ACT can dispense prescriptions - Open the pharmacy and sell GSL and P medicines and shop products - Open the pharmacy and only methadone can be handed out - Open the pharmacy and only sell shop products (e.g. cosmetics) - Open the pharmacy and sell GSL medicines and shop products - Open the pharmacy and sell GSL medicines, shop products and hand out prescriptions already dispensed - The pharmacy should be closed

Open the pharmacy and sell GSL medicines and shop products Suitably trained staff can sell GSL medicines in the absence of a responsible pharmacist.

You are the Responsible Pharmacist for a community pharmacy. For each of the scenarios select the correct response. You need to visit a GP surgery for 30 minutes in the middle of the day. You signed in as the responsible pharmacist at 9am and will remain signed in during your absence. What is the correct operating procedure to tell your staff? - Close the pharmacy and only compliance aids can be made up - Normal service, the dispenser and ACT can dispense prescriptions - Open the pharmacy and sell GSL and P medicines and shop products - Open the pharmacy and only methadone can be handed out - Open the pharmacy and only sell shop products (e.g. cosmetics) - Open the pharmacy and sell GSL medicines and shop products - Open the pharmacy and sell GSL medicines, shop products and hand out prescriptions already dispensed - The pharmacy should be closed

Open the pharmacy and sell GSL medicines and shop products Suitably trained staff can sell GSL medicines in the absence of a responsible pharmacist.

For each of the following scenarios, select the prescriber which is correct in relation to the scenario. This prescriber would be able to prescribe a treatment for a subconjunctival haematoma but not for tinea corporis. Community practitioner nurse Dentist registered in the UK Doctor registered in the UK EEA or Swiss doctor or dentist Nurse independent prescriber Optometrist independent prescriber Pharmacist independent prescriber Supplementary prescriber

Optometrist independent prescriber A subconjunctival haematoma is an ocular condition and tinea corporis is a dermatological condition. An optometrist independent prescriber would be able to prescribe treatment for the ocular condition but not the dermatological condition.

For each of the following scenarios, select the prescriber which is correct in relation to the scenario. These prescribers are restricted to only prescribing medicines which are for ocular use and peri-orbital use. Community practitioner nurse Dentist registered in the UK Doctor registered in the UK EEA or Swiss doctor or dentist Nurse independent prescriber Optometrist independent prescriber Pharmacist independent prescriber Supplementary prescriber

Optometrist independent prescriber, Ocular and peri-orbital medications relate specifically to optometrists.

Mrs GX suffers from frequent migraines. Which of the following drugs on her PMR is known to precipitate migraine? GI motility stimulants Oral contraceptives Benzodiazepines Beta-blockers NSAIDs

Oral contraceptives

Herpes infection which presents near the oral region is a common complaint encountered by pharmacy staff working in the community. Which of the following types of herpes infection presents near the oral region? Cytomegalovirus Epstein-Barr virus Human herpesevirus-8 Herpes labialis Varicella-zoster

Oral herpes is known as herpes labialis.

Oramorph® oral solution contains 2 mg of morphine sulfate per 1 mL. Oramorph® concentrated oral solution contains 20 mg of morphine sulfate per 1 mL. What is the volume of Oramorph® oral solution that contains the same amount of morphine sulfate as 25 mL of Oramorph® concentrated oral solution? 0.4 mL 4 mL 20 mL 250 mL 590 mL

Oramorph® concentrated oral solution contains morphine sulfate 20 mg/mL = 100 mg/5 mL = 500 mg per 25 mL. Oramorph® oral solution contains morphine sulfate 2 mg/mL = 10 mg/5 mL = 10 mg per 5 mL = 50 mg per 25 mL = 500 mg per 250 mL.

A pregnant woman comes in to your pharmacy. She tells you that her friend has told her that 'she should check if she has had a rubella vaccination and if she is immune to rubella'. What can a rubella infection cause in pregnant patients who DON'T have immunity to rubella? - Develop flu-like symptoms, such as sore throat and swollen glands - Future infertility in the woman - Lead to premature labour - Nothing - Organ damage to the foetus during the early stages of the pregnancy

Organ damage to the foetus during the early stages of the pregnancy. The mother passes the virus on the foetus and it damages their organs as they develop. This is known as congenital rubella syndrome (CRS). This includes the eyes (e.g. cataracts), ears (hearing loss), heart and brain. This could affect the child throughout their life.

Which of the following terms is defined as shortness of breath on lying down? Apnoea Dysphagia Dysplasia Dyspraxia Orthopnea

Orthopnea. Orthopnea is defined as shortness of breath (dyspnoea) when lying down flat. Apnoea is cessation of breathing during sleep. Dysphagia is difficulty in swallowing.Dysplasia is abnormal cell growth. Dyspraxia is a developmental disorder.

Mr G is a 78-year-old retired school teacher. He has arthritis and is experiencing severe discomfort in his movements. Which one of the following allied healthcare professionals is the most suitable to assess and support Mr G with his discomfort and could provide the patient with the support aids to assist him? Dietitian Occupational therapist Orthoptist Orthotist Support worker

Orthotist. Orthotists provide support aids to correct problems or deformities in people's nerves, muscles or bones. Examples include splints, braces and special footwear (orthotics).

A patient is prescribed oral prednisolone for polymyalgia rheumatica. What is the greatest long-term risk with continued use of glucocorticoids? Hypernatraemia Hypotension Hypokalaemia Osteoporosis Weight loss

Osteoporosis. Corticosteroids can cause weight gain and an increase in blood pressure. Osteoporosis is the main long-term risk.

Mr K, a 40-year-old man, enters the pharmacy and wants some advice about a sore ear he has had for the last 3 days. He describes having some local itching and pain. What is the most likely diagnosis? Local inflammation of the pinna Meniere's disease Otitis externa Otitis media Perichondritis

Otitis externa. Perichondritis should exhibit outer ear structure swelling; otitis media presents with pain but no itching; Meniere's disease exhibits dizziness. This leaves A and C as the more likely options, but local inflammation is unlikely to present with itching.

Which drug combination is present in the following products? Targinact®. Ramipril and amlodipine Tegafur, gimeracil and oteracil Tegafur and gimeracil Calcium and vitamin D3 Alendronic acid and colecalciferol Ramipril and felodipine Oxycodone and naloxone Morphine and naloxone

Oxycodone and naloxone

For each of the following questions, select the drug which is correct in relation to the scenario. A junior doctor calls you asking for advice. He has a medical emergency and wishes to induce labour in a patient for medical reasons and wishes to know what drug is most appropriate. Sildenafil Dapoxetine Oxytocin Carbetocin Misoprostol Clindamycin Clotrimazole Estriol

Oxytocin Oxytocin is indicated for the induction of labour for medical reasons.

You are the regular pharmacist at a pharmacy. Mr WK who is a regular customer, comes into your pharmacy with his new born baby. He has brought in a prescription for senna tablets and lactulose solution for his wife who is at home resting. You ask him how his wife and he are managing with the new baby. He says that he is fine but his wife is still in a lot of pain and discomfort. She suffered with gestational diabetes so is still required to monitor her blood glucose levels. He wants you to recommend a really strong painkiller. He informs you that his wife is breast-feeding. What would be the SAFEST analgesic to recommend for Mr WK's wife? Panadol® original tablets Nurofen® plus Co-codamol 8/500 Paracetamol and ibuprofen together Anadin® Extra

Panadol® original tablets Aspirin is not recommended due to the risk of Reye's syndrome. Codeine should not be given to breastfeeding women because mother's vary considerably in their capacity to metabolise codeine and there is a risk or morphine overdose in the infant. Ibuprofen should be used with caution. Paracetamol is the simple analgesic of choice in breastfeeding women because the amount excreted in milk is too small to be harmful.

Which of the drugs below requires the following counselling points according to the BNF? It is important to ensure adequate hydration at all times in patients receiving higher-strength preparations of this drug. Rotigotine Alendronic acid Olsalazine sodium Duraphat® Chlorhexidine Pancreatin Felodipine Amiodarone

Pancreatin

For each of the following patients, select the single most likely cause behind their complaint. Mrs J is a 26-year-old shop assistant who has been admitted into hospital after screaming hysterically in a public area. Upon questioning, her doctor finds out that she has frequent recurrent 20 minute episodes where she feels that she is going to die, her heart rate increases, she has chest pain and difficulty breathing. She is not on any medications and is otherwise well. Acute stress disorder Agoraphobia Depression Epilepsy Generalised anxiety disorder Panic disorder Post-traumatic stress disorder Social phobia

Panic disorder Panic disorder presents with physical and psychological signs and symptoms. It is characterised by recurrent, severe panic attacks and behavioural changes associated with these. References: Clinical Pharmacy and Therapeutics, By Roger Walker.

You a running a pharmacy practice workshop helping to educate pharmacy technicians on the legalities of medicines. You receive a question regarding POM drugs. Which of the following would be classified as a POM? Aspirin 75 mg pack size of 16 tablets Aspirin 300 mg pack size of 16 tablets CoCodamol 8 mg/500 mg pack size of 32 tablets Paracetamol 500 mg pack size of 32 tablets Paracetamol 500 mg pack size of 112 tablets

Paracetamol 500 mg pack size of 112 tablets. Paracetamol is most commonly sold as a GSL medicine and P medicine. However, if a packet contains more than 100 regular paracetamol tablets this is then classified as a POM medicine and is subject to prescription requirements.

For each question in this section select one answer from the list of eight options above it. Adult patients who weigh less than 50 kg should use this drug with caution or adjust the dose as they may be at increased risk of toxicity. Aspirin 75 mg tablets Clotrimazole 1% cream Guaifenesin 100 mg/5 mL & Levomenthol 1.1 mg/5 mL oral solution Loratidine 10 mg tablets Paracetamol 500 mg tablets Pholcodine 5 mg/5 mL linctus Sumatriptan succinate 50 mg tablets Xylometazoline 0.1% nasal spray

Paracetamol 500 mg tablets As per BNF, patients weighing <50 kg should be monitored for signs of toxicity if using paracetamol and the max daily dose may be reduced to 500 mg qds to help prevent toxicity.

For each question in this section select one answer from the list of eight options above it. A patient presents to you complaining of an array of symptoms including achy joints and describes flu like symptoms and pyrexia. What would you recommend the patient takes to help alleviate the symptoms? Aspirin 75 mg tablets Clotrimazole 1% cream Guaifenesin 100 mg/5 mL & Levomenthol 1.1 mg/5 mL oral solution Loratidine 10 mg tablets Paracetamol 500 mg tablets Pholcodine 5 mg/5 mL linctus Sumatriptan succinate 50 mg tablets Xylometazoline 0.1% nasal spray

Paracetamol 500 mg tablets Paracetamol has analgesic and anti-pyretic properties.

Mrs D is a 75 year-old patient who was diagnosed with osteoarthritis two months ago. Which of the following medications would be suitable for long-term pain-relief? Ketoprofen Ibuprofen Meloxicam Naproxen Paracetamol

Paracetamol should be used first for pain relief in osteoarthritis and soft-tissue disorders. Given that this is a chronic condition, the use of NSAID's (the other options) long term can result in an increased risk of gastro-intestinal bleeding and ulcers. NSAID's could be used for the short-term relief of pain in osteoarthritis.

A patient comes into your pharmacy complaining of a very tender big toe. He discloses that he has asthma. You suspect that it might be gout and refer him to his GP. To help alleviate his pain while he waits for a GP appointment, what would be the most appropriate pain relief for you to recommend with the information you have been provided? Arnica tablets Diclofenac gel Ibuprofen gel Ibuprofen tablets Paracetamol tablets

Paracetamol tablets. Paracetamol tablets are the most appropriate from these choices as there is a risk of bronchospasm with NSAIDs in asthmatics. Along with the paracetamol tablets, referral to the GP is necessary as paracetamol is a temporary measure.

You are a hospital pharmacist who is discussing treatment options for different patients with the healthcare team. Select the single most suitable treatment option for each of the following cases. Each option may be used once, more than once, or not at all. Mr Q, a 21-year-old student, has a mild headache following a sports injury. He presents with no other signs or symptoms. Acamprosate Buprenorphine Disulfiram Fluoxetine Haloperidol Paracetamol Parenteral thiamine Pimozide

Paracetamol. Paracetamol is suitable for mild-to-moderate pain.

You are a hospital pharmacist who is discussing treatment options for different patients with the healthcare team. Select the single most suitable treatment option for each of the following cases. Mr S, a 46-year-old security guard, presents with acute confusion, leg tremors and droopy eyelids. He recently been enrolled on an alcohol dependence programme but states that it's a 'waste of time'. Acamprosate Buprenorphine Disulfiram Fluoxetine Haloperidol Paracetamol Parenteral thiamine Pimozide

Parenteral thiamine Mr S is experiencing effects caused by acute, severe thiamine deficiency, secondary to alcoholism.

You receive a prescription for haloperidol 500 mcg tablets for one of your patients. Your PMR system flags that haloperidol is contraindicated for this patient. Which one of the following medical conditions would mean that haloperidol is contraindicated for this patient? Diabetes Depression Parkinson's disease Schizophrenia Tuberculosis

Parkinson's disease The symptoms of Parkinson's may be precipitated by haloperidol.

For each of the following statements, select the drug that fits most closely. This drug can be used in the treatment of post-traumatic stress disorder. Desmopressin Diazepam Flucloxacillin Lamotrigine Misoprostol Paroxetine Salbutamol Vigabatrin

Paroxetine

For each of the following statements, select the drug that fits most closely. This drug can be used to help treat symptoms such as depression and anxiety. Desmopressin Diazepam Flucloxacillin Lamotrigine Misoprostol Paroxetine Salbutamol Vigabatrin

Paroxetine

Mrs J is usually prescribed insulin for her diabetes, as well as warfarin and digoxin for atrial fibrillation. She complains of an increased frequency of urination over the past few days, nausea and vomiting and a headache. She compliments you on your yellow shirt, which is in fact white. What is the most appropriate advice you should give Mrs J? This should resolve over the next few days. You should suggest over the counter products for a possible urinary tract infection. Mrs J should make an appointment with an optometrist as she may be suffering from cataracts. Patient should be referred as osmotic diuresis is a sign of uncontrolled diabetes. Patient should be referred as she is exhibiting signs of digoxin toxicity and uncontrolled diabetes.

Patient should be referred as she is exhibiting signs of digoxin toxicity and uncontrolled diabetes. Osmotic diuresis (a sign of uncontrolled diabetes) is associated with hypokalaemia, which would result in digoxin toxicity (which is presenting in this case as nausea and vomiting, headache and xanthopsia).

Mr K is a 21-year-old student who comes to your pharmacy with a new prescription for himself. You look at the prescription and advise Mr K to report immediately any sore throat, fever, infection, unexplained bleeding and bruising, purpura, mouth ulcers or rashes to their doctor. Which of the following drugs is this counselling advice most suitable for? Bezafibrate Glucosamine Loperamide Penicillamine Sumatriptan

Penicillamine. Penicillamine has been associated with blood disorders. Therefore, patients should be counselled to report these signs immediately to their doctor, if and when they develop.

A patient presents in your pharmacy with a prescription. The patient has been prescribed penicillin V for tonsillitis. She asks whether she will be able to continue breastfeeding her 5-month-old baby whilst she is taking the antibiotic. What is the best advice for the patient? Penicillin V is considered compatible for breastfeeding but the infant should be monitored. She should avoid feeding the baby for 45 minutes after taking the medicine because concentrations in the blood are highest then. She should stop breastfeeding until after the course is complete. The patient should be referred to her health visitor. The treatment is not compatible with breastfeeding and the patient should be referred back to her GP

Penicillin V is considered compatible for breastfeeding but the infant should be monitored. Penicillin V is considered compatible for breastfeeding but the infant should be monitored for adverse effects.

The Misuse of Drugs Regulations 2001 (as amended) classify controlled drugs into 5 schedules according to the different levels of control attributed to each. Which schedule includes pethidine? Schedule 1 Schedule 2 Schedule 3 Schedule 4 Schedule 5

Pethidine is a Schedule 2 CD.

Depressive illnesses can be treated using anti-depressive medications. There are several classes of anti-depressive medications available for this condition. One such class is the selective serotonin re-uptake inhibitors or the SSRIs. Which of the following anti-depressive medications is NOT an SSRI? Citalopram Escitalopram Fluoxetine Phenelzine Fluvoxamine Maleate

Phenelzine. Phenelzine is a monoamine-oxidase inhibitor or an MAOI.

Using the information provided, determine which ONE of the following drugs should NOT be given to a lady in her first trimester of pregnancy? Phenindione Flecainide Rifampicin Chlorphenamine Metformin

Phenindione

Which ONE of the following can cause urine to be coloured pink or orange? Warfarin Rivaroxaban Dabigatran Apixaban Phenindione

Phenindione. An adverse effect of phenindione is that it can colour urine pink/orange.

For each of the following statements, select the drug which is the most applicable. Patients of Chinese ancestry are more likely to be associated with developing Stevens-Johnson syndrome. Amiodarone Clozapine Digoxin Gentamicin Lithium Phenytoin Theophylline Warfarin

Phenytoin Evidence (although limited) suggests that HLA-B* 1502 may be a risk factor for the development of SJS/TEN in patients of Asian ancestry taking drugs associated with SJS/TEN, including phenytoin.

For each of the following statements, select the drug which is the most applicable. A small increase in the dose can produce a dramatic change in the plasma levels of this drug. Amiodarone Clozapine Digoxin Gentamicin Lithium Phenytoin Theophylline Warfarin

Phenytoin Phenytoin is approximately 90% bound to plasma proteins, and metabolised in the liver. At low concentrations, elimination is proportional to the concentration. However, as the concentration rises to therapeutic levels, the enzyme system in the liver can become saturated and elimination occurs at a constant rate despite the dose (zero order kinetics). The effect is that a small increase in the dose sometimes causes a large change in the level of free phenytoin in the blood.

Antiepileptic drugs have been divided into three risk-based categories to help healthcare professionals to determine whether a patient needs to be maintained on a specific manufacturer's product. Which one of the following antiepileptic drugs is a patient most likely to need to be maintained on a specific manufacturer's product? Brivaracetam Ethosuximide Gabapentin Phenytoin Pregabalin

Phenytoin. Phenytoin is classed as a category 1 antiepileptic. For these drugs, doctors are advised to ensure that their patient is maintained on a specific manufacturer's product. The other antiepileptic options are all category 3 drugs. For these drugs, it is usually unnecessary to ensure that a patient is maintained on a specific brand.

For each question in this section select one answer from the list of eight options above it. This medication is cautioned for use in those who suffer with asthma. This drug is also a Schedule 5 controlled substance. Aspirin 75 mg tablets Clotrimazole 1% cream Guaifenesin 100 mg/5 mL & Levomenthol 1.1 mg/5 mL oral solution Loratidine 10 mg tablets Paracetamol 500 mg tablets Pholcodine 5 mg/5 mL linctus Sumatriptan succinate 50 mg tablets Xylometazoline 0.1% nasal spray

Pholcodine 5 mg/5 mL linctus Pholcodine is a Schedule 5 CD. It is cautioned for use in asthma.

Which of the listed vitamins and minerals should be prescribed to the following patients? A patient suffering from moderate to severe hypophosphatemia would require an infusion of this. Vitamin E Vitamin K Vitamin A Vitamin B12 Calcium Vitamin B1 Phosphate Vitamin B6

Phosphate

A patient on warfarin has had a recent decrease in their INR test result. What is the most likely cause of this? Clopidogrel Disulfiram Fluconazole Phytomenadione Sodium valproate

Phytomenadione. Phytomenadione is vitamin K which antagonises the vitamin K antagonist warfarin.

You are giving a seminar to accredited checking technicians working in your pharmacy on serious drug interactions. Select the most likely drug combination from the list which results in the serious effects described below. This combination causes ventricular arrhythmias. Digoxin and quinine Digoxin and cefalexin Ketoprofen and paracetamol Sildenafil and isosorbide mononitrate Atenolol and ibuprofen Clonidine and propranolol Pimozide and ketoconazole Metronidazole and cimetidine

Pimozide and ketoconazole. This combination is very dangerous as pimozide is a CYP3A4 enzyme substrate and ketoconazole is a potent inhibitor of CYP3A4 leading to a marked increase in pimozide serum levels. This can cause ventricular arrhythmias.

You have been asked to recommend a beta-blocker for a patient who usually suffers from cold extremities. Which of the following beta-blockers is the most appropriate for the patient concerned? Atenolol Carvedilol Labetalol Pindolol Propranolol

Pindolol ISA and pindolol is the least cardioselective.

A patient has recently been admitted onto your ward with a gastrointestinal bleed. Upon doing a drug history, you find that the patient is on some medication. Which of these drugs is most likely to have caused the GI bleed? Atenolol 50 mg OD. Bendroflumethiazide 2.5 mg OD. Fluoxetine 20 mg OM. Isosorbide mononitrate 10 mg BD. Piroxicam 20 mg ON.

Piroxicam 20 mg ON. NSAIDs can cause serious gastrointestinal events including bleeding, ulceration and perforation of the stomach, small intestine or large intestine, which can be fatal. These serious adverse events can occur at any time, with or without warning symptoms. Piroxicam is particularly gastro-toxic.

Which of the following terms describes 'asbestos-like' scales on the scalp? Pityriasis alba Pityriasis amiantacea Pityriasis rosea Pityriasis versicolor Psoriasis

Pityriasis amiantacea. Pityriasis amiantacea describes 'asbestos-like' scales on the scalp.

For each of the following scenarios, select the drug which is correct in relation to the scenario. This medication is commonly used for prevention of migraine. Benzydamine hydrochloride Dalteparin Duloxetine Isosorbide mononitrate Isotretinoin Pizotifen Timolol 0.25% eye drops Xylometazoline hydrochloride 0.1%

Pizotifen Pizotifen is indicated for the prophylaxis of migraine and is a POM medication.

A patient returns a bag of items to you. Select the most appropriate way to dispose of them. Paracetamol 500 mg tablets bought over the counter Denature and dispose of in the designated medicines 'Doop' disposal bin Denature and pour down the sink Down the toilet In the normal waste bin In the wholesaler returns tray Place in the designated medicines disposal 'Doop' bin Place in the sharps bin Put in the confidential waste bin

Place in the designated medicines disposal 'Doop' bin Paracetamol tablets should be disposed of in a medicines disposal bin, how they were purchased does not change the disposal process.

A patient returns a bag of items to you. Select the most appropriate way to dispose of them. Paracetamol tablets 500 mg that have been dispensed Denature and dispose of in the designated medicines 'Doop' disposal bin Denature and pour down the sink Down the toilet In the normal waste bin In the wholesaler returns tray Place in the designated medicines disposal 'Doop' bin Place in the sharps bin Put in the confidential waste bin

Place in the designated medicines disposal 'Doop' bin Paracetamol tablets should be disposed of in a medicines disposal bin, how they were purchased does not change the disposal process.

A patient returns a bag of items to you. Select the most appropriate way to dispose of them. A box of unopened insulin pen needles Denature and dispose of in the designated medicines 'Doop' disposal bin Denature and pour down the sink Down the toilet In the normal waste bin In the wholesaler returns tray Place in the designated medicines disposal 'Doop' bin Place in the sharps bin Put in the confidential waste bin

Place in the sharps bin Sharps should be disposed of appropriately.

In the morning delivery at the pharmacy you receive the following products. What would be the most appropriate way, with regards to legal requirements or Home Office recommendations, to process each of the products in this delivery? Morphine sulfate oral solution 10 mg/5 mL. Display the product behind the counter Immediately place in the CD cupboard Place on the shelves in the dispensary Place the product on the shelves in the shop Record the delivery in the CD register Record the delivery in the prescription-only medicine (POM) register Store in the fridge not visible from the shop Store in the fridge visible from the shop

Place on the shelves in the dispensary Morphine sulfate oral solution 10 mg/5 mL is a schedule 5 CD. The requirement to record in the CD register or keep in the CD cupboard do not apply.

In the morning delivery at the pharmacy you receive the following products. What would be the most appropriate way, with regards to legal requirements or Home Office recommendations, to process each of the products in this delivery? Tramadol capsules 50 mg. Display the product behind the counter Immediately place in the CD cupboard Place on the shelves in the dispensary Place the product on the shelves in the shop Record the delivery in the CD register Record the delivery in the prescription-only medicine (POM) register Store in the fridge not visible from the shop Store in the fridge visible from the shop

Place on the shelves in the dispensary Tramadol is a schedule 3 CD exempt from safe custody requirements.

A person whose spleen was removed following a road traffic accident is seeing their general practitioner. Which vaccine is indicated for this patient? Hepatitis B vaccine Hepatitis A vaccine Pneumococcal vaccine Rabies vaccine Typhoid vaccine

Pneumococcal vaccine If they have had a splenectomy then they are at an increased risk of pneumococcal infection.

For each of the following patients, select the single most likely cause behind their complaint. Mr L is a 37-year-old soldier who is unable to relax since returning from a conflict zone 3 months ago. He is irritable, has difficulty concentrating and falling asleep and feels helpless. He vividly recalls a traumatic experience where he witnessed a close friend's death in a forest. He now avoids visits to the park. Acute stress disorder Agoraphobia Depression Epilepsy Generalised anxiety disorder Panic disorder Post-traumatic stress disorder Social phobia

Post-traumatic stress disorder Post-traumatic stress disorder usually occurs after exposure to a traumatic event which involved actual or threatened death, or serious injury or threats to the physical integrity of self or others. The person responds with intense fear, helplessness or horror. Sufferers can re-experience symptoms (flashbacks) and avoid situations associated with the trauma. This condition usually occurs within 6 months of the traumatic event.

Which one of the following medicines can increase blood glucose levels in a diabetic patient? Methotrexate Prednisolone Diclofenac Folic Acid Paracetamol

Prednisolone

Different medications require different prescription requirements. You are running a session for medical students regarding the legalities of prescription drugs. A student asks a question regarding the requirements of atenolol. Which of the following is NOT a prescription requirement for a non-electronic private prescription for atenolol 25 mg tablets? If repeatable, the number of times the supply can be dispensed Name and address of patient Prescribers address Prescribers signature in indelible ink Prescription dated within 3 months

Prescription dated within 3 months. Atenolol is a standard POM. The prescription requirements for POMs state that the prescription must be within date to be dispensed. For standard POMs a prescription must be dated within 6 months to be supplied.

For the patients described, select the most likely drug from the list below which may have caused the stated side-effect. Akathisia in a 17-year-old girl. Ciclesonide Ranitidine Sucralfate Docusate sodium Prochlorperazine Felodipine Glyceryl trinitrate Carbocisteine

Prochlorperazine Akathisia (restlessness) is an extrapyramidal side-effect which is commonly associated with high initial doses of piperazine phenothiazines like prochlorperazine.

A patient is initiated on a medication for which they will need to take extra precautions to protect their skin from sunlight whilst taking it. Which of the following drugs when given at higher doses does this advice apply to? Indapamide Lithium carbonate Pizotifen Promazine Ramipril

Promazine. Promazine makes the skin more susceptible to the UV rays from the sun. As such, a patient on promazine therapy is more susceptible to burning/sun burn and must take precaution in sunlight. The BNF entry for promazine lists a specific advice warning on photosenitivity whereas the BNF entry for indapamide does not, and that is why it was chosen in this instance.

Mr G is a 45-year-old mechanic who has angina. His GP contacts you to discuss the options of beta-blockers to prescribe to Mr G. He is looking for a once daily dosing regimen. Which of the following beta-blockers is LESS suitable for a once daily regime? Atenolol Bisoprolol fumarate Celiprolol hydrochloride Nadolol Propanolol

Propanolol. Some beta blockers, such as atenolol, bisoprolol fumarate, celiprolol hydrochloride, and nadolol, have an intrinsically longer duration of action and need to be given only once daily.

For each of the following patients, select the most appropriate antidote. A 40-year old woman has accidentally given herself double her dose of Clexane® (enoxaparin hydrochloride) injection. She does not have any other medical conditions and has no known allergies. Acetylcysteine Aspirin Digoxin-specific antibody European viper snake venom antiserum Flumazenil Methaemoglobinaemia Naloxone hydrochloride Protamine sulfate

Protamine sulfate The antidote for heparin toxicity is protamine sulfate.

In order for a patient to demonstrate capacity, they do NOT need to be able to: Understand the information provided Use and weigh up the information provided Prove they are aged 18 and over Remember the information provided Communicate their decision to you (by any means)

Prove they are aged 18 and over

A patient presents to the community pharmacy in which you are the Responsible Pharmacist. Upon discussing with the patient you ascertain that she is 28 weeks pregnant and suffers with severe hypertension. Which of the following medications is NOT appropriate to sell OTC to this patient? Glycerol suppositories Paracetamol tablets Pseudoephedrine tablets Saline nasal spray Sodium docusate capsules

Pseudoephedrine tablets Pseudoephedrine is contraindicated in patients suffering with severe hypertension.

Mr PR, 22 years old, comes into your pharmacy seeking advice about a white/silver rash on his arm. He tells you that it started 10 days ago and has increased in size since and does not bother him too much, although it is mildly itchy occasionally. He has no other symptoms and suffers from no other medical conditions. What is the most likely cause of his symptoms? Atopic dermatitis Contact dermatitis Discoid eczema Psoriasis Tinea corporis

Psoriasis. Concentrating on mild itch, then contact and atopic dermatitis and discoid eczema can be eliminated as they have itch as a prominent symptom. Tinea corporis is also itchy and varies from mild to severe. Psoriasis is the correct answer as it tends to be a condition that does not cause itch or is sometimes mildly itchy.

Warts are a common skin condition seen in children. Which symptom best describe warts? Raised crusty lesions Raised hyperkeratotic papules Raised papules with central dimples Raised smooth papules Raised waxy lesions

Raised hyperkeratotic papules. All options have a descriptor of 'raised' and hence is of no value. Warts are however associated with excessive cell turnover and results in hypertrophy; thus the answer is B. D could be a host of skin problems as the descriptor is very broad; C could be molluscum contagiosum because of the 'central dimple' reference; likewise E could be seborrhoeic warts as they are associated with waxy-like lesions; finally A could suggest cancer-like growths.

Which one of the following medicines is NOT likely to cause bradycardia during treatment? Bisoprolol Adizem® SR Ivabradine Amiodarone Ramipril

Ramipril

For each of the following statements, select the drug which is correct. This drug inhibits the conversion of angiotensin 1 to angiotensin 2 and can cause a cough as a side effect. Apixaban Aspirin Atenolol Digoxin Lacidipine Methyldopa Ramipril Urokinase

Ramipril It is a commonly known side effect of ACE inhibitors that they can cause a dry cough. Ramipril is the only option which is an ACE inhibitor.

Which drug combination is present in the following products? Triapin®. Ramipril and amlodipine Tegafur, gimeracil and oteracil Tegafur and gimeracil Calcium and vitamin D3 Alendronic acid and colecalciferol Ramipril and felodipine Oxycodone and naloxone Morphine and naloxone

Ramipril and felodipine

For each of the following statements, select the drug which is correct in relation to the scenario. This medication is used at a continuing dose of 1 mg daily in the management of Parkinson's disease. Trihexyphenidyl Procyclidine hydrochloride Orphenadrine hydrochloride Tolcapone Amantadine hydrochloride Selegiline hydrochloride Entacapone Rasagiline

Rasagiline Rasagiline is used at a dose of 1 mg daily in the management of Parkinson's disease. It is either used alone or as an add on therapy with either co-beneldopa or co-careldopa.

Mr J is a 19-year-old patient who mentions his concerns regarding, sore and itchy skin between his toes. You inspect his feet and confirm that he has athlete's foot. What is the most appropriate advice you should give Mr J? - No need to have an intervention, it will resolve by itself in time. - Recommend clotrimazole cream 1% applied twice daily and continuing for two weeks after the infection has cleared. - Recommend clotrimazole cream 2% applied once daily and continuing for one day after the infection has cleared. - Recommend hydrogen peroxide 1% (Crystacide®) cream applied twice daily for two weeks. - Recommend aqueous cream to moisturise between the toes.

Recommend clotrimazole cream 1% applied twice daily and continuing for two weeks after the infection has cleared. Clotrimazole 1% cream is licensed for use in athletes foot. Note that clotrimazole 2% is licensed for external treatment of vaginal thrush.

You are working on a hospital ward when you receive a query from a junior pharmacist. The junior pharmacist is seeking advice regarding the blood glucose levels of a diabetic patient. The patients blood glucose level is reading as 3.5 mmol/L. What would the best advice to give the junior pharmacist regarding treatment for the patient? - Allow the patient bed rest as this blood glucose reading is not concerning - Inform the junior doctor that the patient will need fluid and recommend that they prescribe 1 litre of sodium chloride 0.9% - Inform the junior doctor that the patient will need 1 mg of glucagon STAT - Inform the junior doctor that the patient will require a 20% glucose IV infusion - Recommend that the ward nursing staff give the patient a glucogel STAT

Recommend that the ward nursing staff give the patient a glucogel STAT. Initially 10-20 grams of glucose is given by mouth for treatment of a mild hypo. This can be in the form of a glucogel.

A patient presents in your pharmacy and asks you for advice. When he was cleaning his teeth this morning, he noticed bleeding from his gums. He is taking no medicines and is otherwise healthy. What is the best course of action? Advise the patient to stop brushing his teeth for 48 hours. Recommend the patient brush his teeth four times daily. Recommend the patient floss between his teeth. Tell the patient he needs to see a dentist urgently. Tell the patient he needs to see his GP.

Recommend the patient floss between his teeth Flossing between teeth to clean this area is advised to help with bleeding gums.

You are the on duty pharmacist in a community pharmacy. Miss JA, who is 17 years old, presents in your pharmacy enquiring about a rash she has developed. You note from her patient medication record that you recently dispensed amoxicillin capsules for her. She tells you that this was for tonsillitis. Her patient medication record suggests she has received amoxicillin before but she states that she has never had a rash like this before now. What is the most suitable course of action? Refer Miss JA to her dentist. Refer Miss JA to her GP for further investigation and/or treatment. Refer Miss JA to her GP urgently. Sell Miss JA a throat lozenge. Send Miss JA to the Accident and Emergency department.

Refer Miss JA to her GP for further investigation and/or treatment. Miss JA's symptoms suggest she may have glandular fever and the GP should investigate this further. Patients with glandular fever may experience a rash when given amoxicillin.

You are the pharmacist on duty in a community pharmacy. Mr DA comes to collect his regular prescription for carbimazole from the pharmacy. He enquires about a treatment for his sore throat. What is the most suitable course of action? Refer Mr DA to his dentist Refer Mr DA to his GP for further investigation and/or treatment Refer Mr DA to his GP urgently Sell Mr DA a throat lozenge Send Mr DA to the Accident and Emergency department

Refer Mr DA to his GP urgently. In this case the sore throat could be a sign of bone marrow suppression induced by carbimazole. The patient would need to stop treatment promptly under GP supervision.

Mr J is a 32-year-old male who comes to collect his prescription. As you hand over his medicines, you notice that he looks poorly and has a runny nose. He tells you that he is feverish, feels shaky and has a strong headache. -carbimazole 15 mg o.d. -ramipril 20 mg o.n. -amlodipine 10 mg o.d. Given that Mr J is taking the medications below and that he has not travelled anywhere recently, which one of the following is the most suitable course of action? Recommend OTC paracetamol and an anti-inflammatory. Recommend OTC paracetamol and an antihistamine. Refer Mr J to see a doctor immediately. Advise Mr J that what he is experiencing is a common cold and it will resolve given time. Recommend OTC paracetamol only

Refer Mr J to see a doctor immediately. Carbimazole may cause bone marrow suppression which presents as infection or flu-like symptoms (i.e. sore throat, fever and malaise). Therefore, patients taking this medication must be advised to immediately report to their doctor should they experience any of these symptoms.

You are the pharmacist on duty in a community pharmacy. Mr MC, a 72-year-old male visits your pharmacy complaining of a sore mouth and throat. He wears dentures but describes seeing white spots behind them when he removes them. His patient medication record contains the following medicines: Combivent® nebuliser solution - 2.5 mL (one vial) three or four times daily, Seretide 500 Accuhaler® - one inhalation twice daily, warfarin - according to book. Refer Mr MC to his dentist. Refer Mr MC to his GP for further investigation and/or treatment. Refer Mr MC to her GP urgently. Sell Mr MC a throat lozenge. Send Mr MC to the Accident and Emergency department

Refer Mr MC to his GP for further investigation and/or treatment. Mr MC's symptoms suggest he is suffering from oral thrush, which he is moresusceptible to if he does not rinse his mouth after using the Seretide inhaler. Although miconazole gel is available over the counter, this would not be appropriate in this situation due to the patient taking warfarin and he should be referred to his GP for further treatment.

You are the pharmacist on duty in a community pharmacy. Mrs AB visits the pharmacy with her four month old baby, who she is breastfeeding. She complains of a sore throat. She states she has swelling at the back of her mouth and what look like spots. She has no cough but has had a temperature. She asks what she can buy for her symptoms that is suitable while breastfeeding. What is the most suitable course of action? Refer Mrs AB to her dentist. Refer Mrs AB to her GP for further investigation and/or treatment. Refer Mrs AB to her GP urgently. Sell Mrs AB a throat lozenge. Send Mrs AB to the Accident and Emergency department

Refer Mrs AB to her GP for further investigation and/or treatment. Mrs AB's symptoms suggest she may have tonsillitis and she should be referred to the GP for treatment.

Mrs G is a 48-year-old self-employed woman who comes into your pharmacy for advice. She tells you that she is experiencing pain in her right ear. After further discussion you find that the pain started after she cleaned it with a cotton bud earlier today and that there is swelling and blood in the same ear. Which one of the following actions are the most suitable to take in this situation? Advise Mrs G to gently clean the blood and the pain will go naturally with time. Ask Mrs G to request ear syringing from her GP's practice nurse. Refer her to a doctor. Supply olive oil ear drops. Supply sodium bicarbonate ear drops.

Refer her to a doctor. Cotton ear buds may have caused injury in Mrs G's right ear. The use of cotton ear buds is not recommended as they push ear wax further down the ear canal and may also cause injury or infection.

Mrs H, a 59-year-old patient, enters your pharmacy complaining of "bloating and heartburn" that hasn't been relieved with a number of over the counter products for "some time now". She also adds that she's been sick several times over the past couple of days. She describes the appearance of her vomit as "ground coffee". What is the most appropriate advice you should give Mrs H? - Find out which over the counter products she has used and advice her of the other alternatives she can try - Check which medication(s) she's currently taking, as her symptoms may be a side effect of one or more of these - Give her lifestyle advice, such as "reduce trigger factors, such as alcohol, spicy or fatty foods" - Check if the patient has a history of smoking as this may cause dyspepsia and if she does, enrol her onto the smoking cessation programme you are running - Refer her to her GP for assessment

Refer her to her GP for assessment. Coffee ground vomit and weight loss are dangerous symptoms that require referral as further investigations would need to be carried out to exclude oesophageal/gastric carcinoma.

The patient has been suffering from a mouth ulcer for the past three weeks. It is not causing him any pain. What is the best course of action? Advise the patient to avoid brushing the area to give it time to recover. Advise the patient to take paracetamol for the pain. Refer the patient to a dentist. Sell the patient an analgesic mouthwash. Sell the patient a topical anaesthetic gel

Refer the patient to a dentist. Patients suffering with a mouth ulcer for more than 2 weeks should be referred to a dentist to exclude oral cancer.

For each of the following scenarios, select the most appropriate action to take. Mrs X is a regular patient at your pharmacy. She comes in today and tells you that she has a fever. On further inquiry, she also tells you that she has pain in her side and an increased frequency and urgency of urination. This is the second time that this has happened to Mrs X over the past 5 months. - Combine several OTC products for greater therapeutic potential - Exercise regularly and eat well - Nothing needs to be done. It will resolve by itself - Recommend an OTC product - Refer the patient to their GP - Try another OTC product for 1 week and if an improvement is not seen within this time then ask them to make an appointment with their GP - Try another OTC product for 2 weeks and if an improvement is not seen within this time then ask them to make an appointment with their GP - Try a herbal remedy

Refer the patient to their GP The symptoms described are those of cystitis. Referral to the GP is necessary if there are repeated episodes of cystitis. Referral is also advised in cases where there is haematuria, loin pain, fever, nausea and vomiting, as well as suspected diabetes. In this scenario, Mrs X has loin pain, fever and repeated episodes of cystitis.

For each of the following scenarios, select the most appropriate action to take. Mr X usually comes into your pharmacy to pick up his insulin pens. Today he tells you that he has a blister on his foot and would like your advice on which product he should purchase to remove the blister. - Combine several OTC products for greater therapeutic potential - Exercise regularly and eat well - Nothing needs to be done. It will resolve by itself - Recommend an OTC product - Refer the patient to their GP - Try another OTC product for 1 week and if an improvement is not seen within this time then ask them to make an appointment with their GP - Try another OTC product for 2 weeks and if an improvement is not seen within this time then ask them to make an appointment with their GP - Try a herbal remedy

Refer the patient to their GP This patient is a diabetic and therefore referral to a doctor is advised in cases of all foot problems.

A 15-year-old woman presents to you at a community pharmacy - she has a complaint of a thick white vaginal discharge. What is the most appropriate course of action in this instance? - Carry out a consultation with the patient and ascertain a recent sexual history to rule out an STI - Inform the patient that they must go to a sexual health clinic as symptoms are indicative of an STI - Refer the woman to her GP - Sell the woman an OTC pessary to use for treatment of vaginal thrush - Sell the woman clotrimazole 1% cream to alleviate external symptoms of vaginal thrush

Refer the woman to her GP. As the patient is under 16 years of age, she must be referred to her GP for treatment.

Mrs. JS is an asthmatic patient who regularly sees you for her asthma. She has come to see you today because her optician found that she had raised intraocular pressure when she went to have her glasses changed. You are an independent prescriber running respiratory clinics in a surgery, Mrs JS mentions she had raised intraocular pressure when she had her eyes tested. What would be the most appropriate course of action. Refer to GP Xalacom® (Latanoprost, Timolol) Refer back to optician Azarga® (Brinzolamide, Timolol) Trusopt® (Dorzolamide)

Refer to GP. As the patient has not stated that the optician has not been treating the raised intraocular pressure, the pharmacist would need to do something about it - the optician may not be the most appropriate person. Even though the pharmacist is an independent prescriber, their specialism is respiratory and professional guidelines state that you should only practise within areas in which you are competent. Therefore prescribing the treatments would not be appropriate either. The most professional course of action is to refer the patient to their GP.

A patient comes into your pharmacy complaining of a very tender big toe. You consider that it might be gout. Which OTC drug or action would be the most appropriate? Diclofenac gel. Ibuprofen gel. Arnica tablets. Paracetamol tablets. Refer to GP

Refer to GP. Best to refer to the GP as high dose NSAIDs are usually given and this is not a suitable recommendation for NSAID OTC sale.

You are working as an independent prescribing pharmacist in a GP surgery running a clinic for patients with asthma. What next step treatment option would you chose for the patient? Your patient reports having weekly asthma attacks as well as struggling with exercise. They are currently using salbutamol, beclometasone, salmeterol and montelukast at optimised doses. Add on an inhaled long-acting beta-2 agonist Add on an inhaled corticosteroid Add on leukotriene receptor agonist Add on tiotropium Increase dose of corticosteroid Increase frequency of corticosteroid Maintain current treatment Refer to specialist

Refer to specialist Patients who are still experiencing asthma symptoms despite the use of four therapies should be referred to a specialist.

For each of the following patients, select the most appropriate treatment option for the scenario described. Miss W is a 19-year-old patient who has developed several painful red lumps and patches (that measure 1-5 cm) on her lower legs. Betamethasone Clobetasone butyrate Griseofulvin Hydrocortisone Ketoconazole shampoo Miconazole Rest and ibuprofen Tetracycline

Rest and ibuprofen. These lumps are characteristic of erythema nodosum. Treatment is usually rest and painkillers such as NSAIDs.

With regards to the processing of prescriptions/orders in the pharmacy, how must the following be processed? A veterinary prescription for a Schedule 2 CD. Retained for one year Retained for two years Retained for five years Retained for six years Returned to the patient Returned to the prescriber Submitted to the local accountable officer Submitted to the relevant NHS agency

Retained for five years Veterinary prescriptions for Schedule 2 CDs should be retained for 5 years, there is no requirement to submit them to the relevant NHS agency.

With regards to the processing of prescriptions/orders in the pharmacy, how must the following be processed? A veterinary prescription for a Schedule 3 CD Retained for one year Retained for two years Retained for five years Retained for six years Returned to the patient Returned to the prescriber Submitted to the local accountable officer Submitted to the relevant NHS agency

Retained for five years Veterinary prescriptions for Schedule 3 CDs should be retained for 5 years, there is no requirement to submit them to the relevant NHS agency.

With regards to the processing of prescriptions/orders in the pharmacy, how must the following be processed? A requisition for a Schedule 2 CD from a veterinarian. Retained for one year Retained for two years Retained for five years Retained for six years Returned to the patient Returned to the prescriber Submitted to the local accountable officer Submitted to the relevant NHS agency

Retained for five years Veterinary requisitions for Schedule 2 CDs should be retained for 5 years, there is no requirement to submit them to the relevant NHS agency.

With regards to the processing of prescriptions/orders in the pharmacy, how must the following be processed? A requisition for a Schedule 3 CD from a veterinarian. Retained for one year Retained for two years Retained for five years Retained for six years Returned to the patient Returned to the prescriber Submitted to the local accountable officer Submitted to the relevant NHS agency

Retained for five years Veterinary requisitions for Schedule 3 CDs should be retained for 5 years, there is no requirement to submit them to the relevant NHS agency.

With regards to the processing of prescriptions/orders in the pharmacy, how must the following be processed? An invoice for a Schedule 3 CD taking into account tax requirements. Retained for one year Retained for two years Retained for five years Retained for six years Returned to the patient Returned to the prescriber Submitted to the local accountable officer Submitted to the relevant NHS agency

Retained for six years Although the law relating to CDs requires the invoices to be retained for 2 years (as per the BNF advice), the law relating to tax requires the invoices to be kept for 6 years (as per SPS advice).

For each of the following questions, select the vitamin or mineral which is the correct answer. You should advise pregnant patients to avoid liver products due to the fact they contain this. Ascorbic acid Calcium Folic acid Iron Retinol Vitamin D Vitamin K Zinc

Retinol Retinol is vitamin A which is found in large volumes in liver products, which can be harmful to the unborn baby.

In your pharmacy there is a dispensing error leading to a patient safety incident. What is the most appropriate technique to identify the cause of the dispensing error? Clinical audit. Root cause analysis. SMART criteria. SWOT analysis. Systematic review.

Root cause analysis A root cause analysis should be carried out in the event of a patient safety incident.

You are the pharmacist explaining to a pharmacy summer placement student which medicines are not suitable for use in children, including those in their mother's womb. Select the reason why this drug should not be used in children. Aspirin for a 6-year-old child to relieve a fever. Bone marrow toxicity Damage to skeletal development Discolouration of teeth Drug toxicity Reye's syndrome Stunted growth The medicine is teratogenic The risk of using the medicine outweighs the benefits

Reye's syndrome Aspirin is associated with Reye's syndrome in children and should not be given to those under 16 years of age.

There are several drugs which may interact with the oral contraceptive Yasmin. Which of the following drugs reduces the effectiveness of Yasmin? Naloxone Paracetamol Ranolazine Rifabutin Selegiline

Rifabutin Rifabutin is predicted to decrease the efficacy of combined oral contraceptives.

Which drug should NOT be given to patients with acute porphyria? Phenindione Flecainide Rifampicin Chlorphenamine Metformin

Rifampicin

For each of the following statements, select the drug that fits most closely. Use of this drug may cause the urine of the patient to turn reddish-brown in colour. Alendronate Disulfiram Isoniazid Isotretinoin Ketoconazole Lithium Rifampicin Sulfasalazine

Rifampicin Rifampicin commonly colours the urine of patients using it.

Patrick is 58 years old and has tuberculosis. What is the initial phase treatment in conjunction with ethambutol, pyrazinamide, isoniazid ?

Rifampicin Use of 4 drugs for 2 months and then Rifampicin and isoniazid + pyridoxine for 4 months after.

You are working on a hospital ward and prior to discharge you are counselling a patient on their medication and its side effects. Which of the following can cause a patients urine to turn red-orange? Azithromycin Ciclosporin Mycophenolate Rifampicin Sodium valproate

Rifampicin. Approximately 30% of rifampicin is excreted in the urine. It turns the urine an orange-red colour. This is a counselling point that must be reiterated to patients.

A patient is admitted into hospital with newly diagnosed tuberculosis. The patient has also been on warfarin for atrial fibrillation for the past 6 years. Which of the following medicines may reduce the effect of the patient's regular warfarin? Ethambutol Isoniazid Omeprazole Pyrazinamide Rifampicin

Rifampicin. Rifampicin is a potent enzyme inducer and therefore will increase the metabolism of certain drugs metabolised by the same enzyme, such as warfarin.

A patient calls into the community pharmacy in which you are working, complaining that the soft contact lenses they wear keep becoming discoloured. The patient wants to know if any of the medications they are taking may be a causing this. Which of the patients' medications is likely to be causing the discolouration of the contact lenses? Amiloride Azithromycin Codeine Finastaride Rifampicin

Rifampicin. Rifampicin is known to discolour soft contact lenses.

You are trying to prevent prescribing errors in your pharmacy. Which of the following is NOT a recommended check point for avoiding prescribing errors? Right dose Right medicine Right patient Right quantity Right route

Right quantity. Double checking when prescribing or administering any medicine is important to avoid any medication errors. You can double check it is the: right medicine, right patient, right dose, right route, right time.

You are trying to prevent prescribing errors in your pharmacy. Which of the following is NOT an MHRA recommended check point for avoiding prescribing errors? Right dose Right medicine Right patient Right strength Right time

Right strength. Double checking when prescribing or administering any medicines is important to avoid any medication errors. You can double check it is the: right medicine, right patient, right dose, right route, right time.

You have just finished using a stock bottle of methadone in the pharmacy. You are unable to pour anything out of the bottle, but you can see that there are still traces inside. In what way should you deal with the disposal of the remaining liquid inside the bottle? Arrange for a specialist disposal company to dispose of the bottle and contents. Place the bottle and remaining contents in the dupe bin. Pour the remaining contents into the dupe bin, Rinse the bottle with water and pour down the sink. Rinse the bottle with water and pour into a denaturing kit.

Rinse the bottle with water and pour into a denaturing kit. Waste medicines should not be disposed of in the sink. The rinsings should be added to a denaturing kit and disposed as pharmaceutical waste.

For each of the following patients, select the most relevant safety concern associated with the patient's medication. A 37-year-old woman prescribed insulin from a pen device. metformin 500 mg twice daily aspirin 75 mg daily amlodipine 10 mg daily She has been newly prescribed pioglitazone 15 mg once daily. Atypical femoral fractures Benign idiopathic osteonecrosis Fatal diabetic ketoacidosis Increased incidence of heart failure Risk of severe harm and death Subacute cutaneous lupus erythematosus Suicidal ideation Uterine perforation

Risk of severe harm and death MHRA has issued a safety alert for increased risk of severe harm and death due to withdrawing insulin from pen devices.

There are a variety of different drugs used in the treatment of HIV, one such class is protease inhibitors. Which of the following drugs is a protease inhibitor? Azathioprine Ciclosporin Ketamine Rifampicin Ritonavir

Ritonavir is a protease inhibitor used in the treatment of HIV.

You are a pharmacist working in a GP surgery creating a prescribing guideline for anticoagulant drugs. Which direct-acting oral anticoagulant is likely to be recommended by NICE for prevention of adverse outcomes after acute management of ACS with raised biomarkers? Apixaban Dabigatran Edoxaban Rivaroxaban Warfarin

Rivaroxaban is the only DOAC licensed for that indication and is recommended by NICE. Warfarin is not a DOAC.

You are completing some CPD on the use of anticoagulant medicines. Which direct-acting oral anticoagulant should be taken with food? Apixaban Dabigatran Edoxaban Rivaroxaban Warfarin

Rivaroxaban. Rivaroxaban should be taken with food due to lack of efficacy on an empty stomach.

You are supporting a hospital outpatient service for patients with Parkinson's disease. Impulse control disorders are most likely a side effect of which of the following medicines? Domperidone Levodopa Midodrine Modafinil Ropinirole

Ropinirole. Impulse control disorders (compulsive gambling, hypersexuality, binge eating, or obsessive shopping) can develop in a person with Parkinson's disease who is on any dopaminergic therapy at any stage in the disease. Levodopa is also associated with impulse control disorders, however the answer reference given states: 'Conversely, excessive sleepiness, hallucinations, and impulse control disorders are more likely to occur with dopamine-receptor agonists than with levodopa.' Thus ropinirole is the best answer.

Which of the drugs below requires the following counselling points according to the BNF? Care should be taken when driving or operating machinery as this drug is associated with hypotensive reactions and a sudden onset of sleep. Rotigotine Alendronic acid Olsalazine sodium Duraphat® Chlorhexidine Pancreatin Felodipine Amiodarone

Rotigotine

Mrs S is a 26-year-old female who comes into your pharmacy to ask for your advice. Her doctor has asked her to book an appointment to receive MMR vaccination. She tells you that she is planning to become pregnant and is worried about having this vaccination. Which one of the following is the most suitable response you can give Mrs S? Rubella should be given to individuals who do not have immunity to the condition, including woman of child-bearing age. Avoid pregnancy for at least 1 month after vaccination. Don't take the vaccination it is not necessary. Delay taking the vaccine until after you become pregnant and have given birth. No vaccine affects pregnancy. Ask your doctor for an alternative vaccination

Rubella should be given to individuals who do not have immunity to the condition, including woman of child-bearing age. Avoid pregnancy for at least 1 month after vaccination

For each of the following scenarios, choose the appropriate treatment for the patient. A patient newly diagnosed with COPD who needs treatment to relieve breathlessness and exercise limitation. ICS LABA LABA + ICS LABA + LAMA LAMA LAMA + LABA + ICS SABA SAMA

SABA or SAMA Use short-acting bronchodilators, as necessary, as the initial empirical treatment to relieve breathlessness and exercise limitation.

You are training to become an independent prescriber with the aim of specialising in diabetes. Choose the most appropriate next treatment step for each of the patients described. A patient who has been taking metformin for 5 months with a HbA1c of 7.6%, who is also obese. Address adherence Basal insulin DDP-4 inhibitor GLP-1 agonist Metformin SGLT2 inhibitor Soluble insulin Sulphonylurea

SGLT2 inhibitor In a patient not reaching their target who is obese, a SGLT2 inhibitor would be an appropriate treatment option.

You are training to become an independent prescriber with the aim of specialising in diabetes. Choose the most appropriate next treatment step for each of the patients described. A patient who has been taking metformin for 4 months with a HbA1c of 7.3%, and who has cardiovascular risk factors. Address adherence Basal insulin DDP-4 inhibitor GLP-1 agonist Metformin SGLT2 inhibitor Soluble insulin Sulphonylurea

SGLT2 inhibitor In a patient not reaching their target with cardiovascular risk factors, a SGLT2 inhibitor would be an appropriate treatment option.

You are updating your pharmacy SOPs with regards to the management and dispensing of pregabalin. Which of the following is NOT a requirement for pregabalin? Dose on prescription Drug formulation on prescription Drug strength on prescription Safe custody Total quantity in words and figures on prescription

Safe custody. Pregabalin and gabapentin will be included in the list of "exempted drugs" in the safe custody regulations which means that they will not be required to be kept in the CD cabinet.

For each of the following statements, select the drug that fits most closely. The overuse of this drug may result in potentially serious hypokalaemia. Desmopressin Diazepam Flucloxacillin Lamotrogine Misoprostol Paroxetine Salbutamol Vigabatrin

Salbutamol Salbutamol may cause hypokalaemia, especially if overused.

When looking at a patient's urea and electrolytes levels, you notice that the potassium level is 2.7 mmol/L. Which of the patient's regular medications could cause this low potassium level? Aspirin Atorvastatin Salbutamol nebs Lactulose Soluble paracetamol

Salbutamol nebs. Hypokalaemia may result from beta2agonist therapy.

For each of the following statements, select the drug which is correct in relation to the scenario. Early treatment with this drug can delay the need for levodopa therapy in Parkinson's disease. Trihexyphenidyl Procyclidine hydrochloride Orphenadrine hydrochloride Tolcapone Amantadine hydrochloride Selegiline hydrochloride Entacapone Rasagiline

Selegiline hydrochloride Early treatment with selegiline can delay the need for levodopa therapy.

For each of the following statements, select the drug which is correct in relation to the scenario. This drug may cause hair loss as a side effect. Trihexyphenidyl Procyclidine hydrochloride Orphenadrine hydrochloride Tolcapone Amantadine hydrochloride Selegiline hydrochloride Entacapone Rasagiline

Selegiline hydrochloride Hair loss is a side effect of treatment with selegiline hydrochloride.

You are the on duty pharmacist in a community pharmacy. Mr WC, who is 40 years old, comes into your pharmacy complaining of having a sore throat and a cough. His glands are not swollen and although it is a little uncomfortable to swallow there is no pain upon swallowing. He takes no other medicines. What is the most suitable course of action? Refer Mr WC to his dentist. Refer Mr WC to his GP for further investigation and/or treatment. Refer Mr WC to his GP urgently. Sell Mr WC a throat lozenge. Send Mr WC to the Accident and Emergency department.

Sell Mr WC a throat lozenge. Mr WC's symptoms do not currently indicate an infection and he is on no other medicines which a sore throat could be a side effect of. Something to sooth the pain may be helpful in this instance and Mr WC should be advised to seek further advice if his symptoms persist for longer than a week, get worse or if he gets swelling.

A patient presents in your pharmacy for advice. She gave birth recently and is suffering from constipation. She is asking for advice on treatment. She is breastfeeding and had an uncomplicated delivery. She has tried altering her diet. What is the best option for this lady? Refer her to her GP as you cannot sell a treatment over the counter because she is breastfeeding Sell her ispaghula husk Sell her lactulose Sell her loperamide Sell her senna

Sell her ispaghula husk. A bulk forming laxative is the first-line treatment for constipation in breastfeeding women. It is suitable for sale over the counter.

A patient presents in your pharmacy for advice. She is suffering with headaches and would like to take a painkiller. She is currently breastfeeding her three month old baby and does not want to stop. Which of the following is the most suitable course of action? Advise that she will need to stop breastfeeding to take any painkiller. Refer her to her GP for further advice. Sell her aspirin. Sell her co-codamol. Sell her paracetamol

Sell her paracetamol. Paracetamol is the painkiller of choice in breastfeeding women.

You are working a Sunday morning shift in a high street pharmacy in England. A mother approaches you and asks for a spacer for her five year old son because his has broken. What would be the most appropriate course of action? Advise the mother to ask her GP for a prescription on the Monday. Demonstrate how to use an inhaler without a spacer. Make an emergency supply. Sell the mother a spacer. Send the mother to an out of hours service for a prescription.

Sell the mother a spacer. A spacer is a medical device not a medicine. Therefore there are no restrictions on the sale of the spacer. The clinical nature of asthma means an attack could happen at any point, the patient does not have to be wheezing to even make an emergency supply of a salbutamol inhaler, so would not need to be wheezing to make a supply of a spacer. For the process of making an emergency supply an interview with the patient (the son) would have to be made and there is no indication that he is present. An emergency supply would also not avoid the need for the mother to pay for the spacer as there is no indication of a prescription coming to cover the supply.

A 36-year-old woman asks for OTC vaginal thrush treatment for her second outbreak in eight months. She is particularly complaining about vulval itchiness and soreness. What is the most appropriate course of action in this instance? Recommend that the woman visits her local sexual health clinic for a check up. Refer the woman to her GP Sell the sodium citrate granules that can be used to help with her vulval discomfort. Sell the woman clotrimazole 1% cream to alleviate external symptoms of vaginal thrush. Sell the woman hydrocortisone 1% cream to help reduce the vulval itchiness.

Sell the woman clotrimazole 1% cream to alleviate external symptoms of vaginal thrush. If the patient has more than 2 episodes of thrush in 6 months she should be referred to her GP. As this is not the case - she can be treated with clotrimazole 1% cream.

For each of the following scenarios or statements, select the drug which is correct. This laxative has an onset of action of 8 to 12 hours. Bisacodyl Co-danthramer Docusate sodium Glycerol Ispaghula husk Lactulose Methylcellulose Senna

Senna Senna has onset of action of 8 to 12 hours. Note that bisacodyl has an onset of action of 10-12 hours

Mrs A is 63 years old. She recently had a myocardial infarction and has been feeling depressed since. Her doctor has prescribed her a selective serotonin re-uptake inhibitor (SSRI). Which of the following SSRIs is the most safe for someone who has had a recent myocardial infarction? Citalopram Escitalopram Fluoxetine Paroxetine Sertraline

Sertraline. According to the BNF, SSRIs are considered first line in the treatment of depression, given their safety profile. Sertraline in particular has been shown to be safe for those patients with unstable angina or who have had a recent myocardial infarction.

Mr GU is a 67-year-old man, weighing 57 kg, admitted to hospital for the treatment of status epilepticus. His past medical history is myocardial infarction, depression and epilepsy. His hepatic and renal function are normal. The next day the ward pharmacist carries out a medicines reconciliation with Mr GU. Which medicine is most likely to be contributing to the reason for Mr GU's admission to hospital? Aspirin Bisoprolol Ramipril Isosorbide mononitrate modified release Sertraline

Sertraline. Sertraline is an SSRI and can lower seizure threshold.

Miss Q is a 27-year-old sales assistant, who is admitted into hospital following signs of infection. The doctor requests non-specific laboratory tests to identify the infectious disease. Which one of the following is LEAST likely to be used in the identification of an infectious disease? C-reactive protein Erythrocyte sedimentation rate Serum complement Serum ferritin White blood cell count

Serum ferritin Ferritin is an intracellular iron storage protein and a marker of iron stores.

Mrs K is a 62-year-old computer analyst who has been initiated on hormone replacement therapy (HRT). She has heard many rumours from her friends and is concerned about any serious effects which her new medication may cause. During your discussion, you mention that there are some instances when HRT should be stopped immediately. Which one of the following is a reason for Mrs K to stop her HRT immediately? Alopecia Exfoliative dermatitis Nausea Severe stomach pain Severe unexplained sore throat, fever or malaise

Severe stomach pain. Hormone replacement therapy should be stopped (pending investigation and treatment), if severe stomach pains occur.

A 36-year-old woman has been experiencing irregular periods, weight gain and hair loss in the past few months. Her GP has prescribed metformin 500 mg tablets which she has been taking for 7 weeks. She returns to the pharmacy and explains that she is experiencing diarrhoea. What is the most appropriate advice to give to the patient? - She is experiencing a side effect of metformin and can see her GP for a sustained-release formulation of metformin to help with this. - She is experiencing a side effect of metformin which is normally short-term. - She should reduce her dose of metformin to minimise the problem. - She should stop taking metformin and see her GP for further advice. - The symptom she describes is not known to be caused by metformin

She is experiencing a side effect of metformin and can see her GP for a sustained-release formulation of metformin to help with this Gastrointestinal disturbances are a common side effect of metformin and may be minimised by sustained-release formulations.

You are the pharmacist on duty in a community pharmacy. Miss BF presents to your pharmacy. She has just turned 17 and would like to give blood. She has seen on the NHS website that when people are on certain medicines they may not be able to give blood. She asks you about her medicine, isotretinoin. What is the most appropriate advice? She can give blood without any concerns. She can never give blood. She must not give blood until 4 weeks after the last dose of the medicine. She must not give blood until 8 weeks after the last dose of the medicine. She must not give blood until 2 years after the last dose of the medicine.

She must not give blood until 4 weeks after the last dose of the medicine Patients should not donate blood during therapy and for 1 month following discontinuation of isotretinoin because of the potential risk to the foetus of a pregnant transfusion recipient.

For each of the following scenarios, select the most likely condition. Mr D started to feel generally unwell a week ago and had a tingling feeling on the left side of his trunk associated with back pain. A band-like rash appeared in that same area a few days later . He described the rash as red and blotchy, developing into fluid filled blisters, which then dried out and formed scabs. Allergic reaction Chickenpox Eczema Impetigo Psoriasis Scabies Shingles Urticaria

Shingles The symptoms described are those of shingles. Note that if the rash appears on both sides of the body, it is unlikely to be shingles. Also although the rash associated with shingles usually appears on the trunk it could also affect other areas of the body, such as the face, eyes and genitals.

You are learning about the use of cytotoxic medicines. According to NPSA guidance, which of the following does a patient NOT need written information on when taking oral cytotoxic medicines? Intended drug regimen Monitoring arrangements Side effects they have experienced Their specialist's contact details Treatment plan

Side effects they have experienced. Patients should be fully informed and receive verbal and up-to-date written information about their oral anticancer therapy from the initiating hospital. This information should include contact details for specialist advice, which can be shared with non-specialist practitioners. Written information including details of the intended oral anti-cancer regimen and treatment plan including arrangements for monitoring, taken from the original protocol, should be given to the patient. When shared with pharmacists and dispensing staff, this would enable the above dispensing requirements to be satisfied.

Which one of the following drugs is suitable for use in children aged less than 6 years for the relief of coughs and colds? Diphenhydramine Dextromethorphan Xylometazoline Sodium chloride 0.9% nasal drops Chlorphenamine

Sodium chloride 0.9% nasal drops. The MHRA has concluded that some medicines for coughs and colds do not have any evidence which shows that they are effective and in many cases have been linked to undesirable effects including sleep disturbances and hallucinations. Chlorphenamine is licensed for use in children aged 1 year and above but not for use in coughs and colds.

For each of the following questions, select the drug which is correct in relation to the scenario. You are giving a training session on the treatment on acne vulgaris to some pre-registration trainees. A trainee asks which oral antibiotics are suitable for use in the treatment of acne vulgaris. Ceftazidime Ertapenem Tetracycline Amikacin Clarithromycin Sodium fusidate Co-trimoxazole Fidaxomicin

Sodium fusidate. Sodium fusidate is cautioned for use in biliary disease or biliary obstruction as elimination may be reduced leading to toxic effects.

For each of the following statements, select the drug that fits most closely. Before initiating treatment with this medication, a review of oral health should be carried out. Amoxicillin Baclofen Chloramphenicol Chlorpromazine Ciprofloxacin Fludrocortisone Sodium risedronate Tamsulosin hydrochloride

Sodium risedronate Bisphosphonates can cause osteonecrosis of the jaw. As such, prior to treatment an oral health check or review should be carried out.

For each of the following statements, select the drug that fits most closely. This drug may used to prevent osteoporosis. Amoxicillin Baclofen Chloramphenicol Chlorpromazine Ciprofloxacin Fludrocortisone Sodium risedronate Tamsulosin hydrochloride

Sodium risedronate Sodium risedronate is a bisphosphonate. Bisphosphonates are adsorbed onto hydroxyapatite crystals in bone, slowing both their rate of growth and dissolution, and therefore reducing the rate of bone turnover. This in turn can help prevent osteoporosis.

For each of the following statements, select the drug that fits most closely. This drug belongs to a group of non-hormonal medicines called bisphosphonates which are used to treat bone diseases. Amoxicillin Baclofen Chloramphenicol Chlorpromazine Ciprofloxacin Fludrocortisone Sodium risedronate Tamsulosin hydrochloride

Sodium risedronate Sodium risedronate is a bisphosphonate. Bisphosphonates are adsorbed onto hydroxyapatite crystals in bone, slowing both their rate of growth and dissolution, and therefore reducing the rate of bone turnover. This in turn can help prevent osteoporosis. References: BNF, Sodium risedronate monograph

A GP rings the community pharmacy you work in to ask for advice regarding declining renal function in a patient. Which of the following medications would not be recommended in a patient with a renal function of CrCl 27mLs/min? Acethylcholine chloride Isoniazid Sodium docusate Sodium hyaluronate Sodium risedronate

Sodium risedronate Sodium risedronate is a bisphosphonate. Bisphosphonates are routinely used in osteoporosis. This drug should not be used if renal function drops below CrCl 30 mLs/min.

Lithium salts have a narrow therapeutic index, therefore, serum lithium levels need to be closely monitored. Changes in dietary intake of electrolytes can affect serum lithium levels. Which of the following electrolytes is most likely to have an adverse effect on serum lithium levels? Calcium Magnesium Phosphate Potassium Sodium

Sodium. The sodium-lithium channels can't differentiate between sodium and lithium, therefore, changes in the diet that reduce or increase sodium intake affect serum lithium levels resulting in sub-therapeutic or toxic levels. Raised calcium levels also need to be monitored but have less impact that sodium. BNF states that patients should not do anything that affects their sodium levels.

A patient has the following prescription items from you on a monthly basis: Paracetamol 500 mg tablets 2 QDS, morphine sulfate 10 mg SR tablets 2 BD, lactulose 10 mL BD. The patient presents you with a prescription for two months' supply. What is the most appropriate course of action and the explanation for this course of action? Dispense the prescription as it is ok to do so. Do not dispense the prescription because it is not legal to supply more than 100 paracetamol. Do not dispense the prescription because it is not legal to supply two months' worth of morphine. Speak to the prescriber to clarify if there is a reason for giving the patient two months' supply. Tell the patient you'll give them half of the morphine now and half in a months time.

Speak to the prescriber to clarify if there is a reason for giving the patient two months' supply. There is no legal restriction on how many months supply of a CD can be prescribed; however, Home Office guidance says it should be restricted to 30 days. Therefore it is appropriate to clarify with the prescriber why two months' supply has been prescribed.

For each of the following patients with hypertension, select the treatment that is most likely to cause the adverse effect described. Mrs Q is a 49-year-old patient with resistant hypertension. She has noticed unusual vaginal bleeding. Azilsartan medoxomil Bendroflumethiazide Captopril Eprosartan Furosemide Indapamide Spironolactone Telmisartan

Spironolactone Menstrual disturbances, such as post-menopausal bleeding, are a side effect of spironolactone use.

Mrs P is a 27-year-old female who comes to collect her combined oral contraceptive pill, Microgynon® 30. As she leaves the pharmacy you notice that she takes time to look at your selection of herbal remedies. Which one of the following herbal remedies is most likely to reduce the effectiveness of Mrs P's contraceptive pill? Agnus Castus fruit extract tablets Echinacea drops Ginseng extract tablets St John's Wort extract tablets Valerian root extract tablets

St John's Wort extract tablets St John's Wort is a drug that induces hepatic enzyme activity. It interacts with the combined oral contraceptive pill and can considerably reduce its effectiveness by increasing its metabolism.

For each of the following patients, select the condition they are likely to be suffering from. A man asks you to look at his right eye. His upper eyelid is swollen. He describes it as being painful and sensitive to touch. Bacterial conjunctivitis Basel cell carcinoma Blepharitis Corneal ulcer Ectropion Stye Subconjunctival haemorrhage Viral conjunctivitis

Stye Indicative of a stye - painful swollen eye lid.

For each of the following descriptions, select the condition that is most accurate. Pain is occipital. Cluster headache Medication-overuse headache Meningitis Migraine Sinusitis Sub-arachnoid haemorrhage Temporal arteritis Tension-type headache

Sub-arachnoid haemorrhage Sinusitis, cluster headache, migraine and temporal arteritis are associated with frontal headache; meningitis, tension-type headache and medication overuse headache are more generalised in nature.

For each of the following patients, select the condition they are likely to be suffering from. A patient asks you to look at his left eye. You can see that a small segment of the white of his eye is bright red. He thinks it happened when he coughed. There is no pain and his vision is not affected. Bacterial conjunctivitis Basel cell carcinoma Blepharitis Corneal ulcer Ectropion Stye Subconjunctival haemorrhage Viral conjunctivitis

Subconjunctival haemorrhage Non-painful blood shot eye that came on suddenly is an indication of subconjunctival haemorrhage.

With regards to the processing of prescriptions/orders in the pharmacy, how must the following be processed? An NHS prescription for a Schedule 2 CD. Retained for one year Retained for two years Retained for five years Retained for six years Returned to the patient Returned to the prescriber Submitted to the local accountable officer Submitted to the relevant NHS agency

Submitted to the relevant NHS agency NHS prescriptions for Schedule 2 CDs should be sent to the relevant NHS agency.

With regards to the processing of prescriptions/orders in the pharmacy, how must the following be processed? An NHS prescription for a Schedule 3 CD. Retained for one year Retained for two years Retained for five years Retained for six years Returned to the patient Returned to the prescriber Submitted to the local accountable officer Submitted to the relevant NHS agency

Submitted to the relevant NHS agency NHS prescriptions for Schedule 3 CDs should be sent to the relevant NHS agency.

With regards to the processing of prescriptions/orders in the pharmacy, how must the following be processed? Private prescription for a Schedule 2 CD. Retained for one year Retained for two years Retained for five years Retained for six years Returned to the patient Returned to the prescriber Submitted to the local accountable officer Submitted to the relevant NHS agency

Submitted to the relevant NHS agency Private prescriptions for Schedule 2 CDs should be sent to the relevant NHS agency.

With regards to the processing of prescriptions/orders in the pharmacy, how must the following be processed? A private prescription for a Schedule 3 CD. Retained for one year Retained for two years Retained for five years Retained for six years Returned to the patient Returned to the prescriber Submitted to the local accountable officer Submitted to the relevant NHS agency

Submitted to the relevant NHS agency Private prescriptions for Schedule 3 CDs should be sent to the relevant NHS agency.

With regards to the processing of prescriptions/orders in the pharmacy, how must the following be processed? A requisition for a Schedule 3 CD from a registered medical practitioner. Retained for one year Retained for two years Retained for five years Retained for six years Returned to the patient Returned to the prescriber Submitted to the local accountable officer Submitted to the relevant NHS agency

Submitted to the relevant NHS agency Requisitions for Schedule 3 CDs should be sent to the relevant NHS agency.

With regards to the processing of prescriptions/orders in the pharmacy, how must the following be processed? A requisition for a Schedule 2 CD from a registered doctor. Retained for one year Retained for two years Retained for five years Retained for six years Returned to the patient Returned to the prescriber Submitted to the local accountable officer Submitted to the relevant NHS agency

Submitted to the relevant NHS agency Requisitions from doctors for Schedule 2 CDs should be sent to the relevant NHS agency.

Miss J is a 23-year-old female who has multiple sclerosis. She comes into your pharmacy on Saturday evening looking very concerned. After taking her into the consultation room, she tells you that she is beginning to see objects other people can't see and that she is sweating profusely. She tells you that she stayed over at her sister's house in Cornwall over the past 3 days and that she only took her glyceryl trinitrate spray with her. Given that Miss J takes the following medicines: Glyceryl trinitrate 400 microgram sublingual spray prn, Verapamil 120 mg t.d.s, Baclofen 20 mg t.d.s, Loperamide m.d.u, and that there are no other signs or symptoms, which one of the following is the most likely cause of her symptoms? A mosquito bite Travel sickness Food poisoning Sudden withdrawal of baclofen Sudden withdrawal of verapamil

Sudden withdrawal of baclofen. Serious life-threatening side-effects such as hallucinations and fever result from sudden withdrawal of baclofen therapy. Patients must be warned not to suddenly stop taking this medication.

For each of the following statements, select the drug that fits most closely. This drug is used in the treatment of ulcerative colitis. Alendronate Disulfiram Isoniazid Isotretinoin Ketoconazole Lithium Rifampicin Sulfasalazine

Sulfasalazine. Sulfasalazine is used in the treatment of bowel disorders including ulcerative colitis.

You are training to become an independent prescriber with the aim of specialising in diabetes. Choose the most appropriate next treatment step for each of the patients described. A patient who has been taking metformin for 4 months with a HbA1c of 7.2% with no other health concerns. Address adherence Basal insulin DDP-4 inhibitor GLP-1 agonist Metformin SGLT2 inhibitor Soluble insulin Sulphonylurea

Sulphonylurea In a patient not reaching their target who does not have any specific considerations, a sulphonylurea would be an appropriate medicine to consider prescribing.

For each question in this section select one answer from the list of eight options above it. This medication is most often prescribed by a doctor. If you are to sell this medication OTC to a patient, the indication must be confirmed first and the indication previously diagnosed by a qualified doctor. Aspirin 75 mg tablets Clotrimazole 1% cream Guaifenesin 100 mg/5 mL & Levomenthol 1.1 mg/5 mL oral solution Loratidine 10 mg tablets Paracetamol 500 mg tablets Pholcodine 5 mg/5 mL linctus Sumatriptan succinate 50 mg tablets Xylometazoline 0.1% nasal spray

Sumatriptan succinate 50 mg tablets

For each question in this section select one answer from the list of eight options above it. This medication must be used with caution in patients who have a sensitivity to sulfonamides. Aspirin 75 mg tablets Clotrimazole 1% cream Guaifenesin 100 mg/5 mL & Levomenthol 1.1 mg/5 mL oral solution Loratidine 10 mg tablets Paracetamol 500 mg tablets Pholcodine 5 mg/5 mL linctus Sumatriptan succinate 50 mg tablets Xylometazoline 0.1% nasal spray

Sumatriptan succinate 50 mg tablets Sumatriptan should be used with caution in patients with sensitivity to sulfonamides.

For each of the following scenarios, select the prescriber which is correct in relation to the scenario. This prescriber must prescribe items which are subject to clinical competence and included within an agreed clinical management plan. Community practitioner nurse Dentist registered in the UK Doctor registered in the UK EEA or Swiss doctor or dentist Nurse independent prescriber Optometrist independent prescriber Pharmacist independent prescriber Supplementary prescriber

Supplementary prescriber Supplementary prescribers must prescribe items which are subject to clinical competence and included within an agreed clinical management plan.

The responsible pharmacist has left the pharmacy to deliver a medication to the nearby nursing home. Given that the responsible pharmacist is allowed to be away from the Pharmacy for up to 2 hours, and that they are the only pharmacist working at the pharmacy, which one of the following activities could you do in their absence? Hand over medications to a patient, which had been dispensed and checked earlier in the day. Provide insulin, as an emergency supply, to a diabetic patient. Supply Beconase® 0.05% Hayfever Nasal Spray 100 dose pack to a patient with hayfever. Supply chloramphenicol eye drops to a patient with conjunctivitis. Supply 2 packs of 32 tablets of paracetamol to a patient with osteoarthritis

Supply Beconase® 0.05% Hayfever Nasal Spray 100 dose pack to a patient with hayfever The answer is to supply hayfever nasal spray. This is a GSL.

Miss RG asks your advice on taking her combined oral contraceptive as she is 5 hours late in taking her next tablet. She is at the middle of her cycle. Which of the following would be the correct advice? - Take two tablets now and no further action is required - Take the missed tablet straight away and no further action is required - Take the missed tablet straight away and use another form of contraception for 7 days - Take the missed tablet straight away and use another form of contraception for 14 days - Stop the present cycle of the pill and start a new cycle in 7 days time

Take the missed tablet straight away and no further action is required. A 'missed pill' is one that is 24 hours late. Therefore, the missed pill is not classed as 'late' and can be taken immediately and the cycle continued.

For the patients below, select the most suitable counselling advice you will give. A 21-year-old male who is taking alendronic acid tablets for osteoporosis. Drowsiness may affect your driving. This effect is enhanced with alcohol consumption. Take this medication at night. Oral solution should be swallowed as a single 100 mL dose with plenty of water while sitting or standing upright. Take this medication on an empty stomach. Avoid factors which cause overheating or dehydration like exercise. Report symptoms including sore throat, mouth ulcers, bruising, fever, tiredness or non-specific illnesses to your doctor immediately. Report immediately if a skin rash develops or if any signs or symptoms of hypersensitivity develops. Take this medication after food

Take this medication on an empty stomach

Mrs H comes to collect her prescription for candesartan regularly from your pharmacy. She has recently been prescribed lithium for the treatment of depression. She comes into your pharmacy disorientated and complains of diarrhoea and vomiting. What do you suspect may be the most likely cause of Mrs H's presentation? - Gastro-intestinal disturbances are a side effect of candesartan . - Candesartan could cause hypernatraemia which is causing lithium toxicity. - Lithium toxicity secondary to Mrs H taking extra doses of lithium.. - Taking candesartan could lead to lithium toxicity due to increased lithium concentration. - She has an infection.

Taking candesartan could lead to lithium toxicity due to increased lithium concentrations. Candesartan potentially increases the concentration of lithium. Manufacturer advises monitor concentration and adjust dose.

A pregnant woman comes into your pharmacy. She has recently heard of spina bifida. She's unsure of the factors which could increase her foetus' risk of developing this condition. Which of the following is NOT known to cause an increased risk of a foetus developing spina bifida? Lack of folic acid. Taking carbamazepine. Obesity. Taking sodium valproate. Taking vitamin B9.

Taking vitamin B9. Vitamin B9 is folic acid - taking this during the pregnancy reduces the risk of developing spina bifida. The following increase the risk of spina bifida: - Lack of folic acid (vitamin B9) - Family history - Medications e.g. valproate and carbamazepine

For each of the following statements, select the drug that fits most closely. This drug is indicated for use in benign prostatic hyperplasia. Amoxicillin Baclofen Chloramphenicol Chlorpromazine Ciprofloxacin Fludrocortisone Sodium risedronate Tamsulosin hydrochloride

Tamsulosin hydrochloride Tamsulosin is indicated for use in BPH.

For each of the following statements, select the drug that fits most closely. The use of this drug is contraindicated in patients who suffer with, or have a history, postural hypotension. Amoxicillin Baclofen Chloramphenicol Chlorpromazine Ciprofloxacin Fludrocortisone Sodium risedronate Tamsulosin hydrochloride

Tamsulosin hydrochloride Tamsulosin may cause a reduction in blood pressure. As such, it is contraindicated in those who suffer with postural hypotension.

Which drug combination is present in the following products? Teysuno®. Ramipril and amlodipine Tegafur, gimeracil and oteracil Tegafur and gimeracil Calcium and vitamin D3 Alendronic acid and colecalciferol Ramipril and felodipine Oxycodone and naloxone Morphine and naloxone

Tegafur, gimeracil and oteracil

It's late on a Friday evening, Mrs SA, a regular patient, enters your pharmacy worried that she has run out of fentanyl patches. On questioning she informs you that she applied her last one at 6pm today. What would be the most appropriate course of action? - Advise Mrs SA to go to A&E because you can not supply her with any. - Make an emergency supply. - Sell Mrs SA co-codamol over the counter. - Supply Mrs SA with a box of fentanyl patches, you'll ask the prescriber for a prescription on Monday. - Tell Mrs SA that you'll have it ready for her to pick up by 5pm Monday after contacting her GP

Tell Mrs SA that you'll have it ready for her to pick up by 5pm Monday after contacting her GP. Fentanyl patches are applied every 72 hours. Mrs SA applied her patch at 6pm, therefore, should not need to apply a patch until 6pm Monday so supplying a prescription by 5pm Monday would be sufficient.

Benzodiazepine drugs are used for conditions such as insomnia and anxiety. They belong to the list of controlled substances due to their potential to be abused. Which of the following Benzodiazepine drugs belongs to Schedule 3 of the Controlled Drugs list? Alprazolam Diazepam Nitrazepam Oxazepam Temazepam

Temazepam Temazepam is the only benzodiazepine to belong to Class 3 of Controlled Drugs.

You are the Responsible Pharmacist and you are helping your pre-registration pharmacist to revise about Controlled Drugs. Which of the following Controlled Drugs is most likely to require a licence for import/export as well as the need to retain the invoice for 2 years? Diazepam tablets 2 mg Morphine sulfate Oral Solution 10 mg/5 mL Morphine sulfate Oral Solution 100 mg/5 mL Morphine sulfate tablets 10 mg Temazepam 10 mg tablets

Temazepam 10 mg tablets Schedule 3 CD requires imp/exp licence and invoice retention.

A 51-year-old man presents his prescription for temazepam 10 mg tablets. Which of the following statements is true regarding temazepam tablets? Temazepam is a schedule 2 controlled drug. Temazepam must be stored in the controlled drugs cupboard. Temazepam prescriptions are valid for 3 months. Temazepam supply must be recorded in the controlled drug register. The quantity of temazepam should not exceed 28 days supply

Temazepam must be stored in the controlled drugs cupboard. Temazepam is a schedule 3 controlled drug which needs to be stored in the controlled drugs cabinet. None of the controlled drugs in this schedule need to be recorded in the CD register. All temazepam prescriptions are required to meet full prescription writing requirements as for any other Schedule 2 or 3 controlled drugs.

For each of the following descriptions, select the condition that is most accurate. Is frequently seen in elderly women. Cluster headache Medication-overuse headache Meningitis Migraine Sinusitis Sub-arachnoid haemorrhage Temporal arteritis Tension-type headache

Temporal arteritis Temporal arteritis is the only condition listed which is strongly associated with advancing age and is also more common in women.

Mr Patel comes into your pharmacy. He has been suffering from a headache since this morning. He describes it as feeling like there is a tight band around his head. He says that the head ache is about a 5 out of 10 (0 = not painful at all; 10 = extremely painful), is not aggravated by his daily activities and is not accompanied by nausea. Which one of the following is Mr Patel most likely to be suffering from? Chronic daily headache Cluster headache Migraine Referred jaw pain Tension headache

Tension headache. A headache described as being like a tight band is indicative of a tension headache.

For each of the following descriptions, select the condition that is most accurate. Is the most prevalent headache condition. Cluster headache Medication-overuse headache Meningitis Migraine Sinusitis Sub-arachnoid haemorrhage Temporal arteritis Tension-type headache

Tension-type headache Most listed conditions are uncommon or rarely seen in primary care. Only migraine and tension-type headache are regularly encountered. Of the two, tension-type headache is the most prevalent.

Mr K is a 19-year-old student who presents with a lesion on his lower abdomen. He has been using hydrocortisone 1% cream but says that it hasn't helped and the lesion is getting bigger. Given that Mr K has tinea corporis, which one of the following treatments is the most suitable to treat his lesion? Betamethasone valerate 0.1% cream Flucloxacillin 250 mg capsules Ketoconazole shampoo Prednisolone 10 mg tablets Terbinafine 250 mg tablets

Terbinafine 250 mg tablets. Dermatophyte infections such as tinea corporis may be exacerbated by prolonged use of topical steroids such as hydrocortisone as these exhibit immunosuppressive properties. Terbinafine is indicated in the treatment of tinea corporis.

For each of the following questions, select the drug which is correct in relation to the scenario. You are giving a training session on the treatment on acne vulgaris to some pre-registration trainees. A trainee asks which oral antibiotics are suitable for use in the treatment of acne vulgaris. Ceftazidime Ertapenem Tetracycline Amikacin Clarithromycin Sodium fusidate Co-trimoxazole Fidaxomicin

Tetracycline. Deposition of tetracyclines in growing bones and teeth causes staining and occasionally dental hypoplasia and they should NOT be given to children under 12.

For each of the following patients, select the most appropriate treatment option for the scenario described. Miss Q is a 32-year-old female. She is a regular at your pharmacy. For the past year she has been suffering from a malar rash on her face, facial flushing, burning and itching papules and pustules. She has tried potent corticosteroid creams, which initially helped but then worsened her rash considerably. Betamethasone Clobetasone butyrate Griseofulvin Hydrocortisone Ketoconazole shampoo Miconazole Rest and ibuprofen Tetracycline

Tetracycline. Her symptoms are those of acne rosacea which worsens with the use of topical steroids. The treatment is oral tetracycline.

For each of the following questions, select the drug which is correct in relation to the scenario. You are giving a training session on the treatment on acne vulgaris to some pre-registration trainees. A trainee asks which oral antibiotics are suitable for use in the treatment of acne vulgaris. Ceftazidime Ertapenem Tetracycline Amikacin Clarithromycin Sodium fusidate Co-trimoxazole Fidaxomicin

Tetracycline. Tetracycline is indicated for the treatment of acne vulgaris and is normally prescribed at a dose of 500 mg twice daily. Maximum improvement usually occurs after 4 to 6 months of treatment.

You are giving a training session on the treatment of acne vulgaris to a group of pre-registration trainees. A trainee asks which oral antibiotics are suitable for this indication. Which of the following antibiotics would be suitable to prescribe for the treatment of acne vulgaris? Amikacin Ceftazidime Co-trimoxazole Ertapenem Tetracycline

Tetracycline. Tetracylines are commonly used in the treatment and prevention of Acne Vulgaris.

An 81-year-old woman comes into the pharmacy seeking advice about a build-up of wax in both ears. She reports that there has been some discharge from both ears, however there is no pain. She has no other medical conditions and is not on any regular medication. What is the most appropriate advice to give to this lady? Inserting cotton buds into the ears will help remove the ear wax. No action is required as the condition is self-limiting. The ears will need to be syringed. The GP will be able to prescribe appropriate medication to treat the ear infection. Using olive oil in the ears will help to soften the ear wax.

The GP will be able to prescribe appropriate medication to treat the ear infection. The discharge indicates an ear infection which would require further investigation by the GP and treatment with an antibiotic ear drop. Inserting cotton buds in the ear may lead to inner ear damage and it is not appropriate to syringe or use olive oil in the ears if they are infected.

You are labelling prescriptions in your pharmacy. Which of the following is legally required on the dispensing label? Address of prescriber Date of dispensing Expiry date of medicine "Keep out of the sight and reach of children" Recommended BNF warning labels for the relevant medicine

The date of dispensing is a legal requirement. All other options are good practice.

You are presented with a prescription form FP10PCD, written for a 12-year-old child for two items: Rx 1. methylphenidate 10 mg tablets, 1 bd, mitte 60 2. paracetamol 250 mg/5 mL oral suspension, 10 mL qds mitte 200 mL Which of the following statements is the most appropriate to indicate why the prescriber should be contacted? - The doctor should be contacted and asked to amend the quantity for methylphenidate to 56 tablets - The doctor should be contacted as the dose of paracetamol is too high for a 12 year old - The doctor should be contacted as the paracetamol should be prescribed on a different prescription - The doctor should be contacted as they should add the word 'sixty' to the quantity of methylphenidate - The doctor should be contacted as they should add the word 'ten' to the strength

The doctor should be contacted as the paracetamol should be prescribed on a different prescription. FP10PCD should be used for CD schedule 2 and 3, then original sent to NHS agency, whereas a private prescription for paracetamol needs to be retained in the pharmacy and therefore should not be prescribed on FP10PCD.

You are the pharmacist explaining to a pharmacy summer placement student which medicines are not suitable for use in children, including those in their mother's womb. Select the reason why this drug should not be used in children. Sodium valproate for a 15-year-old girl newly diagnosed with epilepsy. Bone marrow toxicity Damage to skeletal development Discolouration of teeth Drug toxicity Reye's syndrome Stunted growth The medicine is teratogenic The risk of using the medicine outweighs the benefits

The medicine is teratogenic. Sodium valproate is teratogenic. As the female is of child bearing age and may want children in the future, it would be better to choose a different antiepileptic as a first-line treatment.

A patient receives Kevzara at a dose of 200 mg every other week on a Wednesday lunchtime. She calls you at 9am on a Saturday to say she forgot to administer the latest injection. Which of the following would be appropriate advice to give the patient? The missed dose should not be administered and she should have her next dose as usual. She should stop treatment until she has seen her specialist. The dose should be missed and the next regularly scheduled dose doubled. The missed dose should be administered and subsequent scheduled doses should now be given on alternate Saturday mornings. The missed dose should be administered and the next dose administered at the regularly scheduled time

The missed dose should be administered and the next dose administered at the regularly scheduled time. If a dose of Kevzara is missed and it has been 3 days or less since the missed dose, the next dose should be administered as soon as possible. The subsequent dose should be administered at the regularly scheduled time. If it has been 4 days or more since the missed dose, the subsequent dose should be administered at the next regularly scheduled time, the dose should not be doubled.

A patient calls you at your pharmacy at 8pm. They use Toujeo and usually administer their medicine at 6pm. The patient got stuck in traffic on his way home from work and his insulin was at home. He has only just arrived home. He asks what to do about taking his dose of Toujeo. Which of the following would be appropriate advice to give the patient?The SPC is provided here: https://www.medicines.org.uk/emc/product/6938 - The patient can take his normal dose now - The patient needs to see an out of hours practitioner for advice - The patient should inject 1/6th of his daily dose in addition to his usual dose to make up for the missed time - The patient should inject a double dose - The patient should skip the dose and take as normal tomorrow

The patient can take his normal dose now When needed, patients can administer Toujeo up to 3 hours before or after their usual time of administration.

A customer requests Flomax Relief MR (tamsulosin hydrochloride) from your pharmacy. What would be the most urgent need for referral? The patient has cataracts. The patient has cloudy urine. The patient has had symptoms for less than 3 months. The patient has other medical problems. The patient is over 75 years of age.

The patient has cloudy urine. Cloudy urine could be a sign of infection (UTI) or chronic retention. Patient age and comorbidities are also reasons to refer but the infection/chronic retention is the most urgent.

For each of the following medicines, select the correct point for the patient. A 67-year-old lady receiving gabapentin for the treatment of epilepsy. - If the patient suffers a severe skin reaction in the first 8 weeks, the medicine should be stopped immediately. - The patient should be advised that they require routine monitoring of blood levels of the medicine. - The patient should be asked to sign the back of the prescription form as confirmation of collection. - The patient should be provided with a Patient Card. - The patient should immediately report a sore throat to a healthcare professional. - The patient should take the medicine once weekly. - The patient should stop taking this medicine if they have a seizure and seek further medical advice. - The same brand must always be supplied.

The patient should be asked to sign the back of the prescription form as confirmation of collection. From 1 April 2019, pregabalin and gabapentin are schedule 3 controlled drugs and should be treated as such when dispensing. They are exempt from safe custody requirements.

For each of the following medicines, select the correct point for the patient. Each option may be used once, more than once, or not at all. A 21-year-old male receiving pregabalin for the treatment of epilepsy. - If the patient suffers a severe skin reaction in the first 8 weeks, the medicine should be stopped immediately. - The patient should be advised that they require routine monitoring of blood levels of the medicine. - The patient should be asked to sign the back of the prescription form as confirmation of collection. - The patient should be provided with a Patient Card. - The patient should immediately report a sore throat to a healthcare professional. - The patient should take the medicine once weekly. - The patient should stop taking this medicine if they have a seizure and seek further medical advice. - The same brand must always be supplied.

The patient should be asked to sign the back of the prescription form as confirmation of collection. From 1 April 2019, pregabalin and gabapentin are schedule 3 controlled drugs and should be treated as such when dispensing. They are exempt from safe custody requirements.

For each of the following medicines, select the correct point for the patient. A 28-year-old lady receiving sodium valproate for the treatment of epilepsy. - If the patient suffers a severe skin reaction in the first 8 weeks, the medicine should be stopped immediately. - The patient should be advised that they require routine monitoring of blood levels of the medicine. - The patient should be asked to sign the back of the prescription form as confirmation of collection. - The patient should be provided with a Patient Card. - The patient should immediately report a sore throat to a healthcare professional. - The patient should take the medicine once weekly. - The patient should stop taking this medicine if they have a seizure and seek further medical advice. - The same brand must always be supplied.

The patient should be provided with a Patient Card. As part of the pregnancy prevention programme, women of childbearing age taking sodium valproate should be given a Patient Card.

You are the Responsible Pharmacist for a community pharmacy. For each of the scenarios select the correct response. You are stuck in traffic on the way to work and will be late. Your pharmacy has a paper responsible pharmacist log. You are able to pull over to call your colleagues. What is the correct operating procedure to tell your staff? - Close the pharmacy and only compliance aids can be made up - Normal service, the dispenser and ACT can dispense prescriptions - Open the pharmacy and sell GSL and P medicines and shop products - Open the pharmacy and only methadone can be handed out - Open the pharmacy and only sell shop products (e.g. cosmetics) - Open the pharmacy and sell GSL medicines and shop products - Open the pharmacy and sell GSL medicines, shop products and hand out prescriptions already dispensed - The pharmacy should be closed

The pharmacy should be closed In the absence of a responsible pharmacist being signed in, the pharmacy cannot open.

You are the Responsible Pharmacist for a community pharmacy. For each of the scenarios select the correct response. You are the responsible pharmacist in a 100 hour pharmacy. You sign in at 3pm. At 5pm you need to visit the local GP surgery for 30 minutes and you will be contactable in this time. You are happy to remain signed in as the responsible pharmacist. You note that the responsible pharmacist from earlier in the day was absent from 11am-1pm. - Close the pharmacy and only compliance aids can be made up - Normal service, the dispenser and ACT can dispense prescriptions - Open the pharmacy and sell GSL and P medicines and shop products - Open the pharmacy and only methadone can be handed out - Open the pharmacy and only sell shop products (e.g. cosmetics) - Open the pharmacy and sell GSL medicines and shop products - Open the pharmacy and sell GSL medicines, shop products and hand out prescriptions already dispensed - The pharmacy should be closed

The pharmacy should be closed Responsible pharmacists can only be absent for 2 hours between midnight and midnight. This 2 hours is inclusive of all responsible pharmacists working in a pharmacy in this period. In the absence of a responsible pharmacist being signed in, the pharmacy must close.

You are the pharmacist explaining to a pharmacy summer placement student which medicines are not suitable for use in children, including those in their mother's womb. Select the reason why this drug should not be used in children. An over-the-counter cough medicine containing guaifenesin for a 4-year-old child. Bone marrow toxicity Damage to skeletal development Discolouration of teeth Drug toxicity Reye's syndrome Stunted growth The medicine is teratogenic The risk of using the medicine outweighs the benefits

The risk of using the medicine outweighs the benefits.

For each of the following medicines, select the correct point for the patient. A 50-year-old male receiving primidone for the treatment of epilepsy. - If the patient suffers a severe skin reaction in the first 8 weeks, the medicine should be stopped immediately. - The patient should be advised that they require routine monitoring of blood levels of the medicine. - The patient should be asked to sign the back of the prescription form as confirmation of collection. - The patient should be provided with a Patient Card. - The patient should immediately report a sore throat to a healthcare professional. - The patient should take the medicine once weekly. - The patient should stop taking this medicine if they have a seizure and seek further medical advice. - The same brand must always be supplied.

The same brand must always be supplied. Primidone is an anti-epileptic where the same brand should be dispensed each time.

You receive a signed order from a school for a salbutamol inhaler. Who must have signed this order for it to be legally valid? Chair of governors Deputy head Pupil's parent Principal School nurse

The supply must be signed by the principal or head teacher.

For each of the following scenarios, select the most likely condition. Miss E comes into your pharmacy. She is a nurse at the local nursing home. She tells you that she has noticed that one of the residents has been complaining of severe itchy hands, particularly at night. She examined the patient's hands and noticed silver lines with a dot at the end in the webs of her fingers. Allergic reaction Chickenpox Eczema Impetigo Psoriasis Scabies Shingles Urticaria

The symptoms described are those of an early infection of scabies. The mites lay eggs under the skin which leave these silver lines in the hand. This can develop into a rash and the rash can spread and turn into spots.

Mrs Smith has been taking 800 units of ergocalciferol for 2 weeks now because she has a lack of vitamin D. She develops nausea and has now vomited. What is the likely reason for this? The vitamin D may have led to an increase in serum calcium causing the symptoms. The vitamin D therapy may have decreased serum calcium causing the symptoms. The preparations of ergocalciferol are generally not well tolerated. If ergocalciferol is given without calcium, it may cause this symptom. These symptoms are unrelated to Mrs Smith's therapy

The vitamin D may have led to an increase in serum calcium causing the symptoms

Several drugs are used in the management of COPD. These drugs are prescribed at various stages of this chronic condition. Which of the following drugs belong to the class of drugs known as xanthines? Clindamycin Indacaterol Indomethacin Olodaterol Theophylline

Theophylline Theophylline belongs to the class of drugs xanthines.

Which drug combinations may be related to the following scenarios? A 37-year-old patient on a hospital ward suffers from hyperglycaemia, dilated pupils and haematemesis. Warfarin and fluconazole Amiodarone and simvastatin Aspirin and salbutamol Salbutamol and beclometasone Carbamazepine and erythromycin Simvastatin and St John's Wort Budesonide and formoterol Theophylline and disulfiram

Theophylline and disulfiram Metabolism of theophylline is inhibited by disulfiram and therefore there is an increased risk of theophylline toxicity.

You are dispensing a prescription for a patient for an emollient which contains 40% paraffin. The patient tells you he's seen on the television that such emollients can catch fire and asks you for a paraffin-free alternative from the shelves in your shop. What advice would you give him? There is also a risk of fire with emollients that don't contain paraffin. There is only a risk of fire if you're a smoker. There is no risk of fire with paraffin-containing products. The product contains less than 50% paraffin so there is no fire risk. The risk is only if the patient is using bandages over their emollient.

There is also a risk of fire with emollients that don't contain paraffin. There is a fire risk with all paraffin-containing emollients, regardless of paraffin concentration, and it also cannot be excluded with paraffin-free emollients. A similar risk may apply for other products which are applied to the skin over large body areas, or in large volumes for repeated use for more than a few days.

A 15-year-old presents at your pharmacy to collect their own prescription. In order to collect a prescription for themselves from a pharmacy, how old should the patient be? 10 years 12 years 16 years 18 years There is no legal restriction, the pharmacist should exercise their professional judgement

There is no legal restriction, the pharmacist should exercise their professional judgement

A 10-year-old presents at your pharmacy to collect his parent's prescription. In order to collect a prescription for someone else from a pharmacy, how old should the patient who is collecting it be? 10 years 12 years 16 years 18 years There is no legal restriction, the pharmacist should exercise their professional judgement.

There is no legal restriction, the pharmacist should exercise their professional judgement.

What is the last date you could dispense the following prescriptions? A final repeat on a private prescription requesting 3 repeats for a schedule 4 CD. The prescription was signed and first dispensed on 02/09/21. 30/06/21 28/07/21 30/09/21 30/03/22 30/06/22 30/06/26 Repeats are not allowed There is no legally defined final dispensing date, it is based on clinical judgement

There is no legally defined final dispensing date, it is based on clinical judgement There is no legal time limit on the dispensing of private repeat prescriptions for schedule 4 CDs if the first dispensing took place within 28 days of the appropriate date. It should be based on clinical judgement.

What is the last date you could dispense the following prescriptions? The last repeat on a private prescription for candesartan signed and first dispensed on 30/06/21 and repeated on 30/07/21 and 30/08/21. The prescription requested 3 repeats. 30/06/21 28/07/21 30/09/21 30/03/22 30/06/22 30/06/26 Repeats are not allowed There is no legally defined final dispensing date, it is based on clinical judgement

There is no legally defined final dispensing date, it is based on clinical judgement There is no legal time limit on the dispensing of private repeat prescriptions if the first dispensing took place within 6 months of the appropriate date. It should be based on clinical judgement.

You are a registered pharmacist working in a medicines information role speaking to healthcare professionals and completing the occasional locum shift in a community pharmacy. How do the GPhC Standards for Pharmacy Professionals apply to you? They apply in all job roles They only apply when you are locuming They only apply when you are a responsible pharmacist They only apply when you are working with patients They only apply when you are working as a pharmacist

They apply in all job roles The GPhC states: The standards apply to all pharmacists and pharmacy technicians. We know that pharmacy professionals practise in a number of sectors and settings and may use different ways to communicate with the people they provide care to. The standards apply whatever their form of practice.

A mother comes into your pharmacy to ask for advice. Her 3-year-old daughter has been waking at night and has recently started wetting the bed. Which of the following is most likely to be causing the symptoms? Hook worm Ringworm Roundworm Threadworm Tick

Threadworm can cause children to wake at night and occasionally wet the bed.

A woman presents in your pharmacy asking for advice. She has been breastfeeding her child for 5 weeks. She is experiencing pain in both breasts which has recently started. Her baby has been unsettled during this time and also has bad nappy rash, along with white spots on it's tongue. Which of the following is the lady most likely experiencing? Blocked ducts Engorgment Mastitis Reduced milk supply Thrush in the nipple/breast

Thrush in the nipple/breast. The symptoms described are those of thrush (candida) infection in the breast. Breastfed babies can also develop thrush in their mouths, a sign for which in some babies is nappy rash that won't clear up.

For each of the following patients, select the most likely cause for the weight loss described in the scenario. Miss O is a 24-year-old patient who tells you that she has weight loss accompanied by insomnia, diarrhoea and palpitations. Anorexia nervosa Bulimia nervosa Cancer Coeliac disease Crohn's disease Diabetes mellitus Thyrotoxicosis Tuberculosis

Thyrotoxicosis Her symptoms are those of thyrotoxicosis. This includes: weight loss, palpitations (associated with atrial fibrillation), insomnia, and diarrhoea.

For each of the following scenarios, select the drug which is correct in relation to the scenario. You are on a respiratory ward and upon counselling an asthmatic patient he informs you that his breathing has become slightly more laboured since being initiated on a new medication whilst an inpatient. Which is the most likely cause of the breathing difficulty? Benzydamine hydrochloride Dalteparin Duloxetine Isosorbide mononitrate Isotretinoin Pizotifen Timolol 0.25% eye drops Xylometazoline hydrochloride 0.1%

Timolol 0.25% eye drops Timolol is a beta-blocker. Beta-blockers are often contraindicated for use in asthmatics as they may worsen symptoms.

A patient visits the pharmacy on a Saturday afternoon for advice. They have had a cough for 2 weeks. When it first started they were unwell with other cold symptoms, but apart from the cough persisting they are now otherwise well. What would you advise the patient to do? To follow self-care treatments and visit the GP if the cough still persists after 4 weeks from its start. To follow self-care treatments and visit the GP if the cough still persists after 8 weeks from its start. To go to A & E. To go to the out of hours GP today. To make an appointment with their GP this week

To follow self-care treatments and visit the GP if the cough still persists after 4 weeks from its start. Acute cough is usually self-limiting and gets better within 3 to 4 weeks without antibiotics. Coughs are most commonly caused by a viral upper respiratory tract infection, such as a cold or flu.

You have a patient ask you about purchasing sumatriptan for migraines. Which of the following medicines is known to interact with sumatriptan? Aspirin Ibuprofen Mefenamic acid Paracetamol Tramadol

Tramadol. Both sumatriptan and tramadol can increase the risk of serotonin syndrome.

A patient has recently been initiated on a new medication and is complaining of bad side effects. Upon questioning they tell you the most severe side effect is general fatigue. Which of the following drugs is most likely to cause this side effect? Bisacodyl Clindamycin Ibuprofen Isphagula husk Tramadol

Tramadol. Fatigue is listed as a common or very common side effect of tramadol.

Which one of the following statements is NOT correct regarding malaria prophylaxis? Chloroquine is unsuitable for malaria prophylaxis in individuals with a history of epilepsy. Mefloquine has an increased risk of ventricular arrhythmias when used with amiodarone. Prophylaxis should be continued for 4 weeks after leaving with the exception of Malarone.® Prophylaxis is not absolute and therefore infection can occur with any of the drugs recommended. Travel to malarious areas should be avoided during pregnancy; if travel is unavoidable, then only use personal protection (such as mosquito nets), as proguanil is contraindicated in pregnancy.

Travel to malarious areas should be avoided during pregnancy; if travel is unavoidable, then only use personal protection (such as mosquito nets), as proguanil is contraindicated in pregnancy. Proguanil can be given in the usual doses during pregnancy as benefit of prophylaxis in malaria outweighs risk.

It is mid-March and a patient presents to you in the pharmacy displaying cold-like symptoms. On questioning she says she remembers having a really bad cold last year at the same time. On questioning the patient, you suspect she might have an allergy rather than a cold. What is the likely cause of her allergy? Dust mites Grass pollen Mould Pet allergy Tree pollen

Tree pollen In March, the responsible allergen is most likely to be tree pollen.

You are the pharmacist explaining to a pharmacy summer placement student which medicines are not suitable for use in children, including those in their mother's womb. Select the reason why this drug should not be used in children. Trimethoprim in a lady who is 10 weeks pregnant. Bone marrow toxicity Damage to skeletal development Discolouration of teeth Drug toxicity Reye's syndrome Stunted growth The medicine is teratogenic The risk of using the medicine outweighs the benefits

Trimethoprim in a lady who is 10 weeks pregnant.

You receive a prescription for morphine sulfate tablets 10 mg. What is the appropriate wording to meet legal requirements for the total quantity on the prescription when the prescriber wants you to give a total of 60 tablets? 60 tablets Sixty tablets Two (2) packs of 30 (thirty) tablets Two packs of 30 (thirty) tablets Two (2) packs of 30 tablets

Two (2) packs of 30 (thirty) tablets. The total quantity must be expressed in words and figures and the Home Office advises this should be as the number of dosage units. Multiplication of pack sizes is acceptable.

Mrs XR is ready to be discharged home. The hospital policy is to give patients a 14 day supply of insulin on discharge, providing the patient does not have a supply at home. Mrs XR does not have a supply at home and she is now on NovoRapid® FlexPen 14 units three times daily and Levemir® FlexPen 20 units at night. How many insulin pens would cover the amount of insulin the patient will inject in 14 days? The SPCs are provided here: NovoRapid®: http://www.medicines.org.uk/emc/medicine/25033 Levemir®: http://www.medicines.org.uk/emc/medicine/14584 One NovoRapid® and one Levemir® Two NovoRapid® and one Levemir® One NovoRapid® and two Levemir® Two NovoRapid® and two Levemir® Three NovoRapid® and two Levemir®

Two NovoRapid® and one Levemir® NovoRapid® = 14 units x 3 times daily x 14 days = 588 units in total. Each disposable pen contains 300 units therefore will require 2 NovoRapid® pens. Levemir® = 20 units x 1 times daily x 14 days = 280 units in total. Each disposable pen contains 300 units therefore will require 1 Levemir® pen.

A person is travelling to an area where there is poor sanitation and poor food hygiene and requires advice. Which vaccine is indicated for this patient? Hepatitis B vaccine Meningococcal A, C, W135 and Y vaccine Pneumococcal vaccine Rabies vaccine Typhoid vaccine

Typhoid vaccine

Jessica Wright is 23 years old and has recurrent lower UTI. What is low dose therapy?

Uncomplicated lower UTIs respond to trimethoprim or nitrofurantoin. Alternative amoxicillin or cephalosporins. 7 days treatment but sometimes 3 days is okay in women. Long term low dose therapy may be required in some patients to prevent recurrence. In that case lower doses will be given.

For the following scenarios, select the most likely abbreviation from the list below. The latin term used for 'ointment' on a prescription for betamethasone. o.m. Nocte Pulv. Unguentum narist. i.c. u.a. p.c.

Unguentum Unguentum is the latin word for ointment.

An elderly woman presents in the pharmacy on a Friday morning complaining of a bothersome headache. Which one of the following symptom clusters require you to make an urgent referral to the doctor? Bilateral generalised headache Bilateral headache with symptoms that worsen as the day progresses Unilateral frontal headache which worsens on bending down Unilateral headache with associated nausea Unilateral headache with malaise

Unilateral headache with malaise. Unilateral headache with associated nausea suggests migraine; bilateral generalised headache and bilateral headache with symptoms that worse as the day progresses suggests tension-type headache; unilateral frontal headache which worsens on bending down suggests sinusitis; unilateral headache with malaise could be temporal arteritis.

For each of the following patients, select the drug which is contraindicated due to drug disease interactions. On a ward round you discover a patient has been admitted with acute inflammation of the gall bladder. Which medication is contraindicated in this patient? Ursodeoxycholic acid Amiodarone Bendroflumethiazide Pseudoephedrine Piperacillin with tazobactam Levomemprazine Sodium hyaluronate 0.1% eye drops Colestyramine

Ursodeoxycholic acid Ursodeoxycholic Acid is contraindicated in patients with acute inflammation of the gall bladder.

Mr VK comes to your pharmacy. He says that he is suffering from an allergic reaction to something. On questioning him you find that he started a course of antibiotics two days ago for a chest infection. Which one of the following is a typical symptom of a drug induced allergic reaction? Nausea and vomiting Headache Urticaria Diarrhoea Constipation

Urticaria Skin reactions such as urticaria can be associated with drug reactions.

You are a pre-registration pharmacist learning about the reconstitution of cytotoxic drugs in the aseptic unit at the hospital that you are training at. Which of the following is NOT true with regards to the handling of cytotoxic medicines? Pregnant staff should avoid exposure to cytotoxic drugs. Protective clothing (including gloves, gowns, and masks) should be worn. Reconstitution should be carried out in designated pharmacy areas. Trained personnel should reconstitute cytotoxics. Use national procedures for dealing with spillages and safe disposal of waste material, including syringes, containers, and absorbent material.

Use national procedures for dealing with spillages and safe disposal of waste material, including syringes, containers, and absorbent material. The BNF states that local, rather than national procedures should be use "Use local procedures for dealing with spillages and safe disposal of waste material, including syringes, containers, and absorbent material".

You have received a hand written FP10 prescription from a GP. It is not for a controlled drug. The doctor has included the strength of the preparation on the prescription. The product does not come in that strength but you note that the marketed strengths available are all lower in strength and you can use these to make the requested strength. What is your best course of action? - Ask the dispenser what you've got on the shelf and dispense that - Ask the doctor's receptionist what strength to give - Ask the patient to return to the prescriber to get the strength added - Ask the patient what strength they usually have - Contact the prescriber to clarify the strength and add a 'PC' endorsement to the prescription - Dispense the prescription with no additional actions - Dispense the strength available and endorse the prescription accordingly - Use your professional judgement about which strength to dispense

Use your professional judgement about which strength to dispense. The product strength is not a legal requirement on this prescription, but appropriate actions should be taken to dispense the correct product. Where the strength prescribed does not exist, professional judgement should be used to make the strength using lower strengths. Where this cannot be done, the prescriber should be contacted for guidance.

You are running a workshop helping to educate pharmacy staff on the safe handling of medicines. You receive a question from an audience member regarding drugs which require caution in handling. Which of the following drugs would you be able to use as an example of a drug which requires 'caution in handling'? Chloroquine Famciclovir Mebendazole Pyrimethamine Valganciclovir

Valganciclovir. Valganciclovir is a potential teratogen and carcinogen and caution is advised when handling the powder, reconstituted solution or broken tablets; if these come into contact with skin or mucosa, wash off immediately with water; avoid inhalation of powder.

For each of the following statements or scenarios, select the drug which is correct. You receive a phone call from a GP regarding the use of a drug as a prophylactic treatment for migraines in a 27-year-old male patient. Dexamfetamine sulfate Lisdexamfetamine mesilate Atomoxetine Methylphenidate hydrochloride Valproic acid Asenapine Lithium carbonate Lithium citrate

Valproic acid. Valproic acid is used for prophylaxis of migraine but this is an 'unlicensed use'.

The metabolism of which one of the following medicines is NOT increased by cigarette smoking? Theophylline Clozapine Pirfenidone Ropinirole Varenicline

Varenicline. Theophylline - Smoking may increase theophylline clearance and increased doses of theophylline are therefore required. Clozapine - in cases of sudden cessation of smoking, the plasma clozapine concentration may be increased, thus leading to an increase in adverse effects. Pirfenidine - smoking has the potential to induce hepatic enzyme production and thus increase medicinal product clearance and decrease exposure. Ropinirole - smoking is known to induce CYP1A2 metabolism, therefore if patients stop or start smoking during treatment with ropinirole, dose adjustment maybe required.

For each of the following patients, select the vitamin deficiency that is the most likely cause of the signs and symptoms described in the scenario. Mr D comes into your pharmacy. He tells you that he has recently been to visit his 93 year old father. He express his concern regarding his father's health over the past 3 months. He says that he has swollen and bleeding gums and that he has developed red spots on his shins. He has also noticed a few bruises on his body. Vitamin A Vitamin B1 Vitamin B3 Vitamin B12 Vitamin C Vitamin D Vitamin E Vitamin K

Vitamin C His fathers symptoms are indicative of scurvy (vitamin C deficiency). Although rare, it could occur in very elderly patients who may find it harder to cook or maintain a healthy diet.

For each of the following patients, select the vitamin deficiency that is the most likely cause of the signs and symptoms described in the scenario. Mrs A comes into your pharmacy. She tells you that her 5 year old daughter tires easily, is reluctant to walk and it looks like her legs are bowed. Vitamin A Vitamin B1 Vitamin B3 Vitamin B12 Vitamin C Vitamin D Vitamin E Vitamin K

Vitamin D The child's symptoms are indicative of rickets, as a result of vitamin D deficiency.

For each of the following questions, select the vitamin or mineral which is the correct answer. This helps the absorption of calcium in the gut. Ascorbic acid Calcium Folic acid Iron Retinol Vitamin D Vitamin K Zinc

Vitamin D Vitamin D is needed for the body to absorb calcium effectively. Unlike other vitamins, we do not need to get vitamin D from food. Most of the vitamin D we have is made by our own bodies. It is made in the skin by the action of sunlight. This is a good thing because most foods contain no, or very little, vitamin D naturally.

For each of the following questions, select the vitamin or mineral which is the correct answer. This requires hydroxylation by the kidneys to its active form, therefore, in patients with a severe reduction in renal function, alfacalcidol should be prescribed instead of this. Ascorbic acid Calcium Folic acid Iron Retinol Vitamin D Vitamin K Zinc

Vitamin D Vitamin D, colecalciferol, is the non-activated form of vitamin D. It is activated in the kidneys, so if a patient has reduced renal function then the activated form (alfacalcidol) should be given instead.

Which of the listed vitamins and minerals should be prescribed to the following patients? 1 mg of this vitamin could be given as a single intramuscular injection at birth to prevent serious bleeding, including intracranial bleeding. Vitamin E Vitamin K Vitamin A Vitamin B12 Calcium Vitamin B1 Phosphate Vitamin B6

Vitamin K

For each of the following questions, select the vitamin or mineral which is the correct answer. A patient who takes warfarin for atrial fibrillation and has an INR of >8 is admitted onto your ward. What should be given by slow intravenous injection? Ascorbic acid Calcium Folic acid Iron Retinol Vitamin D Vitamin K Zinc

Vitamin K Vitamin K (Phytomenadione) helps to counteract the warfarin action and slow down the bleeding.

Which drug combinations may be related to the following scenarios? A patient comes into your pharmacy suffering from ecchymosis and bleeding from the gums. Warfarin and fluconazole Amiodarone and simvastatin Aspirin and salbutamol Salbutamol and beclometasone Carbamazepine and erythromycin Simvastatin and St John's Wort Budesonide and formoterol Theophylline and disulfiram

Warfarin and fluconazole Anticoagulant effect of warfarin increased with fluconazole.

Which one of the following medicines has an interaction with naproxen which is graded as severe? Ramipril Warfarin Amlodipine Allopurinol Paracetamol

Warfarin. The BNF grades the interaction with warfarin as severe (red) but that with ramipril is graded as less severe (amber).

You are providing a training session on food interactions with medicines. Which of the following is the LEAST likely to interact with grapefruit juice? Budesonide Lercanidipine Simvastatin Tacrolimus Warfarin

Warfarin. Warfarin is not noted in the BNF as interacting with grapefruit juice. This gives an indication of the severity of the interaction. As warfarin isn't listed here, it indicates that it is least likely to interact because there isn't as much documented evidence.

You are working on a ward in the hospital and you have had several interactions with a nurse who quite clearly is not competent to deal with the patients and their medication. Which of the following actions is NOT appropriate to do? Find the policy on raising concerns within the hospital. Speak to a senior member of the nursing team. Tell your senior manager what your thoughts are. Report to the Nursing and Midwifery Council. Warn all the patients who are being looked after by this nurse.

Warn all the patients who are being looked after by this nurse. It would not be an appropriate course of action to notify the patients.

A patient attends anticoagulation clinic for a routine INR check. The target INR for this patient is 2.5-3.5, however in clinic the INR reading is 5.2 with no associated bleeding. What is the most appropriate course of action in this instance? Stop warfarin and administer intravenous phytomenadione Stop warfarin and administer oral phytomenadione Stop warfarin and review in 5 days Withold 1 or 2 doses of warfarin and reduce subsequent maintenance dose Withold 1 or 2 doses of warfarin

Withold 1 or 2 doses of warfarin and reduce subsequent maintenance dose. Withhold 1 or 2 doses of warfarin and reduce subsequent maintenance dose as per BNF guidance.

You have recently qualified as a pharmacist. Which of the following is NOT a standard you must meet as a qualified pharmacist? Behave in a professional manner Communicate effectively Maintain, develop and use your professional knowledge and skills Provide person-centred care Work independently

Work independently The standards state that pharmacists should work in partnership with others.

For each question in this section select one answer from the list of eight options above it. This medication is a sympathomimetic which if used for more than seven days can cause rhinitis medicamentosa. Aspirin 75 mg tablets Clotrimazole 1% cream Guaifenesin 100 mg/5 mL & Levomenthol 1.1 mg/5 mL oral solution Loratidine 10 mg tablets Paracetamol 500 mg tablets Pholcodine 5 mg/5 mL linctus Sumatriptan succinate 50 mg tablets Xylometazoline 0.1% nasal spray

Xylometazoline 0.1% nasal spray

A mother requests a product over the counter for her 7-year-old son who is suffering from a cold. She wants to purchase a nasal decongestant containing xylometazoline. What is the maximum length of time the child should use it for? Two days Three days Five days Seven days A child this age must not use it

Xylometazoline should not be used for more than 5 days in a 7 year old.

What would be appropriate counselling for the patient in the following scenario? A prescription for mycophenolate mofetil for a 25-year-old male patient. Do not take if you are allergic to sulphur-containing products. May cause drowsiness. Remain upright for 30 minutes after taking. Swallow whole with a glass of water. There are no specific counselling points. This medicine is only taken once per week. You and your partner should use appropriate contraception. You should use two types of contraception during treatment.

You and your partner should use appropriate contraception As a precautionary measure for male patients, it is now recommended that either the patient or their female partner use reliable contraception during treatment with mycophenolate medicines and for at least 90 days after stopping.

You have just finished using a stock bottle of methadone in the pharmacy. You are unable to pour anything out of the bottle, but you can see that there are still traces inside. How do you record the residue liquid in the CD register? You do not need to make a record of disposal of irretrievable amounts of CDs. You should add 10 ml for end of bottle residue to the CD register. You should alert the accountable officer that you disposed of an unknown amount of residue. You should make an estimate and record in the CD register. You should record the amount that balances out the register.

You do not need to make a record of disposal of irretrievable amounts of CDs Disposal of irretrievable amounts of CDs does not need to be recorded.

Mrs J is a 71-year-old female who has just been prescribed bromocriptine. Which one of the following is the most suitable counselling advice you should offer Mrs J? Report a skin rash which develops on your chest immediately to your doctor. Report immediately any sign of tremor to your doctor. Take this at least 2 hours after your other medications. Take this medication in the morning. You may experience low blood pressure initially with this medication so take extra care to avoid falls.

You may experience low blood pressure initially with this medication so take extra care to avoid falls Hypotensive reactions can occur in some patients taking bromocriptine, especially during the first few days of treatment.

A 24-year-old female comes into your pharmacy to ask for a hormonal emergency contraceptive pill. After asking her all of the relevant questions you supply ulipristal and counsel her. She is not currently taking the oral contraceptive pill, she is not taking any other medicines, has no medical conditions and no allergies. Which one of the following counselling points is the LEAST suitable to tell the patient? You should use a barrier method of contraception until your next period. Your next period may be earlier or later than usual. If you are sick within the first 3 hours of taking this medicine you should come back for a replacement dose. You may experience lower abdominal pain as a side effect of this medication. If the next menstrual period is more than 7 days late a pregnancy test should be performed.

You may experience lower abdominal pain as a side effect of this medication. Lower abdominal pain may be indicative of an ectopic pregnancy and medical attention should be sought.

For the described patients, select the most likely counselling advice you would give them from the list below. A 67-year-old male who has been prescribed co-careldopa for Parkinson's disease. - Take this medication on an empty stomach. - Take this medication after food. - You may experience sudden onset of sleep. Please exercise caution when performing skilled tasks. - Report any signs of visual impairment immediately to your prescriber. - Your medication may colour your urine blue. This is harmless. - Do not take this medication at the same time as antacids. - Immediately report any signs or symptoms of anaemia, bruising or infection to your doctor. - Take this medication at night

You may experience sudden onset of sleep. Please exercise caution when preforming skilled tasks Dopamine receptor agonists are associated with sudden onset of sleep.

Indemnity cover is a requirement for pharmacists, but what is said about it? You should make sure you have appropriate indemnity cover. There are certain companies it is better to buy from. If you have complaints made against you, you should get some. Your employer will always cover you. If you have cover, you must use it.

You should make sure you have appropriate indemnity cover.

What would be appropriate counselling for the patient in the following scenario? A prescription for a mycophenolate mofetil for a 25-year-old female patient. Do not take if you are allergic to sulphur-containing products. May cause drowsiness. Remain upright for 30 minutes after taking. Swallow whole with a glass of water. There are no specific counselling points. This medicine is only taken once per week. You and your partner should use appropriate contraception. You should use two types of contraception during treatment.

You should use two types of contraception during treatment Female patients of childbearing age receiving mycophenolate should always use contraception - 2 methods of effective contraception are preferred.

Ahmed is a 4-year-old child with chronic asthma and he has been taking Salbutamol MDI 100 micrograms 1-2 puffs when required for the last 6 months. Three months ago the GP started Ahmed on beclometasone MDI but this was found to be ineffective and this was stopped two weeks ago when the GP started Ahmed on montelukast 4 mg chewable tablets 4 mg once daily. Ahmed's mother, who picks up his son's medicines from your pharmacy regularly, tells you that he does not like chewing the montelukast tablets and spits it out. She asks you if there is an alternative to the chewable tablets. Which one of the following is the most appropriate reply to Ahmed's mother? - There is no alternative - Ask Ahmed to swallow the chewable tablet whole - Crush the tablet and mix with cold food and take immediately - You will contact the GP and ask to switch to montelukast granules - You will contact the GP to find out if Ahmed has tried an inhaled corticosteroid and, if not, suggest that this is prescribed instead of montelukast

You will contact the GP and ask to switch to montelukast granules. Asking the patient to swallow the chewable tablet whole or crushing the tablet and mixing into food would be an unlicensed method of administration hence contacting the GP and asking to switch to montelukast granules is the correct answer.

Based on the below days/packs supplied, what is the maximum number of days/packs of medication that you could legally give to a patient as an emergency supply for the following medications, if any? Zoely®. Please give number of day(s). 0 1 3 5 14 21 28 30

Zoely is a 28-day preparation with 24 active tablets and 4 inactive tablets, rather than a seven day tablet free interval. Under emergency supply regulations, a full cycle of a contraceptive may be supplied so in this case 28 days would be classed as the full cycle even with the inactive tablets.

Miss J is a 23-year-old female who has just been admitted into hospital following a car accident. She is recovering from a hip fracture. Her consultant prescribes her zoledronic acid. Which one of the following action is the most suitable for you to take? Dispense zoledronic acid, no further action is required. Advise Miss J's consultant to add calcium 500 mg daily and vitamin D 400 units daily Zolendronic acid may not be suitable in woman of child bearing age. Contact Miss J's consultant to discuss this further. Advise Miss J to eat before her dosage is given. Dispense an alternative bisphosphonate.

Zolendronic acid may not be suitable in woman of child bearing age. Contact Miss J's consultant to discuss this further. Zolendronic acid has shown toxicity in animal studies and should be avoided in woman of child-bearing potential.

For the following scenarios, select the most likely abbreviation from the list below. The form written on a prescription for xylometazoline hydrochloride nasal drops. o.m. Nocte Pulv. Unguentum narist. i.c. u.a. p.c.

narist. Narist. is the latin term for nasal drops.


Ensembles d'études connexes

Chapter 24 (Level 31-40): Structure and Function of the Liver

View Set

Estimating Square Roots, Rational and Irrational Numbers Vocabulary, The Real Number System, The Real Number System, Repeating Decimals as Fractions, Ordering and Comparing Rational Numbers, Classifying Real Numbers, Classifying numbers

View Set

Sun Salutation C (lunging salutations)

View Set

Principles of Accounting - D074: UNIT 4 & UNIT 5

View Set

Kinn's Chapter 1 The Professional MA and Healthcare Team

View Set

Leadership, Managing and Delegating ch.10 PrepU

View Set

Lee cuidadosamente cada enunciado y selecciona la respuesta correcta.

View Set

SB 8.3-8.5 & 8.6A. 8.7 Cell Cycle/Mitosis/ Cell Cycle Control/Cancer/Cell Death

View Set